HESI Pharmacology Exam Practice

Pataasin ang iyong marka sa homework at exams ngayon gamit ang Quizwiz!

Tamoxifen Citrate

antineoplastic, suppression of tumor growth , reduced incidence of breast cancer

Which action is most important for the nurse to implement during the administration of the antirrhythmic drug adenosine (Adenocard)

apply continuous cardiac monitoring

prescription for a scopolamine patch (Transderm Scop) to prevent motion sickness while on a cruise

apply the patch at least 4 hrs prior to departure

The healthcare provider prescribes digitalis (Digoxin) for a client Dx with congestive heart failure

assess the serum potassium level

208.) A client with myasthenia gravis verbalizes complaints of feeling much weaker than normal. The health care provider plans to implement a diagnostic test to determine if the client is experiencing a myasthenic crisis and administers edrophonium (Enlon). Which of the following would indicate that the client is experiencing a myasthenic crisis? 1. Increasing weakness 2. No change in the condition 3. An increase in muscle spasms 4. A temporary improvement in the condition

auto-define "A client with myasthen..." Rationale: Edrophonium (Enlon) is administered to determine whether the client is reacting to an overdose of a medication (cholinergic crisis) or to an increasing severity of the disease (myasthenic crisis). When the edrophonium (Enlon) injection is given and the condition improves temporarily, the client is in myasthenic crisis. This is known as a positive test. Increasing weakness would occur in cholinergic crisis. Options 2 and 3 would not occur in either crisis.

The nurse is teaching a client with cancer about opioid management for intractable pain and tolerance related side effects. The nurse should prepare the client for which side effect that is most likely to persist during long-term use of opioids? A) Sedation. B) Constipation. C) Urinary retention. D) Respiratory depression.

b. The client should be prepared to implement measures for constipation (B) which is the most likely persistent side effect related to opioid use. Tolerance to opiate narcotics is common, and the client may experience less sedation (A) and respiratory depression (D) as analgesic use continues. Opioids increase the tone in the urinary bladder sphincter, which causes retention (C) but may subside.

•-oxacin

broad spectrum antibiotics

receiving Albuterol (Proventil) tablets complains of nausea every evening with her 9pm

doseadminister the dose with a snack

avoid what with Calcium channel blockers?

grapefruit

Parkinson's Dz and is taking carbidopa-levodopa (Sinemet) indicate that the desired outcome of the medication is being achieved

lessing of tremors

•-caine

local anesthetics

An antacid, (Maalox) is prescribed for a client with peptic ulcer Dz

maintain a gastric pH of 3.5 or above

prescribed atorvastatin (Lipitor) one month ago calls the triage nurse at the clinic complaining of muscle pain and weakness in his legs

make an appointment to see the healthcare provider because muscle pain may be a serious side effect

Medication dosing-heparin 25000 units at 7ml/hr doctor changed rate to 900 units what is the

mls/hr

A healthcare provider prescribes cephalexin monohydrate (Keflex) for a postoperative infection

penicillins

When assessing an adolescent who recently overdosed on acetaminophen (Tylenol), it is most important for the nurse to assess for pain in which area of the body

pentobarbital sodium Nembutal Sodium for sleep

teaching for liver transplant about cyclosporine (Sandimmune) the nurse should encourage the pt to report which adverse response

presence of hand tremors

A nurse is giving morphine 2 mg IV to a pt after surgery and she has followed the "six rights of administration"... possible reactions to morphine are?

resp depression bradycardia urinary retention flushing physical dependence

Which nursing Dx is important to include in the plan of care for a client receiving the angiotension II receptor antagonist irbesartan (Avapro)

risk for injury

receiving methylprednisolone (Solu-Medrol) 40 mg IV daily anticipates an increase in which lab value

serum glucose

•-amil

calcium channel blockers

•-pine

calcium channel blockers

magnesium antidote

calcium gluconate

After abdominal surgery prescribed low molecular weight heparin (LMWH)

this medication is a blood thinner given to prevent blood clot formation

Patient on folliculitis medication-what to teach

Drink with full glass of water

Drug X has a therapeutic index of 10 and drug Y has an index of 2

Drug Y - more potent

Which antidiarrheal agent should be used with caution in clients taking high dosages of aspirin for arthritis

bismuth subsalicylate (Pepto Bismol)

receiving metoprolol (Lopressor SR)

blood pressure

receiving clonidine (Catapres) 0.1 mg q24h via transdermal patch

blood pressure changed from 180/120 to 140/70

The pt takes Oxybutinin (anticholenergic) for OOB takes an OTC antihistamine (anticholenergic) for hay fever

Dry mouth, increased temp, and blurry vision (myosis)

congestive heart failure (CHF) is being discharged with a new Rx for the angiotension-converting enzyme (ACE) inhibitor captopril (Capoten)

dizziness

The nitrate isosorbide denitrate (Isordil) is prescribed for a client with angina

do not get up quickly, always rise slowly

A pt with OCD is prescribed Zoloft - which is not a true statement about the medication

"I will get better in 3 weeks." (full effect in 1 - 2 months, you won't get "better" necessarily)

Pt with depression is proscribed Fluoxitine (Prozac)

"It may take 3-4 weeks before my mood is elevated"

A pt is prescribed a Dopamine agonist (Mirapex) (adrenergic agonist) for Parkinson's

"This med will stop the progression of Parkinson's." - can't cure Parkinson's

A client who has been taking levadopa PO Tid to control the symptoms of Parkinson's disease has a new prescription for sustained-release levadopa/carbidopa (Simemet 25/100) PO Bid and took his levadopa at 0800

"you can begin taking the Sinemet this evening, but do not take any more levadopa"

A pt is prescribed Bethanocol (Muscarinic Agonist) for urinary retention - if pt exhibits signs of overdose such as: increased salivation, sweating, bradycardia, hypotention, the RN would administer

Atropine (anticholenergic - makes us dry)

A client who was prescribed atorvastatin (Lipitor) one month ago calls the triage nurse at the clinic complaining of muscle pain and weakness in his legs. Which statement reflects the correct drug-specific teaching the nurse should provide to this client? A) Increase consumption of potassium-rich foods since low potassium levels can cause muscle spasms. B) Have serum electrolytes checked at the next scheduled appointment to assess hyponatremia, a cause of cramping. C) Make an appointment to see the healthcare provider, because muscle pain may be an indication of a serious side effect. D) Be sure to consume a low-cholesterol diet while taking the drug to enhance the effectiveness of the drug.

. Myopathy, suggested by the leg pain and weakness, is a serious, and potentially life-threatening, complication of Lipitor, and should be evaluated immediately by the healthcare provider (C). Although electrolyte imbalances such as (A or B) can cause muscle spasms in some cases, this is not the likely cause of leg pain in the client receiving Lipitor, and evaluation by the healthcare provider should not be delayed for any reason. A low-cholesterol diet is recommended for those taking Lipitor since the drug is used to lower total cholesterol (D), but diet is not related to the leg pain symptom.

Med was ordered 100mg in 4 divided doses in 24 hours available in 25mg, how many will you give every 6 hours

1

173.) A nurse reviews the medication history of a client admitted to the hospital and notes that the client is taking leflunomide (Arava). During data collection, the nurse asks which question to determine medication effectiveness? 1. "Do you have any joint pain?" 2. "Are you having any diarrhea?" 3. "Do you have frequent headaches?" 4. "Are you experiencing heartburn?"

1. "Do you have any joint pain?" Rationale: Leflunomide is an immunosuppressive agent and has an anti-inflammatory action. The medication provides symptomatic relief of rheumatoid arthritis. Diarrhea can occur as a side effect of the medication. The other options are unrelated to medication effectiveness.

127.) The nurse provides medication instructions to an older hypertensive client who is taking 20 mg of lisinopril (Prinivil, Zestril) orally daily. The nurse evaluates the need for further teaching when the client states which of the following? 1. "I can skip a dose once a week." 2. "I need to change my position slowly." 3. "I take the pill after breakfast each day." 4. "If I get a bad headache, I should call my doctor immediately."

1. "I can skip a dose once a week." Rationale: Lisinopril is an antihypertensive angiotensin-converting enzyme (ACE) inhibitor. The usual dosage range is 20 to 40 mg per day. Adverse effects include headache, dizziness, fatigue, orthostatic hypotension, tachycardia, and angioedema. Specific client teaching points include taking one pill a day, not stopping the medication without consulting the health care provider (HCP), and monitoring for side effects and adverse reactions. The client should notify the HCP if side effects occur.

31.) A community health nurse visits a client at home. Prednisone 10 mg orally daily has been prescribed for the client and the nurse reinforces teaching for the client about the medication. Which statement, if made by the client, indicates that further teaching is necessary? 1. "I can take aspirin or my antihistamine if I need it." 2. "I need to take the medication every day at the same time." 3. "I need to avoid coffee, tea, cola, and chocolate in my diet." 4. "If I gain more than 5 pounds a week, I will call my doctor."

1. "I can take aspirin or my antihistamine if I need it." Rationale: Aspirin and other over-the-counter medications should not be taken unless the client consults with the health care provider (HCP). The client needs to take the medication at the same time every day and should be instructed not to stop the medication. A slight weight gain as a result of an improved appetite is expected, but after the dosage is stabilized, a weight gain of 5 lb or more weekly should be reported to the HCP. Caffeine-containing foods and fluids need to be avoided because they may contribute to steroid-ulcer development.

179.) A nurse provides medication instructions to a client who had a kidney transplant about therapy with cyclosporine (Sandimmune). Which statement by the client indicates a need for further instruction? 1. "I need to obtain a yearly influenza vaccine." 2. "I need to have dental checkups every 3 months." 3. "I need to self-monitor my blood pressure at home." 4. "I need to call the health care provider (HCP) if my urine volume decreases or my urine becomes cloudy."

1. "I need to obtain a yearly influenza vaccine." Rationale: Cyclosporine is an immunosuppressant medication. Because of the medication's effects, the client should not receive any vaccinations without first consulting the HCP. The client should report decreased urine output or cloudy urine, which could indicate kidney rejection or infection, respectively. The client must be able to self-monitor blood pressure to check for the side effect of hypertension. The client needs meticulous oral care and dental cleaning every 3 months to help prevent gingival hyperplasia.

77.) Phenytoin (Dilantin), 100 mg orally three times daily, has been prescribed for a client for seizure control. The nurse reinforces instructions regarding the medication to the client. Which statement by the client indicates an understanding of the instructions? 1. "I will use a soft toothbrush to brush my teeth." 2. "It's all right to break the capsules to make it easier for me to swallow them." 3. "If I forget to take my medication, I can wait until the next dose and eliminate that dose." 4. "If my throat becomes sore, it's a normal effect of the medication and it's nothing to be concerned about."

1. "I will use a soft toothbrush to brush my teeth." Rationale: Phenytoin (Dilantin) is an anticonvulsant. Gingival hyperplasia, bleeding, swelling, and tenderness of the gums can occur with the use of this medication. The client needs to be taught good oral hygiene, gum massage, and the need for regular dentist visits. The client should not skip medication doses, because this could precipitate a seizure. Capsules should not be chewed or broken and they must be swallowed. The client needs to be instructed to report a sore throat, fever, glandular swelling, or any skin reaction, because this indicates hematological toxicity.

220.) A adult client with muscle spasms is taking an oral maintenance dose of baclofen (Lioresal). The nurse reviews the medication record, expecting that which dose should be prescribed? 1. 15 mg four times a day 2. 25 mg four times a day 3. 30 mg four times a day 4. 40 mg four times a day

1. 15 mg four times a day Rationale: Baclofen is dispensed in 10- and 20-mg tablets for oral use. Dosages are low initially and then gradually increased. Maintenance doses range from 15 to 20 mg administered three or four times a day.

115.) A client received 20 units of NPH insulin subcutaneously at 8:00 AM. The nurse should check the client for a potential hypoglycemic reaction at what time? 1. 5:00 PM 2. 10:00 AM 3. 11:00 AM 4. 11:00 PM

1. 5:00 PM Rationale: NPH is intermediate-acting insulin. Its onset of action is 1 to 2½ hours, it peaks in 4 to 12 hours, and its duration of action is 24 hours. Hypoglycemic reactions most likely occur during peak time.

10.) The clinic nurse is performing an admission assessment on a client. The nurse notes that the client is taking azelaic acid (Azelex). Because of the medication prescription, the nurse would suspect that the client is being treated for: 1. Acne 2. Eczema 3. Hair loss 4. Herpes simplex

1. Acne Rationale: Azelaic acid is a topical medication used to treat mild to moderate acne. The acid appears to work by suppressing the growth of Propionibacterium acnes and decreasing the proliferation of keratinocytes. Options 2, 3, and 4 are incorrect.

145.) A nurse has a prescription to give a client albuterol (Proventil HFA) (two puffs) and beclomethasone dipropionate (Qvar) (nasal inhalation, two puffs), by metered-dose inhaler. The nurse administers the medication by giving the: 1. Albuterol first and then the beclomethasone dipropionate 2. Beclomethasone dipropionate first and then the albuterol 3. Alternating a single puff of each, beginning with the albuterol 4. Alternating a single puff of each, beginning with the beclomethasone dipropionate

1. Albuterol first and then the beclomethasone dipropionate Rationale: Albuterol is a bronchodilator. Beclomethasone dipropionate is a glucocorticoid. Bronchodilators are always administered before glucocorticoids when both are to be given on the same time schedule. This allows for widening of the air passages by the bronchodilator, which then makes the glucocorticoid more effective.

26.) Glimepiride (Amaryl) is prescribed for a client with diabetes mellitus. A nurse reinforces instructions for the client and tells the client to avoid which of the following while taking this medication? 1. Alcohol 2. Organ meats 3. Whole-grain cereals 4. Carbonated beverages

1. Alcohol Rationale: When alcohol is combined with glimepiride (Amaryl), a disulfiram-like reaction may occur. This syndrome includes flushing, palpitations, and nausea. Alcohol can also potentiate the hypoglycemic effects of the medication. Clients need to be instructed to avoid alcohol consumption while taking this medication. The items in options 2, 3, and 4 do not need to be avoided.

142.) A health care provider has written a prescription for ranitidine (Zantac), once daily. The nurse should schedule the medication for which of the following times? 1. At bedtime 2. After lunch 3. With supper 4. Before breakfast

1. At bedtime Rationale: A single daily dose of ranitidine is usually scheduled to be given at bedtime. This allows for a prolonged effect, and the greatest protection of the gastric mucosa. **recall that ranitidine suppresses secretions of gastric acids**

100.) Saquinavir (Invirase) is prescribed for the client who is human immunodeficiency virus seropositive. The nurse reinforces medication instructions and tells the client to: 1. Avoid sun exposure. 2. Eat low-calorie foods. 3. Eat foods that are low in fat. 4. Take the medication on an empty stomach.

1. Avoid sun exposure. Rationale: Saquinavir (Invirase) is an antiretroviral (protease inhibitor) used with other antiretroviral medications to manage human immunodeficiency virus infection. Saquinavir is administered with meals and is best absorbed if the client consumes high-calorie, high-fat meals. Saquinavir can cause photosensitivity, and the nurse should instruct the client to avoid sun exposure.

202.) A nurse is collecting data from a client about medications being taken, and the client tells the nurse that he is taking herbal supplements for the treatment of varicose veins. The nurse understands that the client is most likely taking which of the following? 1. Bilberry 2. Ginseng 3. Feverfew 4. Evening primrose

1. Bilberry Rationale: Bilberry is an herbal supplement that has been used to treat varicose veins. This supplement has also been used to treat cataracts, retinopathy, diabetes mellitus, and peripheral vascular disease. Ginseng has been used to improve memory performance and decrease blood glucose levels in type 2 diabetes mellitus. Feverfew is used to prevent migraine headaches and to treat rheumatoid arthritis. Evening primrose is used to treat eczema and skin irritation.

156.) A nurse is reviewing the laboratory results for a client receiving tacrolimus (Prograf). Which laboratory result would indicate to the nurse that the client is experiencing an adverse effect of the medication? 1. Blood glucose of 200 mg/dL 2. Potassium level of 3.8 mEq/L 3. Platelet count of 300,000 cells/mm3 4. White blood cell count of 6000 cells/mm3

1. Blood glucose of 200 mg/dL Rationale: A blood glucose level of 200 mg/dL is elevated above the normal range of 70 to 110 mg/dL and suggests an adverse effect. Other adverse effects include neurotoxicity evidenced by headache, tremor, insomnia; gastrointestinal (GI) effects such as diarrhea, nausea, and vomiting; hypertension; and hyperkalemia.

32.) Desmopressin acetate (DDAVP) is prescribed for the treatment of diabetes insipidus. The nurse monitors the client after medication administration for which therapeutic response? 1. Decreased urinary output 2. Decreased blood pressure 3. Decreased peripheral edema 4. Decreased blood glucose level

1. Decreased urinary output Rationale: Desmopressin promotes renal conservation of water. The hormone carries out this action by acting on the collecting ducts of the kidney to increase their permeability to water, which results in increased water reabsorption. The therapeutic effect of this medication would be manifested by a decreased urine output. Options 2, 3, and 4 are unrelated to the effects of this medication.

105.) A nurse is collecting data from a client and the client's spouse reports that the client is taking donepezil hydrochloride (Aricept). Which disorder would the nurse suspect that this client may have based on the use of this medication? 1. Dementia 2. Schizophrenia 3. Seizure disorder 4. Obsessive-compulsive disorder

1. Dementia Rationale: Donepezil hydrochloride is a cholinergic agent used in the treatment of mild to moderate dementia of the Alzheimer type. It enhances cholinergic functions by increasing the concentration of acetylcholine. It slows the progression of Alzheimer's disease. Options 2, 3, and 4 are incorrect.

33.) The home health care nurse is visiting a client who was recently diagnosed with type 2 diabetes mellitus. The client is prescribed repaglinide (Prandin) and metformin (Glucophage) and asks the nurse to explain these medications. The nurse should reinforce which instructions to the client? Select all that apply. 1. Diarrhea can occur secondary to the metformin. 2. The repaglinide is not taken if a meal is skipped. 3. The repaglinide is taken 30 minutes before eating. 4. Candy or another simple sugar is carried and used to treat mild hypoglycemia episodes. 5. Metformin increases hepatic glucose production to prevent hypoglycemia associated with repaglinide. 6. Muscle pain is an expected side effect of metformin and may be treated with acetaminophen (Tylenol).

1. Diarrhea can occur secondary to the metformin. 2. The repaglinide is not taken if a meal is skipped. 3. The repaglinide is taken 30 minutes before eating. 4. Candy or another simple sugar is carried and used to treat mild hypoglycemia episodes. Rationale: Repaglinide is a rapid-acting oral hypoglycemic agent that stimulates pancreatic insulin secretion that should be taken before meals, and that should be withheld if the client does not eat. Hypoglycemia is a side effect of repaglinide and the client should always be prepared by carrying a simple sugar with her or him at all times. Metformin is an oral hypoglycemic given in combination with repaglinide and works by decreasing hepatic glucose production. A common side effect of metformin is diarrhea. Muscle pain may occur as an adverse effect from metformin but it might signify a more serious condition that warrants health care provider notification, not the use of acetaminophen.

116.) A nurse administers a dose of scopolamine (Transderm-Scop) to a postoperative client. The nurse tells the client to expect which of the following side effects of this medication? 1. Dry mouth 2. Diaphoresis 3. Excessive urination 4. Pupillary constriction

1. Dry mouth Rationale: Scopolamine is an anticholinergic medication for the prevention of nausea and vomiting that causes the frequent side effects of dry mouth, urinary retention, decreased sweating, and dilation of the pupils. The other options describe the opposite effects of cholinergic-blocking agents and therefore are incorrect.

193.) Sodium hypochlorite (Dakin's solution) is prescribed for a client with a leg wound containing purulent drainage. The nurse is assisting in developing a plan of care for the client and includes which of the following in the plan? 1. Ensure that the solution is freshly prepared before use. 2. Soak a sterile dressing with solution and pack into the wound. 3. Allow the solution to remain in the wound following irrigation. 4. Apply the solution to the wound and on normal skin tissue surrounding the wound.

1. Ensure that the solution is freshly prepared before use. Rationale: Dakin solution is a chloride solution that is used for irrigating and cleaning necrotic or purulent wounds. It can be used for packing necrotic wounds. It cannot be used to pack purulent wounds because the solution is inactivated by copious pus. It should not come into contact with healing or normal tissue, and it should be rinsed off immediately if used for irrigation. Solutions are unstable and the nurse must ensure that the solution has been prepared fresh before use. **Eliminate options 2 and 3 first because they are comparable or alike. It makes sense to ensure that the solution is freshly prepared; therefore, select option 1**

102.) A client with human immunodeficiency virus is taking nevirapine (Viramune). The nurse should monitor for which adverse effects of the medication? Select all that apply. 1. Rash 2. Hepatotoxicity 3. Hyperglycemia 4. Peripheral neuropathy 5. Reduced bone mineral density

1. Rash 2. Hepatotoxicity Rationale: Nevirapine (Viramune) is a non-nucleoside reverse transcriptase inhibitors (NRTI) that is used to treat HIV infection. It is used in combination with other antiretroviral medications to treat HIV. Adverse effects include rash, Stevens-Johnson syndrome, hepatitis, and increased transaminase levels. Hyperglycemia, peripheral neuropathy, and reduced bone density are not adverse effects of this medication.

112.) A hospitalized client is started on phenelzine sulfate (Nardil) for the treatment of depression. The nurse instructs the client to avoid consuming which foods while taking this medication? Select all that apply. 1. Figs 2. Yogurt 3. Crackers 4. Aged cheese 5 Tossed salad 6. Oatmeal cookies

1. Figs 2. Yogurt 4. Aged cheese Rationale: Phenelzine sulfate (Nardil) is a monoamine oxidase inhibitor(MAOI). The client should avoid taking in foods that are high in tyramine. Use of these foods could trigger a potentially fatal hypertensive crisis. Some foods to avoid include yogurt, aged cheeses, smoked or processed meats, red wines, and fruits such as avocados, raisins, and figs.

93.) The client who is human immunodeficiency virus seropositive has been taking stavudine (d4t, Zerit). The nurse monitors which of the following most closely while the client is taking this medication? 1. Gait 2. Appetite 3. Level of consciousness 4. Hemoglobin and hematocrit blood levels

1. Gait Rationale: Stavudine (d4t, Zerit) is an antiretroviral used to manage human immunodeficiency virus infection in clients who do not respond to or who cannot tolerate conventional therapy. The medication can cause peripheral neuropathy, and the nurse should monitor the client's gait closely and ask the client about paresthesia. Options 2, 3, and 4 are unrelated to the use of the medication.

91.) Cyclobenzaprine (Flexeril) is prescribed for a client to treat muscle spasms, and the nurse is reviewing the client's record. Which of the following disorders, if noted in the client's record, would indicate a need to contact the health care provider regarding the administration of this medication? 1. Glaucoma 2. Emphysema 3. Hyperthyroidism 4. Diabetes mellitus

1. Glaucoma Rationale: Because this medication has anticholinergic effects, it should be used with caution in clients with a history of urinary retention, angle-closure glaucoma, and increased intraocular pressure. Cyclobenzaprine hydrochloride should be used only for short-term 2- to 3-week therapy.

125.) A nurse is preparing to administer digoxin (Lanoxin), 0.125 mg orally, to a client with heart failure. Which vital sign is most important for the nurse to check before administering the medication? 1. Heart rate 2. Temperature 3. Respirations 4. Blood pressure

1. Heart rate Rationale: Digoxin is a cardiac glycoside that is used to treat heart failure and acts by increasing the force of myocardial contraction. Because bradycardia may be a clinical sign of toxicity, the nurse counts the apical heart rate for 1 full minute before administering the medication. If the pulse rate is less than 60 beats/minute in an adult client, the nurse would withhold the medication and report the pulse rate to the registered nurse, who would then contact the health care provider.

158.) A client with chronic renal failure is receiving epoetin alfa (Epogen, Procrit). Which laboratory result would indicate a therapeutic effect of the medication? 1. Hematocrit of 32% 2. Platelet count of 400,000 cells/mm3 3. White blood cell count of 6000 cells/mm3 4. Blood urea nitrogen (BUN) level of 15 mg/dL

1. Hematocrit of 32% Rationale: Epoetin alfa is used to reverse anemia associated with chronic renal failure. A therapeutic effect is seen when the hematocrit is between 30% and 33%. The laboratory tests noted in the other options are unrelated to the use of this medication.

167.) A nurse prepares to reinforce instructions to a client who is taking allopurinol (Zyloprim). The nurse plans to include which of the following in the instructions? 1. Instruct the client to drink 3000 mL of fluid per day. 2. Instruct the client to take the medication on an empty stomach. 3. Inform the client that the effect of the medication will occur immediately. 4. Instruct the client that, if swelling of the lips occurs, this is a normal expected response.

1. Instruct the client to drink 3000 mL of fluid per day. Rationale: Allopurinol (Zyloprim) is an antigout medication used to decrease uric acid levels. Clients taking allopurinol are encouraged to drink 3000 mL of fluid a day. A full therapeutic effect may take 1 week or longer. Allopurinol is to be given with or immediately following meals or milk to prevent gastrointestinal irritation. If the client develops a rash, irritation of the eyes, or swelling of the lips or mouth, he or she should contact the health care provider because this may indicate hypersensitivity.

113.) A nurse is reinforcing discharge instructions to a client receiving sulfisoxazole. Which of the following would be included in the plan of care for instructions? 1. Maintain a high fluid intake. 2. Discontinue the medication when feeling better. 3. If the urine turns dark brown, call the health care provider immediately. 4. Decrease the dosage when symptoms are improving to prevent an allergic response.

1. Maintain a high fluid intake. Rationale: Each dose of sulfisoxazole should be administered with a full glass of water, and the client should maintain a high fluid intake. The medication is more soluble in alkaline urine. The client should not be instructed to taper or discontinue the dose. Some forms of sulfisoxazole cause the urine to turn dark brown or red. This does not indicate the need to notify the health care provider.

43.) A histamine (H2)-receptor antagonist will be prescribed for a client. The nurse understands that which medications are H2-receptor antagonists? Select all that apply. 1. Nizatidine (Axid) 2. Ranitidine (Zantac) 3. Famotidine (Pepcid) 4. Cimetidine (Tagamet) 5. Esomeprazole (Nexium) 6. Lansoprazole (Prevacid)

1. Nizatidine (Axid) 2. Ranitidine (Zantac) 3. Famotidine (Pepcid) 4. Cimetidine (Tagamet) Rationale: H2-receptor antagonists suppress secretion of gastric acid, alleviate symptoms of heartburn, and assist in preventing complications of peptic ulcer disease. These medications also suppress gastric acid secretions and are used in active ulcer disease, erosive esophagitis, and pathological hypersecretory conditions. The other medications listed are proton pump inhibitors. H2-receptor antagonists medication names end with -dine. Proton pump inhibitors medication names end with -zole.

109.) A client taking buspirone (BuSpar) for 1 month returns to the clinic for a follow-up visit. Which of the following would indicate medication effectiveness? 1. No rapid heartbeats or anxiety 2. No paranoid thought processes 3. No thought broadcasting or delusions 4. No reports of alcohol withdrawal symptoms

1. No rapid heartbeats or anxiety Rationale: Buspirone hydrochloride is not recommended for the treatment of drug or alcohol withdrawal, paranoid thought disorders, or schizophrenia (thought broadcasting or delusions). Buspirone hydrochloride is most often indicated for the treatment of anxiety and aggression.

12.) A nurse is caring for a client who is receiving an intravenous (IV) infusion of an antineoplastic medication. During the infusion, the client complains of pain at the insertion site. During an inspection of the site, the nurse notes redness and swelling and that the rate of infusion of the medication has slowed. The nurse should take which appropriate action? 1. Notify the registered nurse. 2. Administer pain medication to reduce the discomfort. 3. Apply ice and maintain the infusion rate, as prescribed. 4. Elevate the extremity of the IV site, and slow the infusion.

1. Notify the registered nurse. Rationale: When antineoplastic medications (Chemotheraputic Agents) are administered via IV, great care must be taken to prevent the medication from escaping into the tissues surrounding the injection site, because pain, tissue damage, and necrosis can result. The nurse monitors for signs of extravasation, such as redness or swelling at the insertion site and a decreased infusion rate. If extravasation occurs, the registered nurse needs to be notified; he or she will then contact the health care provider.

235.) A tricyclic antidepressant is administered to a client daily. The nurse plans to monitor for the common side effects of the medication and includes which of the following in the plan of care? 1. Offer hard candy or gum periodically. 2. Offer a nutritious snack between meals. 3. Monitor the blood pressure every 2 hours. 4. Review the white blood cell (WBC) count results daily.

1. Offer hard candy or gum periodically. Rationale: Dry mouth is a common side effect of tricyclic antidepressants. Frequent mouth rinsing with water, sucking on hard candy, and chewing gum will alleviate this common side effect. It is not necessary to monitor the blood pressure every 2 hours. In addition, it is not necessary to check the WBC daily. Weight gain is a common side effect and frequent snacks will aggravate this problem.

18.) The nurse is reviewing the history and physical examination of a client who will be receiving asparaginase (Elspar), an antineoplastic agent. The nurse consults with the registered nurse regarding the administration of the medication if which of the following is documented in the client's history? 1. Pancreatitis 2. Diabetes mellitus 3. Myocardial infarction 4. Chronic obstructive pulmonary disease

1. Pancreatitis Rationale: Asparaginase (Elspar) is contraindicated if hypersensitivity exists, in pancreatitis, or if the client has a history of pancreatitis. The medication impairs pancreatic function and pancreatic function tests should be performed before therapy begins and when a week or more has elapsed between administration of the doses. The client needs to be monitored for signs of pancreatitis, which include nausea, vomiting, and abdominal pain. The conditions noted in options 2, 3, and 4 are not contraindicated with this medication.

171.) A nurse is preparing to administer furosemide (Lasix) to a client with a diagnosis of heart failure. The most important laboratory test result for the nurse to check before administering this medication is: 1. Potassium level 2. Creatinine level 3. Cholesterol level 4. Blood urea nitrogen

1. Potassium level Rationale: Furosemide is a loop diuretic. The medication causes a decrease in the client's electrolytes, especially potassium, sodium, and chloride. Administering furosemide to a client with low electrolyte levels could precipitate ventricular dysrhythmias. Options 2 and 4 reflect renal function. The cholesterol level is unrelated to the administration of this medication.

30.) A client with diabetes mellitus visits a health care clinic. The client's diabetes mellitus previously had been well controlled with glyburide (DiaBeta) daily, but recently the fasting blood glucose level has been 180 to 200 mg/dL. Which medication, if added to the client's regimen, may have contributed to the hyperglycemia? 1. Prednisone 2. Phenelzine (Nardil) 3. Atenolol (Tenormin) 4. Allopurinol (Zyloprim)

1. Prednisone Rationale: Prednisone may decrease the effect of oral hypoglycemics, insulin, diuretics, and potassium supplements. Option 2, a monoamine oxidase inhibitor, and option 3, a β-blocker, have their own intrinsic hypoglycemic activity. Option 4 decreases urinary excretion of sulfonylurea agents, causing increased levels of the oral agents, which can lead to hypoglycemia.

152.) Intravenous heparin therapy is prescribed for a client. While implementing this prescription, a nurse ensures that which of the following medications is available on the nursing unit? 1. Protamine sulfate 2. Potassium chloride 3. Phytonadione (vitamin K ) 4. Aminocaproic acid (Amicar)

1. Protamine sulfate Rationale: The antidote to heparin is protamine sulfate; it should be readily available for use if excessive bleeding or hemorrhage occurs. Potassium chloride is administered for a potassium deficit. Vitamin K is an antidote for warfarin sodium. Aminocaproic acid is the antidote for thrombolytic therapy.

72.) Cinoxacin (Cinobac), a urinary antiseptic, is prescribed for the client. The nurse reviews the client's medical record and should contact the health care provider (HCP) regarding which documented finding to verify the prescription? Refer to chart. 1. Renal insufficiency 2. Chest x-ray: normal 3. Blood glucose, 102 mg/dL 4. Folic acid (vitamin B6) 0.5 mg, orally daily

1. Renal insufficiency Rationale: Cinoxacin should be administered with caution in clients with renal impairment. The dosage should be reduced, and failure to do so could result in accumulation of cinoxacin to toxic levels. Therefore the nurse would verify the prescription if the client had a documented history of renal insufficiency. The laboratory and diagnostic test results are normal findings. Folic acid (vitamin B6) may be prescribed for a client with renal insufficiency to prevent anemia.

90.) A nurse is reviewing the record of a client who has been prescribed baclofen (Lioresal). Which of the following disorders, if noted in the client's history, would alert the nurse to contact the health care provider? 1. Seizure disorders 2. Hyperthyroidism 3. Diabetes mellitus 4. Coronary artery disease

1. Seizure disorders Rationale: Clients with seizure disorders may have a lowered seizure threshold when baclofen is administered. Concurrent therapy may require an increase in the anticonvulsive medication. The disorders in options 2, 3, and 4 are not a concern when the client is taking baclofen.

53.) Rifabutin (Mycobutin) is prescribed for a client with active Mycobacterium avium complex (MAC) disease and tuberculosis. The nurse monitors for which side effects of the medication? Select all that apply. 1. Signs of hepatitis 2. Flu-like syndrome 3. Low neutrophil count 4. Vitamin B6 deficiency 5. Ocular pain or blurred vision 6. Tingling and numbness of the fingers

1. Signs of hepatitis 2. Flu-like syndrome 3. Low neutrophil count 5. Ocular pain or blurred vision Rationale: Rifabutin (Mycobutin) may be prescribed for a client with active MAC disease and tuberculosis. It inhibits mycobacterial DNA-dependent RNA polymerase and suppresses protein synthesis. Side effects include rash, gastrointestinal disturbances, neutropenia (low neutrophil count), red-orange body secretions, uveitis (blurred vision and eye pain), myositis, arthralgia, hepatitis, chest pain with dyspnea, and flu-like syndrome. Vitamin B6 deficiency and numbness and tingling in the extremities are associated with the use of isoniazid (INH). Ethambutol (Myambutol) also causes peripheral neuritis.

92.) In monitoring a client's response to disease-modifying antirheumatic drugs (DMARDs), which findings would the nurse interpret as acceptable responses? Select all that apply. 1. Symptom control during periods of emotional stress 2. Normal white blood cell counts, platelet, and neutrophil counts 3. Radiological findings that show nonprogression of joint degeneration 4. An increased range of motion in the affected joints 3 months into therapy 5. Inflammation and irritation at the injection site 3 days after injection is given 6. A low-grade temperature upon rising in the morning that remains throughout the day

1. Symptom control during periods of emotional stress 2. Normal white blood cell counts, platelet, and neutrophil counts 3. Radiological findings that show nonprogression of joint degeneration 4. An increased range of motion in the affected joints 3 months into therapy Rationale: Because emotional stress frequently exacerbates the symptoms of rheumatoid arthritis, the absence of symptoms is a positive finding. DMARDs are given to slow progression of joint degeneration. In addition, the improvement in the range of motion after 3 months of therapy with normal blood work is a positive finding. Temperature elevation and inflammation and irritation at the medication injection site could indicate signs of infection.

149.) A client taking fexofenadine (Allegra) is scheduled for allergy skin testing and tells the nurse in the health care provider's office that a dose was taken this morning. The nurse determines that: 1. The client should reschedule the appointment. 2. A lower dose of allergen will need to be injected. 3. A higher dose of allergen will need to be injected. 4. The client should have the skin test read a day later than usual.

1. The client should reschedule the appointment. Rationale: Fexofenadine is an antihistamine, which provides relief of symptoms caused by allergy. Antihistamines should be discontinued for at least 3 days (72 hours) before allergy skin testing to avoid false-negative readings. This client should have the appointment rescheduled for 3 days after discontinuing the medication.

221.) A nurse is reviewing the health care provider's prescriptions for an adult client who has been admitted to the hospital following a back injury. Carisoprodol (Soma) is prescribed for the client to relieve the muscle spasms; the health care provider has prescribed 350 mg to be administered four times a day. When preparing to give this medication, the nurse determines that this dosage is: 1. The normal adult dosage 2. A lower than normal dosage 3. A higher than normal dosage 4. A dosage requiring further clarification

1. The normal adult dosage Rationale: The normal adult dosage for carisoprodol is 350 mg orally three or four times daily.

21.) A nurse is assisting with caring for a client with cancer who is receiving cisplatin. Select the adverse effects that the nurse monitors for that are associated with this medication. Select all that apply. 1. Tinnitus 2. Ototoxicity 3. Hyperkalemia 4. Hypercalcemia 5. Nephrotoxicity 6. Hypomagnesemia

1. Tinnitus 2. Ototoxicity 5. Nephrotoxicity 6. Hypomagnesemia Rationale: Cisplatin is an alkylating medication. Alkylating medications are cell cycle phase-nonspecific medications that affect the synthesis of DNA by causing the cross-linking of DNA to inhibit cell reproduction. Cisplatin may cause ototoxicity, tinnitus, hypokalemia, hypocalcemia, hypomagnesemia, and nephrotoxicity. Amifostine (Ethyol) may be administered before cisplatin to reduce the potential for renal toxicity.

213.) A client is admitted to the hospital with complaints of back spasms. The client states, "I have been taking two or three aspirin every 4 hours for the past week and it hasn't helped my back." Aspirin intoxication is suspected. Which of the following complaints would indicate aspirin intoxication? 1. Tinnitus 2. Constipation 3. Photosensitivity 4. Abdominal cramps

1. Tinnitus Rationale: Mild intoxication with acetylsalicylic acid (aspirin) is called salicylism and is commonly experienced when the daily dosage is higher than 4 g. Tinnitus (ringing in the ears) is the most frequently occurring effect noted with intoxication. Hyperventilation may occur because salicylate stimulates the respiratory center. Fever may result because salicylate interferes with the metabolic pathways involved with oxygen consumption and heat production. Options 2, 3, and 4 are incorrect.

3.) Salicylic acid is prescribed for a client with a diagnosis of psoriasis. The nurse monitors the client, knowing that which of the following would indicate the presence of systemic toxicity from this medication? 1. Tinnitus 2. Diarrhea 3. Constipation 4. Decreased respirations

1. Tinnitus Rationale: Salicylic acid is absorbed readily through the skin, and systemic toxicity (salicylism) can result. Symptoms include tinnitus, dizziness, hyperpnea, and psychological disturbances. Constipation and diarrhea are not associated with salicylism.

110.) A client taking lithium carbonate (Lithobid) reports vomiting, abdominal pain, diarrhea, blurred vision, tinnitus, and tremors. The lithium level is checked as a part of the routine follow-up and the level is 3.0 mEq/L. The nurse knows that this level is: 1. Toxic 2. Normal 3. Slightly above normal 4. Excessively below normal

1. Toxic Rationale: The therapeutic serum level of lithium is 0.6 to 1.2 mEq/L. A level of 3 mEq/L indicates toxicity.

8.) A client with severe acne is seen in the clinic and the health care provider (HCP) prescribes isotretinoin. The nurse reviews the client's medication record and would contact the (HCP) if the client is taking which medication? 1. Vitamin A 2. Digoxin (Lanoxin) 3. Furosemide (Lasix) 4. Phenytoin (Dilantin)

1. Vitamin A Rationale: Isotretinoin is a metabolite of vitamin A and can produce generalized intensification of isotretinoin toxicity. Because of the potential for increased toxicity, vitamin A supplements should be discontinued before isotretinoin therapy. Options 2, 3, and 4 are not contraindicated with the use of isotretinoin.

239.) Which of the following precautions will the nurse specifically take during the administration of ribavirin (Virazole) to a child with respiratory syncytial virus (RSV)? 1. Wearing goggles 2. Wearing a gown 3. Wearing a gown and a mask 4. Handwashing before administration

1. Wearing goggles Rationale: Some caregivers experience headaches, burning nasal passages and eyes, and crystallization of soft contact lenses as a result of administration of ribavirin. Specific to this medication is the use of goggles. A gown is not necessary. A mask may be worn. Handwashing is to be performed before and after any child contact.

79.) Ibuprofen (Advil) is prescribed for a client. The nurse tells the client to take the medication: 1. With 8 oz of milk 2. In the morning after arising 3. 60 minutes before breakfast 4. At bedtime on an empty stomach

1. With 8 oz of milk Rationale: Ibuprofen is a nonsteroidal anti-inflammatory drug (NSAID). NSAIDs should be given with milk or food to prevent gastrointestinal irritation. Options 2, 3, and 4 are incorrect.

24.) A nurse is reinforcing teaching for a client regarding how to mix regular insulin and NPH insulin in the same syringe. Which of the following actions, if performed by the client, indicates the need for further teaching? 1. Withdraws the NPH insulin first 2. Withdraws the regular insulin first 3. Injects air into NPH insulin vial first 4. Injects an amount of air equal to the desired dose of insulin into the vial

1. Withdraws the NPH insulin first Rationale: When preparing a mixture of regular insulin with another insulin preparation, the regular insulin is drawn into the syringe first. This sequence will avoid contaminating the vial of regular insulin with insulin of another type. Options 2, 3, and 4 identify the correct actions for preparing NPH and regular insulin.

195.) A nurse is caring for a client who is taking metoprolol (Lopressor). The nurse measures the client's blood pressure (BP) and apical pulse (AP) immediately before administration. The client's BP is 122/78 mm/Hg and the AP is 58 beats/min. Based on this data, which of the following is the appropriate action? 1. Withhold the medication. 2. Notify the registered nurse immediately. 3. Administer the medication as prescribed. 4. Administer half of the prescribed medication.

1. Withhold the medication. Rationale: Metoprolol (Lopressor) is classified as a beta-adrenergic blocker and is used in the treatment of hypertension, angina, and myocardial infarction. Baseline nursing assessments include measurement of BP and AP immediately before administration. If the systolic BP is below 90 mm/Hg and the AP is below 60 beats/min, the nurse should withhold the medication and document this action. Although the registered nurse should be informed of the client's vital signs, it is not necessary to do so immediately. The medication should not be administered because the data is outside of the prescribed parameters for this medication. The nurse should not administer half of the medication, or alter any dosages at any point in time.

234.) A hospitalized client is started on phenelzine sulfate (Nardil) for the treatment of depression. At lunchtime, a tray is delivered to the client. Which food item on the tray will the nurse remove? 1. Yogurt 2. Crackers 3. Tossed salad 4. Oatmeal cookies

1. Yogurt Rationale: Phenelzine sulfate is a monoamine oxidase inhibitor (MAOI). The client should avoid taking in foods that are high in tyramine. These foods could trigger a potentially fatal hypertensive crisis. Foods to avoid include yogurt, aged cheeses, smoked or processed meats, red wines, and fruits such as avocados, raisins, or figs.

6.) The burn client is receiving treatments of topical mafenide acetate (Sulfamylon) to the site of injury. The nurse monitors the client, knowing that which of the following indicates that a systemic effect has occurred? 1.Hyperventilation 2.Elevated blood pressure 3.Local pain at the burn site 4.Local rash at the burn site

1.Hyperventilation Rationale: Mafenide acetate is a carbonic anhydrase inhibitor and can suppress renal excretion of acid, thereby causing acidosis. Clients receiving this treatment should be monitored for signs of an acid-base imbalance (hyperventilation). If this occurs, the medication should be discontinued for 1 to 2 days. Options 3 and 4 describe local rather than systemic effects. An elevated blood pressure may be expected from the pain that occurs with a burn injury.

A pt with depression is prescribed an antidepressant - the medication will reach full therapeutic effect

2 to 3 weeks (more like 1 to 2 months but this is the best response)

176.) A nurse notes that a client is taking lansoprazole (Prevacid). On data collection, the nurse asks which question to determine medication effectiveness? 1. "Has your appetite increased?" 2. "Are you experiencing any heartburn?" 3. "Do you have any problems with vision?" 4. "Do you experience any leg pain when walking?"

2. "Are you experiencing any heartburn?" Rationale: Lansoprazole is a gastric acid pump inhibitor used to treat gastric and duodenal ulcers, erosive esophagitis, and hypersecretory conditions. It also is used to treat gastroesophageal reflux disease (GERD). It is not used to treat visual problems, problems with appetite, or leg pain. **NOTE: "-zole" refers to gastric acid pump inhibitors**

183.) A client who received a kidney transplant is taking azathioprine (Imuran), and the nurse provides instructions about the medication. Which statement by the client indicates a need for further instructions? 1. "I need to watch for signs of infection." 2. "I need to discontinue the medication after 14 days of use." 3. "I can take the medication with meals to minimize nausea." 4. "I need to call the health care provider (HCP) if more than one dose is missed."

2. "I need to discontinue the medication after 14 days of use." Rationale: Azathioprine is an immunosuppressant medication that is taken for life. Because of the effects of the medication, the client must watch for signs of infection, which are reported immediately to the HCP. The client should also call the HCP if more than one dose is missed. The medication may be taken with meals to minimize nausea.

194.) A nurse provides instructions to a client regarding the use of tretinoin (Retin-A). Which statement by the client indicates the need for further instructions? 1. "Optimal results will be seen after 6 weeks." 2. "I should apply a very thin layer to my skin." 3. "I should wash my hands thoroughly after applying the medication." 4. "I should cleanse my skin thoroughly before applying the medication."

2. "I should apply a very thin layer to my skin." Rationale: Tretinoin is applied liberally to the skin. The hands are washed thoroughly immediately after applying. Therapeutic results should be seen after 2 to 3 weeks but may not be optimal until after 6 weeks. The skin needs to be cleansed thoroughly before applying the medication.

78.) A client is taking phenytoin (Dilantin) for seizure control and a sample for a serum drug level is drawn. Which of the following indicates a therapeutic serum drug range? 1. 5 to 10 mcg/mL 2. 10 to 20 mcg/mL 3. 20 to 30 mcg/mL 4. 30 to 40 mcg/mL

2. 10 to 20 mcg/mL Rationale: The therapeutic serum drug level range for phenytoin (Dilantin) is 10 to 20 mcg/mL. ** A helpful hint may be to remember that the theophylline therapeutic range and the acetaminophen (Tylenol) therapeutic range are the same as the phenytoin (Dilantin) therapeutic range.**

215.) A client with rheumatoid arthritis is taking acetylsalicylic acid (aspirin) on a daily basis. Which medication dose should the nurse expect the client to be taking? 1. 1 g daily 2. 4 g daily 3. 325 mg daily 4. 1000 mg daily

2. 4 g daily Rationale: Aspirin may be used to treat the client with rheumatoid arthritis. It may also be used to reduce the risk of recurrent transient ischemic attack (TIA) or brain attack (stroke) or reduce the risk of myocardial infarction (MI) in clients with unstable angina or a history of a previous MI. The normal dose for clients being treated with aspirin to decrease thrombosis and MI is 300 to 325 mg/day. Clients being treated to prevent TIAs are usually prescribed 1.3 g/day in two to four divided doses. Clients with rheumatoid arthritis are treated with 3.6 to 5.4 g/day in divided doses. **Eliminate options 1 and 4 because they are alike**

29.) A client is taking Humulin NPH insulin daily every morning. The nurse reinforces instructions for the client and tells the client that the most likely time for a hypoglycemic reaction to occur is: 1. 2 to 4 hours after administration 2. 4 to 12 hours after administration 3. 16 to 18 hours after administration 4. 18 to 24 hours after administration

2. 4 to 12 hours after administration Rationale: Humulin NPH is an intermediate-acting insulin. The onset of action is 1.5 hours, it peaks in 4 to 12 hours, and its duration of action is 24 hours. Hypoglycemic reactions most likely occur during peak time.

64.) Nalidixic acid (NegGram) is prescribed for a client with a urinary tract infection. On review of the client's record, the nurse notes that the client is taking warfarin sodium (Coumadin) daily. Which prescription should the nurse anticipate for this client? 1. Discontinuation of warfarin sodium (Coumadin) 2. A decrease in the warfarin sodium (Coumadin) dosage 3. An increase in the warfarin sodium (Coumadin) dosage 4. A decrease in the usual dose of nalidixic acid (NegGram)

2. A decrease in the warfarin sodium (Coumadin) dosage Rationale: Nalidixic acid can intensify the effects of oral anticoagulants by displacing these agents from binding sites on plasma protein. When an oral anticoagulant is combined with nalidixic acid, a decrease in the anticoagulant dosage may be needed.

88.) Dantrolene sodium (Dantrium) is prescribed for a client experiencing flexor spasms, and the client asks the nurse about the action of the medication. The nurse responds, knowing that the therapeutic action of this medication is which of the following? 1. Depresses spinal reflexes 2. Acts directly on the skeletal muscle to relieve spasticity 3. Acts within the spinal cord to suppress hyperactive reflexes 4. Acts on the central nervous system (CNS) to suppress spasms

2. Acts directly on the skeletal muscle to relieve spasticity Rationale: Dantrium acts directly on skeletal muscle to relieve muscle spasticity. The primary action is the suppression of calcium release from the sarcoplasmic reticulum. This in turn decreases the ability of the skeletal muscle to contract. **Options 1, 3, and 4 are all comparable or alike in that they address CNS suppression and the depression of reflexes. Therefore, eliminate these options.**

74.) A client with myasthenia gravis is receiving pyridostigmine (Mestinon). The nurse monitors for signs and symptoms of cholinergic crisis caused by overdose of the medication. The nurse checks the medication supply to ensure that which medication is available for administration if a cholinergic crisis occurs? 1. Vitamin K 2. Atropine sulfate 3. Protamine sulfate 4. Acetylcysteine (Mucomyst)

2. Atropine sulfate Rationale: The antidote for cholinergic crisis is atropine sulfate. Vitamin K is the antidote for warfarin (Coumadin). Protamine sulfate is the antidote for heparin, and acetylcysteine (Mucomyst) is the antidote for acetaminophen (Tylenol).

86.) A nurse is reinforcing discharge instructions to a client receiving baclofen (Lioresal). Which of the following would the nurse include in the instructions? 1. Restrict fluid intake. 2. Avoid the use of alcohol. 3. Stop the medication if diarrhea occurs. 4. Notify the health care provider if fatigue occurs.

2. Avoid the use of alcohol. Rationale: Baclofen is a central nervous system (CNS) depressant. The client should be cautioned against the use of alcohol and other CNS depressants, because baclofen potentiates the depressant activity of these agents. Constipation rather than diarrhea is an adverse effect of baclofen. It is not necessary to restrict fluids, but the client should be warned that urinary retention can occur. Fatigue is related to a CNS effect that is most intense during the early phase of therapy and diminishes with continued medication use. It is not necessary that the client notify the health care provider if fatigue occurs.

9.) The nurse is applying a topical corticosteroid to a client with eczema. The nurse would monitor for the potential for increased systemic absorption of the medication if the medication were being applied to which of the following body areas? 1. Back 2. Axilla 3. Soles of the feet 4. Palms of the hands

2. Axilla Rationale: Topical corticosteroids can be absorbed into the systemic circulation. Absorption is higher from regions where the skin is especially permeable (scalp, axilla, face, eyelids, neck, perineum, genitalia), and lower from regions in which permeability is poor (back, palms, soles).

199.) A nurse is applying a topical glucocorticoid to a client with eczema. The nurse monitors for systemic absorption of the medication if the medication is being applied to which of the following body areas? 1. Back 2. Axilla 3. Soles of the feet 4. Palms of the hands

2. Axilla Rationale: Topical glucocorticoids can be absorbed into the systemic circulation. Absorption is higher from regions where the skin is especially permeable (scalp, axillae, face, eyelids, neck, perineum, genitalia), and lower from regions where penetrability is poor (back, palms, soles). **Eliminate options 3 and 4 because these body areas are similar in terms of skin characteristics**

123.) A nurse is planning to administer amlodipine (Norvasc) to a client. The nurse plans to check which of the following before giving the medication? 1. Respiratory rate 2. Blood pressure and heart rate 3. Heart rate and respiratory rate 4. Level of consciousness and blood pressure

2. Blood pressure and heart rate Rationale: Amlodipine is a calcium channel blocker. This medication decreases the rate and force of cardiac contraction. Before administering a calcium channel blocking agent, the nurse should check the blood pressure and heart rate, which could both decrease in response to the action of this medication. This action will help to prevent or identify early problems related to decreased cardiac contractility, heart rate, and conduction. **amlodipine is a calcium channel blocker, and this group of medications decreases the rate and force of cardiac contraction. This in turn lowers the pulse rate and blood pressure.**

224.) Neuroleptic malignant syndrome is suspected in a client who is taking chlorpromazine. Which medication would the nurse prepare in anticipation of being prescribed to treat this adverse effect related to the use of chlorpromazine? 1. Protamine sulfate 2. Bromocriptine (Parlodel) 3. Phytonadione (vitamin K) 4. Enalapril maleate (Vasotec)

2. Bromocriptine (Parlodel) Rationale: Bromocriptine is an antiparkinsonian prolactin inhibitor used in the treatment of neuroleptic malignant syndrome. Vitamin K is the antidote for warfarin (Coumadin) overdose. Protamine sulfate is the antidote for heparin overdose. Enalapril maleate is an antihypertensive used in the treatment of hypertension.

20.) The client with metastatic breast cancer is receiving tamoxifen. The nurse specifically monitors which laboratory value while the client is taking this medication? 1. Glucose level 2. Calcium level 3. Potassium level 4. Prothrombin time

2. Calcium level Rationale: Tamoxifen may increase calcium, cholesterol, and triglyceride levels. Before the initiation of therapy, a complete blood count, platelet count, and serum calcium levels should be assessed. These blood levels, along with cholesterol and triglyceride levels, should be monitored periodically during therapy. The nurse should assess for hypercalcemia while the client is taking this medication. Signs of hypercalcemia include increased urine volume, excessive thirst, nausea, vomiting, constipation, hypotonicity of muscles, and deep bone and flank pain.

170.) Atenolol hydrochloride (Tenormin) is prescribed for a hospitalized client. The nurse should perform which of the following as a priority action before administering the medication? 1. Listen to the client's lung sounds. 2. Check the client's blood pressure. 3. Check the recent electrolyte levels. 4. Assess the client for muscle weakness.

2. Check the client's blood pressure. Rationale: Atenolol hydrochloride is a beta-blocker used to treat hypertension. Therefore the priority nursing action before administration of the medication is to check the client's blood pressure. The nurse also checks the client's apical heart rate. If the systolic blood pressure is below 90 mm Hg or the apical pulse is 60 beats per minute or lower, the medication is withheld and the registered nurse and/or health care provider is notified. The nurse would check baseline renal and liver function tests. The medication may cause weakness, and the nurse would assist the client with activities if weakness occurs. **Beta-blockers have "-lol" at the end of the medication name**

34.) A client with Crohn's disease is scheduled to receive an infusion of infliximab (Remicade). The nurse assisting in caring for the client should take which action to monitor the effectiveness of treatment? 1. Monitoring the leukocyte count for 2 days after the infusion 2. Checking the frequency and consistency of bowel movements 3. Checking serum liver enzyme levels before and after the infusion 4. Carrying out a Hematest on gastric fluids after the infusion is completed

2. Checking the frequency and consistency of bowel movements Rationale: The principal manifestations of Crohn's disease are diarrhea and abdominal pain. Infliximab (Remicade) is an immunomodulator that reduces the degree of inflammation in the colon, thereby reducing the diarrhea. Options 1, 3, and 4 are unrelated to this medication.

146.) A client has begun therapy with theophylline (Theo-24). The nurse tells the client to limit the intake of which of the following while taking this medication? 1. Oranges and pineapple 2. Coffee, cola, and chocolate 3. Oysters, lobster, and shrimp 4. Cottage cheese, cream cheese, and dairy creamers

2. Coffee, cola, and chocolate Rationale: Theophylline is a xanthine bronchodilator. The nurse teaches the client to limit the intake of xanthine-containing foods while taking this medication. These include coffee, cola, and chocolate.

63.) A client with coronary artery disease complains of substernal chest pain. After checking the client's heart rate and blood pressure, a nurse administers nitroglycerin, 0.4 mg, sublingually. After 5 minutes, the client states, "My chest still hurts." Select the appropriate actions that the nurse should take. Select all that apply. 1. Call a code blue. 2. Contact the registered nurse. 3. Contact the client's family. 4. Assess the client's pain level. 5. Check the client's blood pressure. 6. Administer a second nitroglycerin, 0.4 mg, sublingually.

2. Contact the registered nurse. 4. Assess the client's pain level. 5. Check the client's blood pressure. 6. Administer a second nitroglycerin, 0.4 mg, sublingually. Rationale: The usual guideline for administering nitroglycerin tablets for a hospitalized client with chest pain is to administer one tablet every 5 minutes PRN for chest pain, for a total dose of three tablets. The registered nurse should be notified of the client's condition, who will then notify the health care provider as appropriate. Because the client is still complaining of chest pain, the nurse would administer a second nitroglycerin tablet. The nurse would assess the client's pain level and check the client's blood pressure before administering each nitroglycerin dose. There are no data in the question that indicate the need to call a code blue. In addition, it is not necessary to contact the client's family unless the client has requested this.

84.) Baclofen (Lioresal) is prescribed for the client with multiple sclerosis. The nurse assists in planning care, knowing that the primary therapeutic effect of this medication is which of the following? 1. Increased muscle tone 2. Decreased muscle spasms 3. Increased range of motion 4. Decreased local pain and tenderness

2. Decreased muscle spasms Rationale: Baclofen is a skeletal muscle relaxant and central nervous system depressant and acts at the spinal cord level to decrease the frequency and amplitude of muscle spasms in clients with spinal cord injuries or diseases and in clients with multiple sclerosis. Options 1, 3, and 4 are incorrect.

209.) A client with multiple sclerosis is receiving diazepam (Valium), a centrally acting skeletal muscle relaxant. Which of the following would indicate that the client is experiencing a side effect related to this medication? 1. Headache 2. Drowsiness 3. Urinary retention 4. Increased salivation

2. Drowsiness Rationale: Incoordination and drowsiness are common side effects resulting from this medication. Options 1, 3, and 4 are incorrect.

133.) A nurse is monitoring a client receiving desmopressin acetate (DDAVP) for adverse effects to the medication. Which of the following indicates the presence of an adverse effect? 1. Insomnia 2. Drowsiness 3. Weight loss 4. Increased urination

2. Drowsiness Rationale: Water intoxication (overhydration) or hyponatremia is an adverse effect to desmopressin. Early signs include drowsiness, listlessness, and headache. Decreased urination, rapid weight gain, confusion, seizures, and coma also may occur in overhydration. **Recall that this medication is used to treat diabetes insipidus to eliminate weight loss and increased urination.**

240.) A client with Parkinson's disease has been prescribed benztropine (Cogentin). The nurse monitors for which gastrointestinal (GI) side effect of this medication? 1. Diarrhea 2. Dry mouth 3. Increased appetite 4. Hyperactive bowel sounds

2. Dry mouth Rationale: Common GI side effects of benztropine therapy include constipation and dry mouth. Other GI side effects include nausea and ileus. These effects are the result of the anticholinergic properties of the medication. **Eliminate options 1 and 4 because they are comparable or alike. Recall that the medication is an anticholinergic, which causes dry mouth**

108.) A nurse is performing a follow-up teaching session with a client discharged 1 month ago who is taking fluoxetine (Prozac). What information would be important for the nurse to gather regarding the adverse effects related to the medication? 1. Cardiovascular symptoms 2. Gastrointestinal dysfunctions 3. Problems with mouth dryness 4. Problems with excessive sweating

2. Gastrointestinal dysfunctions Rationale: The most common adverse effects related to fluoxetine include central nervous system (CNS) and gastrointestinal (GI) system dysfunction. This medication affects the GI system by causing nausea and vomiting, cramping, and diarrhea. Options 1, 3, and 4 are not adverse effects of this medication.

136.) A nurse performs an admission assessment on a client who visits a health care clinic for the first time. The client tells the nurse that propylthiouracil (PTU) is taken daily. The nurse continues to collect data from the client, suspecting that the client has a history of: 1. Myxedema 2. Graves' disease 3. Addison's disease 4. Cushing's syndrome

2. Graves' disease Rationale: PTU inhibits thyroid hormone synthesis and is used to treat hyperthyroidism, or Graves' disease. Myxedema indicates hypothyroidism. Cushing's syndrome and Addison's disease are disorders related to adrenal function.

41.) The client has been taking omeprazole (Prilosec) for 4 weeks. The ambulatory care nurse evaluates that the client is receiving optimal intended effect of the medication if the client reports the absence of which symptom? 1. Diarrhea 2. Heartburn 3. Flatulence 4. Constipation

2. Heartburn Rationale: Omeprazole is a proton pump inhibitor classified as an antiulcer agent. The intended effect of the medication is relief of pain from gastric irritation, often called heartburn by clients. Omeprazole is not used to treat the conditions identified in options 1, 3, and 4.

131.) The nurse is reinforcing medication instructions to a client with breast cancer who is receiving cyclophosphamide (Neosar). The nurse tells the client to: 1. Take the medication with food. 2. Increase fluid intake to 2000 to 3000 mL daily. 3. Decrease sodium intake while taking the medication. 4. Increase potassium intake while taking the medication.

2. Increase fluid intake to 2000 to 3000 mL daily. Rationale: Hemorrhagic cystitis is a toxic effect that can occur with the use of cyclophosphamide. The client needs to be instructed to drink copious amounts of fluid during the administration of this medication. Clients also should monitor urine output for hematuria. The medication should be taken on an empty stomach, unless gastrointestinal (GI) upset occurs. Hyperkalemia can result from the use of the medication; therefore the client would not be told to increase potassium intake. The client would not be instructed to alter sodium intake.

101.) Ketoconazole is prescribed for a client with a diagnosis of candidiasis. Select the interventions that the nurse includes when administering this medication. Select all that apply. 1. Restrict fluid intake. 2. Instruct the client to avoid alcohol. 3. Monitor hepatic and liver function studies. 4. Administer the medication with an antacid. 5. Instruct the client to avoid exposure to the sun. 6. Administer the medication on an empty stomach.

2. Instruct the client to avoid alcohol. 3. Monitor hepatic and liver function studies. 5. Instruct the client to avoid exposure to the sun. Rationale: Ketoconazole is an antifungal medication. It is administered with food (not on an empty stomach) and antacids are avoided for 2 hours after taking the medication to ensure absorption. The medication is hepatotoxic and the nurse monitors liver function studies. The client is instructed to avoid exposure to the sun because the medication increases photosensitivity. The client is also instructed to avoid alcohol. There is no reason for the client to restrict fluid intake. In fact, this could be harmful to the client.

162.) Carbamazepine (Tegretol) is prescribed for a client with a diagnosis of psychomotor seizures. The nurse reviews the client's health history, knowing that this medication is contraindicated if which of the following disorders is present? 1. Headaches 2. Liver disease 3. Hypothyroidism 4. Diabetes mellitus

2. Liver disease Rationale: Carbamazepine (Tegretol) is contraindicated in liver disease, and liver function tests are routinely prescribed for baseline purposes and are monitored during therapy. It is also contraindicated if the client has a history of blood dyscrasias. It is not contraindicated in the conditions noted in the incorrect options.

89.) A nurse is reviewing the laboratory studies on a client receiving dantrolene sodium (Dantrium). Which laboratory test would identify an adverse effect associated with the administration of this medication? 1. Creatinine 2. Liver function tests 3. Blood urea nitrogen 4. Hematological function tests

2. Liver function tests Rationale: Dose-related liver damage is the most serious adverse effect of dantrolene. To reduce the risk of liver damage, liver function tests should be performed before treatment and periodically throughout the treatment course. It is administered in the lowest effective dosage for the shortest time necessary. **Eliminate options 1 and 3 because these tests both assess kidney function.**

65.) A nurse is reinforcing discharge instructions to a client receiving sulfisoxazole. Which of the following should be included in the list of instructions? 1. Restrict fluid intake. 2. Maintain a high fluid intake. 3. If the urine turns dark brown, call the health care provider (HCP) immediately. 4. Decrease the dosage when symptoms are improving to prevent an allergic response.

2. Maintain a high fluid intake. Rationale: Each dose of sulfisoxazole should be administered with a full glass of water, and the client should maintain a high fluid intake. The medication is more soluble in alkaline urine. The client should not be instructed to taper or discontinue the dose. Some forms of sulfisoxazole cause urine to turn dark brown or red. This does not indicate the need to notify the HCP.

161.) A nurse is caring for a client with severe back pain, and codeine sulfate has been prescribed for the client. Which of the following would the nurse include in the plan of care while the client is taking this medication? 1. Restrict fluid intake. 2. Monitor bowel activity. 3. Monitor for hypertension. 4. Monitor peripheral pulses.

2. Monitor bowel activity. Rationale: While the client is taking codeine sulfate, an opioid analgesic, the nurse would monitor vital signs and monitor for hypotension. The nurse should also increase fluid intake, palpate the bladder for urinary retention, auscultate bowel sounds, and monitor the pattern of daily bowel activity and stool consistency (codeine can cause constipation). The nurse should monitor respiratory status and initiate breathing and coughing exercises. In addition, the nurse monitors the effectiveness of the pain medication.

204.) A client receives a dose of edrophonium (Enlon). The client shows improvement in muscle strength for a period of time following the injection. The nurse interprets that this finding is compatible with: 1. Multiple sclerosis 2. Myasthenia gravis 3. Muscular dystrophy 4. Amyotrophic lateral sclerosis

2. Myasthenia gravis Rationale: Myasthenia gravis can often be diagnosed based on clinical signs and symptoms. The diagnosis can be confirmed by injecting the client with a dose of edrophonium . This medication inhibits the breakdown of an enzyme in the neuromuscular junction, so more acetylcholine binds to receptors. If the muscle is strengthened for 3 to 5 minutes after this injection, it confirms a diagnosis of myasthenia gravis. Another medication, neostigmine (Prostigmin), also may be used because its effect lasts for 1 to 2 hours, providing a better analysis. For either medication, atropine sulfate should be available as the antidote.

227.) When teaching a client who is being started on imipramine hydrochloride (Tofranil), the nurse would inform the client that the desired effects of the medication may: 1. Start during the first week of administration 2. Not occur for 2 to 3 weeks of administration 3. Start during the second week of administration 4. Not occur until after a month of administration

2. Not occur for 2 to 3 weeks of administration Rationale: The therapeutic effects of administration of imipramine hydrochloride may not occur for 2 to 3 weeks after the antidepressant therapy has been initiated. Therefore options 1, 3, and 4 are incorrect.

169.) Insulin glargine (Lantus) is prescribed for a client with diabetes mellitus. The nurse tells the client that it is best to take the insulin: 1. 1 hour after each meal 2. Once daily, at the same time each day 3. 15 minutes before breakfast, lunch, and dinner 4. Before each meal, on the basis of the blood glucose level

2. Once daily, at the same time each day Rationale: Insulin glargine is a long-acting recombinant DNA human insulin used to treat type 1 and type 2 diabetes mellitus. It has a 24-hour duration of action and is administered once a day, at the same time each day.

47.) A client has been taking isoniazid (INH) for 2 months. The client complains to a nurse about numbness, paresthesias, and tingling in the extremities. The nurse interprets that the client is experiencing: 1. Hypercalcemia 2. Peripheral neuritis 3. Small blood vessel spasm 4. Impaired peripheral circulation

2. Peripheral neuritis Rationale: A common side effect of the TB drug INH is peripheral neuritis. This is manifested by numbness, tingling, and paresthesias in the extremities. This side effect can be minimized by pyridoxine (vitamin B6) intake. Options 1, 3, and 4 are incorrect.

228.) A client receiving an anxiolytic medication complains that he feels very "faint" when he tries to get out of bed in the morning. The nurse recognizes this complaint as a symptom of: 1. Cardiac dysrhythmias 2. Postural hypotension 3. Psychosomatic symptoms 4. Respiratory insufficiency

2. Postural hypotension Rationale: Anxiolytic medications can cause postural hypotension. The client needs to be taught to rise to a sitting position and get out of bed slowly because of this adverse effect related to the medication. Options 1, 3, and 4 are unrelated to the use of this medication.

50.) A nurse has given a client taking ethambutol (Myambutol) information about the medication. The nurse determines that the client understands the instructions if the client states that he or she will immediately report: 1. Impaired sense of hearing 2. Problems with visual acuity 3. Gastrointestinal (GI) side effects 4. Orange-red discoloration of body secretions

2. Problems with visual acuity Rationale: Ethambutol causes optic neuritis, which decreases visual acuity and the ability to discriminate between the colors red and green. This poses a potential safety hazard when a client is driving a motor vehicle. The client is taught to report this symptom immediately. The client is also taught to take the medication with food if GI upset occurs. Impaired hearing results from antitubercular therapy with streptomycin. Orange-red discoloration of secretions occurs with rifampin (Rifadin).

25.) A home care nurse visits a client recently diagnosed with diabetes mellitus who is taking Humulin NPH insulin daily. The client asks the nurse how to store the unopened vials of insulin. The nurse tells the client to: 1. Freeze the insulin. 2. Refrigerate the insulin. 3. Store the insulin in a dark, dry place. 4. Keep the insulin at room temperature.

2. Refrigerate the insulin. Rationale: Insulin in unopened vials should be stored under refrigeration until needed. Vials should not be frozen. When stored unopened under refrigeration, insulin can be used up to the expiration date on the vial. Options 1, 3, and 4 are incorrect.

186.) A nurse prepares to administer sodium polystyrene sulfonate (Kayexalate) to a client. Before administering the medication, the nurse reviews the action of the medication and understands that it: 1. Releases bicarbonate in exchange for primarily sodium ions 2. Releases sodium ions in exchange for primarily potassium ions 3. Releases potassium ions in exchange for primarily sodium ions 4. Releases sodium ions in exchange for primarily bicarbonate ions

2. Releases sodium ions in exchange for primarily potassium ions Rationale: Sodium polystyrene sulfonate is a cation exchange resin used in the treatment of hyperkalemia. The resin either passes through the intestine or is retained in the colon. It releases sodium ions in exchange for primarily potassium ions. The therapeutic effect occurs 2 to 12 hours after oral administration and longer after rectal administration.

40.) The client who chronically uses nonsteroidal anti-inflammatory drugs has been taking misoprostol (Cytotec). The nurse determines that the medication is having the intended therapeutic effect if which of the following is noted? 1. Resolved diarrhea 2. Relief of epigastric pain 3. Decreased platelet count 4. Decreased white blood cell count

2. Relief of epigastric pain Rationale: The client who chronically uses nonsteroidal anti-inflammatory drugs (NSAIDs) is prone to gastric mucosal injury. Misoprostol is a gastric protectant and is given specifically to prevent this occurrence. Diarrhea can be a side effect of the medication, but is not an intended effect. Options 3 and 4 are incorrect.

168.) Colcrys (colchicine) is prescribed for a client with a diagnosis of gout. The nurse reviews the client's medical history in the health record, knowing that the medication would be contraindicated in which disorder? 1. Myxedema 2. Renal failure 3. Hypothyroidism 4. Diabetes mellitus

2. Renal failure Rationale: Colchicine is contraindicated in clients with severe gastrointestinal, renal, hepatic or cardiac disorders, or with blood dyscrasias. Clients with impaired renal function may exhibit myopathy and neuropathy manifested as generalized weakness. This medication should be used with caution in clients with impaired hepatic function, older clients, and debilitated clients. **Note that options 1, 3, and 4 are all endocrine-related disorders: Myxedema=Hypothyroidism**

48.) A client is to begin a 6-month course of therapy with isoniazid (INH). A nurse plans to teach the client to: 1. Drink alcohol in small amounts only. 2. Report yellow eyes or skin immediately. 3. Increase intake of Swiss or aged cheeses. 4. Avoid vitamin supplements during therapy.

2. Report yellow eyes or skin immediately. Rationale: INH is hepatotoxic, and therefore the client is taught to report signs and symptoms of hepatitis immediately (which include yellow skin and sclera). For the same reason, alcohol should be avoided during therapy. The client should avoid intake of Swiss cheese, fish such as tuna, and foods containing tyramine because they may cause a reaction characterized by redness and itching of the skin, flushing, sweating, tachycardia, headache, or lightheadedness. The client can avoid developing peripheral neuritis by increasing the intake of pyridoxine (vitamin B6) during the course of INH therapy for TB.

23.) A client who has been newly diagnosed with diabetes mellitus has been stabilized with daily insulin injections. Which information should the nurse teach when carrying out plans for discharge? 1. Keep insulin vials refrigerated at all times. 2. Rotate the insulin injection sites systematically. 3. Increase the amount of insulin before unusual exercise. 4. Monitor the urine acetone level to determine the insulin dosage.

2. Rotate the insulin injection sites systematically. Rationale: Insulin dosages should not be adjusted or increased before unusual exercise. If acetone is found in the urine, it may possibly indicate the need for additional insulin. To minimize the discomfort associated with insulin injections, the insulin should be administered at room temperature. Injection sites should be systematically rotated from one area to another. The client should be instructed to give injections in one area, about 1 inch apart, until the whole area has been used and then to change to another site. This prevents dramatic changes in daily insulin absorption.

216.) A nurse is caring for a client with gout who is taking Colcrys (colchicine). The client has been instructed to restrict the diet to low-purine foods. Which of the following foods should the nurse instruct the client to avoid while taking this medication? 1. Spinach 2. Scallops 3. Potatoes 4. Ice cream

2. Scallops Rationale: Colchicine is a medication used for clients with gout to inhibit the reabsorption of uric acid by the kidney and promote excretion of uric acid in the urine. Uric acid is produced when purine is catabolized. Clients are instructed to modify their diet and limit excessive purine intake. High-purine foods to avoid or limit include organ meats, roe, sardines, scallops, anchovies, broth, mincemeat, herring, shrimp, mackerel, gravy, and yeast.

114.) A postoperative client requests medication for flatulence (gas pains). Which medication from the following PRN list should the nurse administer to this client? 1. Ondansetron (Zofran) 2. Simethicone (Mylicon) 3. Acetaminophen (Tylenol) 4. Magnesium hydroxide (milk of magnesia, MOM)

2. Simethicone (Mylicon) Rationale: Simethicone is an antiflatulent used in the relief of pain caused by excessive gas in the gastrointestinal tract. Ondansetron is used to treat postoperative nausea and vomiting. Acetaminophen is a nonopioid analgesic. Magnesium hydroxide is an antacid and laxative.

172.) A nurse provides dietary instructions to a client who will be taking warfarin sodium (Coumadin). The nurse tells the client to avoid which food item? 1. Grapes 2. Spinach 3. Watermelon 4. Cottage cheese

2. Spinach Rationale: Warfarin sodium is an anticoagulant. Anticoagulant medications act by antagonizing the action of vitamin K, which is needed for clotting. When a client is taking an anticoagulant, foods high in vitamin K often are omitted from the diet. Vitamin K-rich foods include green, leafy vegetables, fish, liver, coffee, and tea.

82.) A client is receiving meperidine hydrochloride (Demerol) for pain. Which of the following are side effects of this medication. Select all that apply. 1. Diarrhea 2. Tremors 3. Drowsiness 4. Hypotension 5. Urinary frequency 6. Increased respiratory rate

2. Tremors 3. Drowsiness 4. Hypotension Rationale: Meperidine hydrochloride is an opioid analgesic. Side effects include respiratory depression, drowsiness, hypotension, constipation, urinary retention, nausea, vomiting, and tremors.

188.) The nurse should anticipate that the most likely medication to be prescribed prophylactically for a child with spina bifida (myelomeningocele) who has a neurogenic bladder would be: 1. Prednisone 2. Sulfisoxazole 3. Furosemide (Lasix) 4. Intravenous immune globulin (IVIG)

2. Sulfisoxazole Rationale: A neurogenic bladder prevents the bladder from completely emptying because of the decrease in muscle tone. The most likely medication to be prescribed to prevent urinary tract infection would be an antibiotic. A common prescribed medication is sulfisoxazole. Prednisone relieves allergic reactions and inflammation rather than preventing infection. Furosemide promotes diuresis and decreases edema caused by congestive heart failure. IVIG assists with antibody production in immunocompromised clients.

147.) A client with a prescription to take theophylline (Theo-24) daily has been given medication instructions by the nurse. The nurse determines that the client needs further information about the medication if the client states that he or she will: 1. Drink at least 2 L of fluid per day. 2. Take the daily dose at bedtime. 3. Avoid changing brands of the medication without health care provider (HCP) approval. 4. Avoid over-the-counter (OTC) cough and cold medications unless approved by the HCP.

2. Take the daily dose at bedtime. Rationale: The client taking a single daily dose of theophylline, a xanthine bronchodilator, should take the medication early in the morning. This enables the client to have maximal benefit from the medication during daytime activities. In addition, this medication causes insomnia. The client should take in at least 2 L of fluid per day to decrease viscosity of secretions. The client should check with the physician before changing brands of the medication. The client also checks with the HCP before taking OTC cough, cold, or other respiratory preparations because they could cause interactive effects, increasing the side effects of theophylline and causing dysrhythmias.

45.) A client has a prescription to take guaifenesin (Humibid) every 4 hours, as needed. The nurse determines that the client understands the most effective use of this medication if the client states that he or she will: 1. Watch for irritability as a side effect. 2. Take the tablet with a full glass of water. 3. Take an extra dose if the cough is accompanied by fever. 4. Crush the sustained-release tablet if immediate relief is needed.

2. Take the tablet with a full glass of water. Rationale: Guaifenesin is an expectorant. It should be taken with a full glass of water to decrease viscosity of secretions. Sustained-release preparations should not be broken open, crushed, or chewed. The medication may occasionally cause dizziness, headache, or drowsiness as side effects. The client should contact the health care provider if the cough lasts longer than 1 week or is accompanied by fever, rash, sore throat, or persistent headache.

214.) A health care provider initiates carbidopa/levodopa (Sinemet) therapy for the client with Parkinson's disease. A few days after the client starts the medication, the client complains of nausea and vomiting. The nurse tells the client that: 1. Taking an antiemetic is the best measure to prevent the nausea. 2. Taking the medication with food will help to prevent the nausea. 3. This is an expected side effect of the medication and will decrease over time. 4. The nausea and vomiting will decrease when the dose of levodopa is stabilized.

2. Taking the medication with food will help to prevent the nausea. Rationale: If carbidopa/levodopa is causing nausea and vomiting, the nurse would tell the client that taking the medication with food will prevent the nausea. Additionally, the client should be instructed not to take the medication with a high-protein meal because the high-protein will affect absorption. Antiemetics from the phenothiazine class should not be used because they block the therapeutic action of dopamine. **eliminate options 3 and 4 because they are comparable or alike**

57.) A nurse is monitoring a client who is taking propranolol (Inderal LA). Which data collection finding would indicate a potential serious complication associated with propranolol? 1. The development of complaints of insomnia 2. The development of audible expiratory wheezes 3. A baseline blood pressure of 150/80 mm Hg followed by a blood pressure of 138/72 mm Hg after two doses of the medication 4. A baseline resting heart rate of 88 beats/min followed by a resting heart rate of 72 beats/min after two doses of the medication

2. The development of audible expiratory wheezes Rationale: Audible expiratory wheezes may indicate a serious adverse reaction, bronchospasm. β-Blockers may induce this reaction, particularly in clients with chronic obstructive pulmonary disease or asthma. Normal decreases in blood pressure and heart rate are expected. Insomnia is a frequent mild side effect and should be monitored.

28.) The health care provider (HCP) prescribes exenatide (Byetta) for a client with type 1 diabetes mellitus who takes insulin. The nurse knows that which of the following is the appropriate intervention? 1. The medication is administered within 60 minutes before the morning and evening meal. 2. The medication is withheld and the HCP is called to question the prescription for the client. 3. The client is monitored for gastrointestinal side effects after administration of the medication. 4. The insulin is withdrawn from the Penlet into an insulin syringe to prepare for administration.

2. The medication is withheld and the HCP is called to question the prescription for the client. Rationale: Exenatide (Byetta) is an incretin mimetic used for type 2 diabetes mellitus only. It is not recommended for clients taking insulin. Hence, the nurse should hold the medication and question the HCP regarding this prescription. Although options 1 and 3 are correct statements about the medication, in this situation the medication should not be administered. The medication is packaged in prefilled pens ready for injection without the need for drawing it up into another syringe.

165.) The client has been on treatment for rheumatoid arthritis for 3 weeks. During the administration of etanercept (Enbrel), it is most important for the nurse to assess: 1. The injection site for itching and edema 2. The white blood cell counts and platelet counts 3. Whether the client is experiencing fatigue and joint pain 4. A metallic taste in the mouth and a loss of appetite

2. The white blood cell counts and platelet counts Rationale: Infection and pancytopenia are adverse effects of etanercept (Enbrel). Laboratory studies are performed before and during treatment. The appearance of abnormal white blood cell counts and abnormal platelet counts can alert the nurse to a potential life-threatening infection. Injection site itching is a common occurrence following administration of the medication. In early treatment, residual fatigue and joint pain may still be apparent. A metallic taste and loss of appetite are not common signs of side effects of this medication.

83.) The client has been on treatment for rheumatoid arthritis for 3 weeks. During the administration of etanercept (Enbrel), it is most important for the nurse to check: 1. The injection site for itching and edema 2. The white blood cell counts and platelet counts 3. Whether the client is experiencing fatigue and joint pain 4. A metallic taste in the mouth, with a loss of appetite

2. The white blood cell counts and platelet counts Rationale: Infection and pancytopenia are side effects of etanercept (Enbrel). Laboratory studies are performed before and during drug treatment. The appearance of abnormal white blood cell counts and abnormal platelet counts can alert the nurse to a potentially life-threatening infection. Injection site itching is a common occurrence following administration. A metallic taste with loss of appetite are not common signs of side effects of this medication.

126.) A nurse is caring for a client who has been prescribed furosemide (Lasix) and is monitoring for adverse effects associated with this medication. Which of the following should the nurse recognize as a potential adverse effect Select all that apply. 1. Nausea 2. Tinnitus 3. Hypotension 4. Hypokalemia 5. Photosensitivity 6. Increased urinary frequency

2. Tinnitus 3. Hypotension 4. Hypokalemia Rationale: Furosemide is a loop diuretic; therefore, an expected effect is increased urinary frequency. Nausea is a frequent side effect, not an adverse effect. Photosensitivity is an occasional side effect. Adverse effects include tinnitus (ototoxicity), hypotension, and hypokalemia and occur as a result of sudden volume depletion.

51.) Cycloserine (Seromycin) is added to the medication regimen for a client with tuberculosis. Which of the following would the nurse include in the client-teaching plan regarding this medication? 1. To take the medication before meals 2. To return to the clinic weekly for serum drug-level testing 3. It is not necessary to call the health care provider (HCP) if a skin rash occurs. 4. It is not necessary to restrict alcohol intake with this medication.

2. To return to the clinic weekly for serum drug-level testing Rationale: Cycloserine (Seromycin) is an antitubercular medication that requires weekly serum drug level determinations to monitor for the potential of neurotoxicity. Serum drug levels lower than 30 mcg/mL reduce the incidence of neurotoxicity. The medication must be taken after meals to prevent gastrointestinal irritation. The client must be instructed to notify the HCP if a skin rash or signs of central nervous system toxicity are noted. Alcohol must be avoided because it increases the risk of seizure activity.

135.) A nurse reinforces medication instructions to a client who is taking levothyroxine (Synthroid). The nurse instructs the client to notify the health care provider (HCP) if which of the following occurs? 1. Fatigue 2. Tremors 3. Cold intolerance 4. Excessively dry skin

2. Tremors Rationale: Excessive doses of levothyroxine (Synthroid) can produce signs and symptoms of hyperthyroidism. These include tachycardia, chest pain, tremors, nervousness, insomnia, hyperthermia, heat intolerance, and sweating. The client should be instructed to notify the HCP if these occur. Options 1, 3, and 4 are signs of hypothyroidism.

7.) Isotretinoin is prescribed for a client with severe acne. Before the administration of this medication, the nurse anticipates that which laboratory test will be prescribed? 1. Platelet count 2. Triglyceride level 3. Complete blood count 4. White blood cell count

2. Triglyceride level Rationale: Isotretinoin can elevate triglyceride levels. Blood triglyceride levels should be measured before treatment and periodically thereafter until the effect on the triglycerides has been evaluated. Options 1, 3, and 4 do not need to be monitored specifically during this treatment.

14.) The client with acute myelocytic leukemia is being treated with busulfan (Myleran). Which laboratory value would the nurse specifically monitor during treatment with this medication? 1. Clotting time 2. Uric acid level 3. Potassium level 4. Blood glucose level

2. Uric acid level Rationale: Busulfan (Myleran) can cause an increase in the uric acid level. Hyperuricemia can produce uric acid nephropathy, renal stones, and acute renal failure. Options 1, 3, and 4 are not specifically related to this medication.

68.) Bethanechol chloride (Urecholine) is prescribed for a client with urinary retention. Which disorder would be a contraindication to the administration of this medication? 1. Gastric atony 2. Urinary strictures 3. Neurogenic atony 4. Gastroesophageal reflux

2. Urinary strictures Rationale: Bethanechol chloride (Urecholine) can be harmful to clients with urinary tract obstruction or weakness of the bladder wall. The medication has the ability to contract the bladder and thereby increase pressure within the urinary tract. Elevation of pressure within the urinary tract could rupture the bladder in clients with these conditions.

238.) Ribavirin (Virazole) is prescribed for the hospitalized child with respiratory syncytial virus (RSV). The nurse prepares to administer this medication via which of the following routes? 1. Orally 2. Via face mask 3. Intravenously 4. Intramuscularly

2. Via face mask Rationale: Ribavirin is an antiviral respiratory medication used mainly in hospitalized children with severe RSV and in high-risk children. Administration is via hood, face mask, or oxygen tent. The medication is most effective if administered within the first 3 days of the infection.

55.) A client who is receiving digoxin (Lanoxin) daily has a serum potassium level of 3.0 mEq/L and is complaining of anorexia. A health care provider prescribes a digoxin level to rule out digoxin toxicity. A nurse checks the results, knowing that which of the following is the therapeutic serum level (range) for digoxin? 1. 3 to 5 ng/mL 2. 0.5 to 2 ng/mL 3. 1.2 to 2.8 ng/mL 4. 3.5 to 5.5 ng/mL

2.) 0.5 to 2 ng/mL Rationale: Therapeutic levels for digoxin range from 0.5 to 2 ng/mL. Therefore, options 1, 3, and 4 are incorrect.

RN is teaching a pt about a new drug... what's most important to teach the pt to improve the intensity of the response to the drug

Take the prescribed dose

Insulin can be kept at room temp

28 days

128.) A nurse is providing instructions to an adolescent who has a history of seizures and is taking an anticonvulsant medication. Which of the following statements indicates that the client understands the instructions? 1. "I will never be able to drive a car." 2. "My anticonvulsant medication will clear up my skin." 3. "I can't drink alcohol while I am taking my medication." 4. "If I forget my morning medication, I can take two pills at bedtime."

3. "I can't drink alcohol while I am taking my medication." Rationale: Alcohol will lower the seizure threshold and should be avoided. Adolescents can obtain a driver's license in most states when they have been seizure free for 1 year. Anticonvulsants cause acne and oily skin; therefore a dermatologist may need to be consulted. If an anticonvulsant medication is missed, the health care provider should be notified.

106.) Fluoxetine (Prozac) is prescribed for the client. The nurse reinforces instructions to the client regarding the administration of the medication. Which statement by the client indicates an understanding about administration of the medication? 1. "I should take the medication with my evening meal." 2. "I should take the medication at noon with an antacid." 3. "I should take the medication in the morning when I first arise." 4. "I should take the medication right before bedtime with a snack."

3. "I should take the medication in the morning when I first arise." Rationale: Fluoxetine hydrochloride is administered in the early morning without consideration to meals. **Eliminate options 1, 2, and 4 because they are comparable or alike and indicate taking the medication with an antacid or food.**

197.) Collagenase (Santyl) is prescribed for a client with a severe burn to the hand. The nurse provides instructions to the client regarding the use of the medication. Which statement by the client indicates an accurate understanding of the use of this medication? 1. "I will apply the ointment once a day and leave it open to the air." 2. "I will apply the ointment twice a day and leave it open to the air." 3. "I will apply the ointment once a day and cover it with a sterile dressing." 4. "I will apply the ointment at bedtime and in the morning and cover it with a sterile dressing."

3. "I will apply the ointment once a day and cover it with a sterile dressing." Rationale: Collagenase is used to promote debridement of dermal lesions and severe burns. It is usually applied once daily and covered with a sterile dressing.

164.) A client receives a prescription for methocarbamol (Robaxin), and the nurse reinforces instructions to the client regarding the medication. Which client statement would indicate a need for further instructions? 1. "My urine may turn brown or green." 2. "This medication is prescribed to help relieve my muscle spasms." 3. "If my vision becomes blurred, I don't need to be concerned about it." 4. "I need to call my doctor if I experience nasal congestion from this medication."

3. "If my vision becomes blurred, I don't need to be concerned about it." Rationale: The client needs to be told that the urine may turn brown, black, or green. Other adverse effects include blurred vision, nasal congestion, urticaria, and rash. The client needs to be instructed that, if these adverse effects occur, the health care provider needs to be notified. The medication is used to relieve muscle spasms.

11.) The health care provider has prescribed silver sulfadiazine (Silvadene) for the client with a partial-thickness burn, which has cultured positive for gram-negative bacteria. The nurse is reinforcing information to the client about the medication. Which statement made by the client indicates a lack of understanding about the treatments? 1. "The medication is an antibacterial." 2. "The medication will help heal the burn." 3. "The medication will permanently stain my skin." 4. "The medication should be applied directly to the wound."

3. "The medication will permanently stain my skin." Rationale: Silver sulfadiazine (Silvadene) is an antibacterial that has a broad spectrum of activity against gram-negative bacteria, gram-positive bacteria, and yeast. It is applied directly to the wound to assist in healing. It does not stain the skin.

42.) A client with a peptic ulcer is diagnosed with a Helicobacter pylori infection. The nurse is reinforcing teaching for the client about the medications prescribed, including clarithromycin (Biaxin), esomeprazole (Nexium), and amoxicillin (Amoxil). Which statement by the client indicates the best understanding of the medication regimen? 1. "My ulcer will heal because these medications will kill the bacteria." 2. "These medications are only taken when I have pain from my ulcer." 3. "The medications will kill the bacteria and stop the acid production." 4. "These medications will coat the ulcer and decrease the acid production in my stomach."

3. "The medications will kill the bacteria and stop the acid production." Rationale: Triple therapy for Helicobacter pylori infection usually includes two antibacterial drugs and a proton pump inhibitor. Clarithromycin and amoxicillin are antibacterials. Esomeprazole is a proton pump inhibitor. These medications will kill the bacteria and decrease acid production.

141.) The nurse has reinforced instructions to a client who has been prescribed cholestyramine (Questran). Which statement by the client indicates a need for further instructions? 1. "I will continue taking vitamin supplements." 2. "This medication will help lower my cholesterol." 3. "This medication should only be taken with water." 4. "A high-fiber diet is important while taking this medication."

3. "This medication should only be taken with water." Rationale: Cholestyramine (Questran) is a bile acid sequestrant used to lower the cholesterol level, and client compliance is a problem because of its taste and palatability. The use of flavored products or fruit juices can improve the taste. Some side effects of bile acid sequestrants include constipation and decreased vitamin absorption. **Note the closed-ended word "only" in option 3**

219.) A health care provider instructs a client with rheumatoid arthritis to take ibuprofen (Motrin). The nurse reinforces the instructions, knowing that the normal adult dose for this client is which of the following? 1. 100 mg orally twice a day 2. 200 mg orally twice a day 3. 400 mg orally three times a day 4. 1000 mg orally four times a day

3. 400 mg orally three times a day Rationale: For acute or chronic rheumatoid arthritis or osteoarthritis, the normal oral adult dose is 400 to 800 mg three or four times daily.

191.) A child is brought to the emergency department for treatment of an acute asthma attack. The nurse prepares to administer which of the following medications first? 1. Oral corticosteroids 2. A leukotriene modifier 3. A β2 agonist 4. A nonsteroidal anti-inflammatory

3. A β2 agonist Rationale: In treating an acute asthma attack, a short acting β2 agonist such as albuterol (Proventil HFA) will be given to produce bronchodilation. Options 1, 2, and 4 are long-term control (preventive) medications.

120.) A client is taking lansoprazole (Prevacid) for the chronic management of Zollinger-Ellison syndrome. The nurse advises the client to take which of the following products if needed for a headache? 1. Naprosyn (Aleve) 2. Ibuprofen (Advil) 3. Acetaminophen (Tylenol) 4. Acetylsalicylic acid (aspirin)

3. Acetaminophen (Tylenol) Rationale: Zollinger-Ellison syndrome is a hypersecretory condition of the stomach. The client should avoid taking medications that are irritating to the stomach lining. Irritants would include aspirin and nonsteroidal antiinflammatory drugs (ibuprofen). The client should be advised to take acetaminophen for headache. **Remember that options that are comparable or alike are not likely to be correct. With this in mind, eliminate options 1 and 2 first.**

233.) Diphenhydramine hydrochloride (Benadryl) is used in the treatment of allergic rhinitis for a hospitalized client with a chronic psychotic disorder. The client asks the nurse why the medication is being discontinued before hospital discharge. The nurse responds, knowing that: 1. Allergic symptoms are short in duration. 2. This medication promotes long-term extrapyramidal symptoms. 3. Addictive properties are enhanced in the presence of psychotropic medications. 4. Poor compliance causes this medication to fail to reach its therapeutic blood level.

3. Addictive properties are enhanced in the presence of psychotropic medications. Rationale: The addictive properties of diphenhydramine hydrochloride are enhanced when used with psychotropic medications. Allergic symptoms may not be short term and will occur if allergens are present in the environment. Poor compliance may be a problem with psychotic clients but is not the subject of the question. Diphenhydramine hydrochloride may be used for extrapyramidal symptoms and mild medication-induced movement disorders.

35.) The client has a PRN prescription for loperamide hydrochloride (Imodium). The nurse understands that this medication is used for which condition? 1. Constipation 2. Abdominal pain 3. An episode of diarrhea 4. Hematest-positive nasogastric tube drainage

3. An episode of diarrhea Rationale: Loperamide is an antidiarrheal agent. It is used to manage acute and also chronic diarrhea in conditions such as inflammatory bowel disease. Loperamide also can be used to reduce the volume of drainage from an ileostomy. It is not used for the conditions in options 1, 2, and 4.

207.) A client is suspected of having myasthenia gravis, and the health care provider administers edrophonium (Enlon) to determine the diagnosis. After administration of this medication, which of the following would indicate the presence of myasthenia gravis? 1. Joint pain 2. A decrease in muscle strength 3. An increase in muscle strength 4. Feelings of faintness, dizziness, hypotension, and signs of flushing in the client

3. An increase in muscle strength Rationale: Edrophonium is a short-acting acetylcholinesterase inhibitor used as a diagnostic agent. When a client with suspected myasthenia gravis is given the medication intravenously, an increase in muscle strength would be seen in 1 to 3 minutes. If no response occurs, another dose is given over the next 2 minutes, and muscle strength is again tested. If no increase in muscle strength occurs with this higher dose, the muscle weakness is not caused by myasthenia gravis. Clients receiving injections of this medication commonly demonstrate a drop of blood pressure, feel faint and dizzy, and are flushed.

139.) Prednisone is prescribed for a client with diabetes mellitus who is taking Humulin neutral protamine Hagedorn (NPH) insulin daily. Which of the following prescription changes does the nurse anticipate during therapy with the prednisone? 1. An additional dose of prednisone daily 2. A decreased amount of daily Humulin NPH insulin 3. An increased amount of daily Humulin NPH insulin 4. The addition of an oral hypoglycemic medication daily

3. An increased amount of daily Humulin NPH insulin Rationale: Glucocorticoids can elevate blood glucose levels. Clients with diabetes mellitus may need their dosages of insulin or oral hypoglycemic medications increased during glucocorticoid therapy. Therefore the other options are incorrect.

107.) A client receiving a tricyclic antidepressant arrives at the mental health clinic. Which observation indicates that the client is correctly following the medication plan? 1. Reports not going to work for this past week 2. Complains of not being able to "do anything" anymore 3. Arrives at the clinic neat and appropriate in appearance 4. Reports sleeping 12 hours per night and 3 to 4 hours during the day

3. Arrives at the clinic neat and appropriate in appearance Rationale: Depressed individuals will sleep for long periods, are not able to go to work, and feel as if they cannot "do anything." Once they have had some therapeutic effect from their medication, they will report resolution of many of these complaints as well as demonstrate an improvement in their appearance.

174.) A client with portosystemic encephalopathy is receiving oral lactulose (Chronulac) daily. The nurse assesses which of the following to determine medication effectiveness? 1. Lung sounds 2. Blood pressure 3. Blood ammonia level 4. Serum potassium level

3. Blood ammonia level Rationale: Lactulose is a hyperosmotic laxative and ammonia detoxicant. It is used to prevent or treat portosystemic encephalopathy, including hepatic precoma and coma. It also is used to treat constipation. The medication retains ammonia in the colon (decreases the blood ammonia concentration), producing an osmotic effect. It promotes increased peristalsis and bowel evacuation, expelling ammonia from the colon.

96.) The nurse is caring for a postrenal transplant client taking cyclosporine (Sandimmune, Gengraf, Neoral). The nurse notes an increase in one of the client's vital signs, and the client is complaining of a headache. What is the vital sign that is most likely increased? 1. Pulse 2. Respirations 3. Blood pressure 4. Pulse oximetry

3. Blood pressure Rationale: Hypertension can occur in a client taking cyclosporine (Sandimmune, Gengraf, Neoral), and because this client is also complaining of a headache, the blood pressure is the vital sign to be monitoring most closely. Other adverse effects include infection, nephrotoxicity, and hirsutism. Options 1, 2, and 4 are unrelated to the use of this medication.

153.) A client is diagnosed with pulmonary embolism and is to be treated with streptokinase (Streptase). A nurse would report which priority data collection finding to the registered nurse before initiating this therapy? 1. Adventitious breath sounds 2. Temperature of 99.4° F orally 3. Blood pressure of 198/110 mm Hg 4. Respiratory rate of 28 breaths/min

3. Blood pressure of 198/110 mm Hg Rationale: Thrombolytic therapy is contraindicated in a number of preexisting conditions in which there is a risk of uncontrolled bleeding, similar to the case in anticoagulant therapy. Thrombolytic therapy also is contraindicated in severe uncontrolled hypertension because of the risk of cerebral hemorrhage. Therefore the nurse would report the results of the blood pressure to the registered nurse before initiating therapy. The findings in options 1, 2, and 4 may be present in the client with pulmonary embolism.

69.) A nurse who is administering bethanechol chloride (Urecholine) is monitoring for acute toxicity associated with the medication. The nurse checks the client for which sign of toxicity? 1. Dry skin 2. Dry mouth 3. Bradycardia 4. Signs of dehydration

3. Bradycardia Rationale: Toxicity (overdose) produces manifestations of excessive muscarinic stimulation such as salivation, sweating, involuntary urination and defecation, bradycardia, and severe hypotension. Treatment includes supportive measures and the administration of atropine sulfate subcutaneously or intravenously.

1) A nurse is caring for a client with hyperparathyroidism and notes that the client's serum calcium level is 13 mg/dL. Which medication should the nurse prepare to administer as prescribed to the client? 1. Calcium chloride 2. Calcium gluconate 3. Calcitonin (Miacalcin) 4. Large doses of vitamin D

3. Calcitonin (Miacalcin) Rationale: The normal serum calcium level is 8.6 to 10.0 mg/dL. This client is experiencing hypercalcemia. Calcium gluconate and calcium chloride are medications used for the treatment of tetany, which occurs as a result of acute hypocalcemia. In hypercalcemia, large doses of vitamin D need to be avoided. Calcitonin, a thyroid hormone, decreases the plasma calcium level by inhibiting bone resorption and lowering the serum calcium concentration.

49.) A client has been started on long-term therapy with rifampin (Rifadin). A nurse teaches the client that the medication: 1. Should always be taken with food or antacids 2. Should be double-dosed if one dose is forgotten 3. Causes orange discoloration of sweat, tears, urine, and feces 4. May be discontinued independently if symptoms are gone in 3 months

3. Causes orange discoloration of sweat, tears, urine, and feces Rationale: Rifampin should be taken exactly as directed as part of TB therapy. Doses should not be doubled or skipped. The client should not stop therapy until directed to do so by a health care provider. The medication should be administered on an empty stomach unless it causes gastrointestinal upset, and then it may be taken with food. Antacids, if prescribed, should be taken at least 1 hour before the medication. Rifampin causes orange-red discoloration of body secretions and will permanently stain soft contact lenses.

38.) An older client recently has been taking cimetidine (Tagamet). The nurse monitors the client for which most frequent central nervous system side effect of this medication? 1. Tremors 2. Dizziness 3. Confusion 4. Hallucinations

3. Confusion Rationale: Cimetidine is a histamine 2 (H2)-receptor antagonist. Older clients are especially susceptible to central nervous system side effects of cimetidine. The most frequent of these is confusion. Less common central nervous system side effects include headache, dizziness, drowsiness, and hallucinations.

16.) The clinic nurse is reviewing a teaching plan for the client receiving an antineoplastic medication. When implementing the plan, the nurse tells the client: 1. To take aspirin (acetylsalicylic acid) as needed for headache 2. Drink beverages containing alcohol in moderate amounts each evening 3. Consult with health care providers (HCPs) before receiving immunizations 4. That it is not necessary to consult HCPs before receiving a flu vaccine at the local health fair

3. Consult with health care providers (HCPs) before receiving immunizations Rationale: Because antineoplastic medications lower the resistance of the body, clients must be informed not to receive immunizations without a HCP's approval. Clients also need to avoid contact with individuals who have recently received a live virus vaccine. Clients need to avoid aspirin and aspirin-containing products to minimize the risk of bleeding, and they need to avoid alcohol to minimize the risk of toxicity and side effects.

5.) Mafenide acetate (Sulfamylon) is prescribed for the client with a burn injury. When applying the medication, the client complains of local discomfort and burning. Which of the following is the most appropriate nursing action? 1. Notifying the registered nurse 2. Discontinuing the medication 3. Informing the client that this is normal 4. Applying a thinner film than prescribed to the burn site

3. Informing the client that this is normal Rationale: Mafenide acetate is bacteriostatic for gram-negative and gram-positive organisms and is used to treat burns to reduce bacteria present in avascular tissues. The client should be informed that the medication will cause local discomfort and burning and that this is a normal reaction; therefore options 1, 2, and 4 are incorrect

150.) A client complaining of not feeling well is seen in a clinic. The client is taking several medications for the control of heart disease and hypertension. These medications include a β-blocker, digoxin (Lanoxin), and a diuretic. A tentative diagnosis of digoxin toxicity is made. Which of the following assessment data would support this diagnosis? 1. Dyspnea, edema, and palpitations 2. Chest pain, hypotension, and paresthesia 3. Double vision, loss of appetite, and nausea 4. Constipation, dry mouth, and sleep disorder

3. Double vision, loss of appetite, and nausea Rationale: Double vision, loss of appetite, and nausea are signs of digoxin toxicity. Additional signs of digoxin toxicity include bradycardia, difficulty reading, visual alterations such as green and yellow vision or seeing spots or halos, confusion, vomiting, diarrhea, decreased libido, and impotence. **gastrointestinal (GI) and visual disturbances occur with digoxin toxicity**

148.) A client is taking cetirizine hydrochloride (Zyrtec). The nurse checks for which of the following side effects of this medication? 1. Diarrhea 2. Excitability 3. Drowsiness 4. Excess salivation

3. Drowsiness Rationale: A frequent side effect of cetirizine hydrochloride (Zyrtec), an antihistamine, is drowsiness or sedation. Others include blurred vision, hypertension (and sometimes hypotension), dry mouth, constipation, urinary retention, and sweating.

85.) A nurse is monitoring a client receiving baclofen (Lioresal) for side effects related to the medication. Which of the following would indicate that the client is experiencing a side effect? 1. Polyuria 2. Diarrhea 3. Drowsiness 4. Muscular excitability

3. Drowsiness Rationale: Baclofen is a central nervous system (CNS) depressant and frequently causes drowsiness, dizziness, weakness, and fatigue. It can also cause nausea, constipation, and urinary retention. Clients should be warned about the possible reactions. Options 1, 2, and 4 are not side effects.

138.) A daily dose of prednisone is prescribed for a client. A nurse reinforces instructions to the client regarding administration of the medication and instructs the client that the best time to take this medication is: 1. At noon 2. At bedtime 3. Early morning 4. Anytime, at the same time, each day

3. Early morning Rationale: Corticosteroids (glucocorticoids) should be administered before 9:00 AM. Administration at this time helps minimize adrenal insufficiency and mimics the burst of glucocorticoids released naturally by the adrenal glands each morning. **Note the suffix "-sone," and recall that medication names that end with these letters are corticosteroids.**

71.) After kidney transplantation, cyclosporine (Sand immune) is prescribed for a client. Which laboratory result would indicate an adverse effect from the use of this medication? 1. Decreased creatinine level 2. Decreased hemoglobin level 3. Elevated blood urea nitrogen level 4. Decreased white blood cell count

3. Elevated blood urea nitrogen level Rationale: Nephrotoxicity can occur from the use of cyclosporine (Sandimmune). Nephrotoxicity is evaluated by monitoring for elevated blood urea nitrogen (BUN) and serum creatinine levels. Cyclosporine is an immunosuppressant but does not depress the bone marrow.

244.) A client has a prescription for valproic acid (Depakene) orally once daily. The nurse plans to: 1. Administer the medication with an antacid. 2. Administer the medication with a carbonated beverage. 3. Ensure that the medication is administered at the same time each day. 4. Ensure that the medication is administered 2 hours before breakfast only, when the client's stomach is empty.

3. Ensure that the medication is administered at the same time each day. Rationale: Valproic acid is an anticonvulsant, antimanic, and antimigraine medication. It may be administered with or without food. It should not be taken with an antacid or carbonated beverage because these products will affect medication absorption. The medication is administered at the same time each day to maintain therapeutic serum levels. **Use general pharmacology guidelines to assist in eliminating options 1 and 2. Eliminate option 4 because of the closed-ended word "only."**

154.) A nurse is reinforcing dietary instructions to a client who has been prescribed cyclosporine (Sandimmune). Which food item would the nurse instruct the client to avoid? 1. Red meats 2. Orange juice 3. Grapefruit juice 4. Green, leafy vegetables

3. Grapefruit juice Rationale: A compound present in grapefruit juice inhibits metabolism of cyclosporine. As a result, the consumption of grapefruit juice can raise cyclosporine levels by 50% to 100%, thereby greatly increasing the risk of toxicity. Grapefruit juice needs to be avoided. Red meats, orange juice, and green leafy vegetables are acceptable to consume.

97.) Amikacin (Amikin) is prescribed for a client with a bacterial infection. The client is instructed to contact the health care provider (HCP) immediately if which of the following occurs? 1. Nausea 2. Lethargy 3. Hearing loss 4. Muscle aches

3. Hearing loss Rationale: Amikacin (Amikin) is an aminoglycoside. Adverse effects of aminoglycosides include ototoxicity (hearing problems), confusion, disorientation, gastrointestinal irritation, palpitations, blood pressure changes, nephrotoxicity, and hypersensitivity. The nurse instructs the client to report hearing loss to the HCP immediately. Lethargy and muscle aches are not associated with the use of this medication. It is not necessary to contact the HCP immediately if nausea occurs. If nausea persists or results in vomiting, the HCP should be notified. **(most aminoglycoside medication names end in the letters -cin)**

60.) A nurse is planning to administer hydrochlorothiazide (HydroDIURIL) to a client. The nurse understands that which of the following are concerns related to the administration of this medication? 1. Hypouricemia, hyperkalemia 2. Increased risk of osteoporosis 3. Hypokalemia, hyperglycemia, sulfa allergy 4. Hyperkalemia, hypoglycemia, penicillin allergy

3. Hypokalemia, hyperglycemia, sulfa allergy Rationale: Thiazide diuretics such as hydrochlorothiazide are sulfa-based medications, and a client with a sulfa allergy is at risk for an allergic reaction. Also, clients are at risk for hypokalemia, hyperglycemia, hypercalcemia, hyperlipidemia, and hyperuricemia.

130.) The nurse is analyzing the laboratory results of a client with leukemia who has received a regimen of chemotherapy. Which laboratory value would the nurse specifically note as a result of the massive cell destruction that occurred from the chemotherapy? 1. Anemia 2. Decreased platelets 3. Increased uric acid level 4. Decreased leukocyte count

3. Increased uric acid level Rationale: Hyperuricemia is especially common following treatment for leukemias and lymphomas because chemotherapy results in a massive cell kill. Although options 1, 2, and 4 also may be noted, an increased uric acid level is related specifically to cell destruction.

122.) A client who has begun taking fosinopril (Monopril) is very distressed, telling the nurse that he cannot taste food normally since beginning the medication 2 weeks ago. The nurse provides the best support to the client by: 1. Telling the client not to take the medication with food 2. Suggesting that the client taper the dose until taste returns to normal 3. Informing the client that impaired taste is expected and generally disappears in 2 to 3 months 4. Requesting that the health care provider (HCP) change the prescription to another brand of angiotensin-converting enzyme (ACE) inhibitor

3. Informing the client that impaired taste is expected and generally disappears in 2 to 3 months Rationale: ACE inhibitors, such as fosinopril, cause temporary impairment of taste (dysgeusia). The nurse can tell the client that this effect usually disappears in 2 to 3 months, even with continued therapy, and provide nutritional counseling if appropriate to avoid weight loss. Options 1, 2, and 4 are inappropriate actions. Taking this medication with or without food does not affect absorption and action. The dosage should never be tapered without HCP approval and the medication should never be stopped abruptly.

243.) A hospitalized client is having the dosage of clonazepam (Klonopin) adjusted. The nurse should plan to: 1. Weigh the client daily. 2. Observe for ecchymosis. 3. Institute seizure precautions. 4. Monitor blood glucose levels.

3. Institute seizure precautions. Rationale: Clonazepam is a benzodiazepine used as an anticonvulsant. During initial therapy and during periods of dosage adjustment, the nurse should initiate seizure precautions for the client. Options 1, 2, and 4 are not associated with the use of this medication.

230.) A client is placed on chloral hydrate (Somnote) for short-term treatment. Which nursing action indicates an understanding of the major side effect of this medication? 1. Monitoring neurological signs every 2 hours 2. Monitoring the blood pressure every 4 hours 3. Instructing the client to call for ambulation assistance 4. Lowering the bed and clearing a path to the bathroom at bedtime

3. Instructing the client to call for ambulation assistance Rationale: Chloral hydrate (a sedative-hypnotic) causes sedation and impairment of motor coordination; therefore, safety measures need to be implemented. The client is instructed to call for assistance with ambulation. Options 1 and 2 are not specifically associated with the use of this medication. Although option 4 is an appropriate nursing intervention, it is most important to instruct the client to call for assistance with ambulation.

241.) A client with a history of simple partial seizures is taking clorazepate (Tranxene), and asks the nurse if there is a risk of addiction. The nurse's response is based on the understanding that clorazepate: 1. Is not habit forming, either physically or psychologically 2. Leads to physical tolerance, but only after 10 or more years of therapy 3. Leads to physical and psychological dependence with prolonged high-dose therapy 4. Can result in psychological dependence only, because of the nature of the medication

3. Leads to physical and psychological dependence with prolonged high-dose therapy Rationale: Clorazepate is classified as an anticonvulsant, antianxiety agent, and sedative-hypnotic (benzodiazepine). One of the concerns with clorazepate therapy is that the medication can lead to physical or psychological dependence with prolonged therapy at high doses. For this reason, the amount of medication that is readily available to the client at any one time is restricted. **Eliminate options 2 and 4 first because of the closed-ended word "only"**

52.) A client with tuberculosis is being started on antituberculosis therapy with isoniazid (INH). Before giving the client the first dose, a nurse ensures that which of the following baseline studies has been completed? 1. Electrolyte levels 2. Coagulation times 3. Liver enzyme levels 4. Serum creatinine level

3. Liver enzyme levels Rationale: INH therapy can cause an elevation of hepatic enzyme levels and hepatitis. Therefore, liver enzyme levels are monitored when therapy is initiated and during the first 3 months of therapy. They may be monitored longer in the client who is greater than age 50 or abuses alcohol.

210.) Dantrolene (Dantrium) is prescribed for a client with a spinal cord injury for discomfort resulting from spasticity. The nurse tells the client about the importance of follow-up and the need for which blood study? 1. Creatinine level 2. Sedimentation rate 3. Liver function studies 4. White blood cell count

3. Liver function studies Rationale: Dantrolene can cause liver damage, and the nurse should monitor liver function studies. Baseline liver function studies are done before therapy starts, and regular liver function studies are performed throughout therapy. Dantrolene is discontinued if no relief of spasticity is achieved in 6 weeks.

59.) A client is diagnosed with an acute myocardial infarction and is receiving tissue plasminogen activator, alteplase (Activase, tPA). Which action is a priority nursing intervention? 1. Monitor for renal failure. 2. Monitor psychosocial status. 3. Monitor for signs of bleeding. 4. Have heparin sodium available.

3. Monitor for signs of bleeding. Rationale: Tissue plasminogen activator is a thrombolytic. Hemorrhage is a complication of any type of thrombolytic medication. The client is monitored for bleeding. Monitoring for renal failure and monitoring the client's psychosocial status are important but are not the most critical interventions. Heparin is given after thrombolytic therapy, but the question is not asking about follow-up medications.

151.) A client is being treated for acute congestive heart failure with intravenously administered bumetanide. The vital signs are as follows: blood pressure, 100/60 mm Hg; pulse, 96 beats/min; and respirations, 24 breaths/min. After the initial dose, which of the following is the priority assessment? 1. Monitoring weight loss 2. Monitoring temperature 3. Monitoring blood pressure 4. Monitoring potassium level

3. Monitoring blood pressure Rationale: Bumetanide is a loop diuretic. Hypotension is a common side effect associated with the use of this medication. The other options also require assessment but are not the priority. **priority ABCs—airway, breathing, and circulation**

201.) A nurse is preparing to administer eardrops to an infant. The nurse plans to: 1. Pull up and back on the ear and direct the solution onto the eardrum. 2. Pull down and back on the ear and direct the solution onto the eardrum. 3. Pull down and back on the ear and direct the solution toward the wall of the canal. 4. Pull up and back on the ear lobe and direct the solution toward the wall of the canal.

3. Pull down and back on the ear and direct the solution toward the wall of the canal. Rationale: When administering eardrops to an infant, the nurse pulls the ear down and straight back. In the adult or a child older than 3 years, the ear is pulled up and back to straighten the auditory canal. The medication is administered by aiming it at the wall of the canal rather than directly onto the eardrum.

37.) The client has begun medication therapy with pancrelipase (Pancrease MT). The nurse evaluates that the medication is having the optimal intended benefit if which effect is observed? 1. Weight loss 2. Relief of heartburn 3. Reduction of steatorrhea 4. Absence of abdominal pain

3. Reduction of steatorrhea Rationale: Pancrelipase (Pancrease MT) is a pancreatic enzyme used in clients with pancreatitis as a digestive aid. The medication should reduce the amount of fatty stools (steatorrhea). Another intended effect could be improved nutritional status. It is not used to treat abdominal pain or heartburn. Its use could result in weight gain but should not result in weight loss if it is aiding in digestion.

137.) A nurse is reinforcing instructions for a client regarding intranasal desmopressin acetate (DDAVP). The nurse tells the client that which of the following is a side effect of the medication? 1. Headache 2. Vulval pain 3. Runny nose 4. Flushed skin

3. Runny nose Rationale: Desmopressin administered by the intranasal route can cause a runny or stuffy nose. Headache, vulval pain, and flushed skin are side effects if the medication is administered by the intravenous (IV) route.

111.) A client arrives at the health care clinic and tells the nurse that he has been doubling his daily dosage of bupropion hydrochloride (Wellbutrin) to help him get better faster. The nurse understands that the client is now at risk for which of the following? 1. Insomnia 2. Weight gain 3. Seizure activity 4. Orthostatic hypotension

3. Seizure activity Rationale: Bupropion does not cause significant orthostatic blood pressure changes. Seizure activity is common in dosages greater than 450 mg daily. Bupropion frequently causes a drop in body weight. Insomnia is a side effect, but seizure activity causes a greater client risk.

95.) The nurse is reviewing the results of serum laboratory studies drawn on a client with acquired immunodeficiency syndrome who is receiving didanosine (Videx). The nurse interprets that the client may have the medication discontinued by the health care provider if which of the following significantly elevated results is noted? 1. Serum protein 2. Blood glucose 3. Serum amylase 4. Serum creatinine

3. Serum amylase Rationale: Didanosine (Videx) can cause pancreatitis. A serum amylase level that is increased 1.5 to 2 times normal may signify pancreatitis in the client with acquired immunodeficiency syndrome and is potentially fatal. The medication may have to be discontinued. The medication is also hepatotoxic and can result in liver failure.

98.) The nurse is assigned to care for a client with cytomegalovirus retinitis and acquired immunodeficiency syndrome who is receiving foscarnet. The nurse should check the latest results of which of the following laboratory studies while the client is taking this medication? 1. CD4 cell count 2. Serum albumin 3. Serum creatinine 4. Lymphocyte count

3. Serum creatinine Rationale: Foscarnet is toxic to the kidneys. Serum creatinine is monitored before therapy, two to three times per week during induction therapy, and at least weekly during maintenance therapy. Foscarnet may also cause decreased levels of calcium, magnesium, phosphorus, and potassium. Thus these levels are also measured with the same frequency.

211.) A client with epilepsy is taking the prescribed dose of phenytoin (Dilantin) to control seizures. A phenytoin blood level is drawn, and the results reveal a level of 35 mcg/ml. Which of the following symptoms would be expected as a result of this laboratory result? 1. Nystagmus 2. Tachycardia 3. Slurred speech 4. No symptoms, because this is a normal therapeutic level

3. Slurred speech Rationale: The therapeutic phenytoin level is 10 to 20 mcg/mL. At a level higher than 20 mcg/mL, involuntary movements of the eyeballs (nystagmus) appear. At a level higher than 30 mcg/mL, ataxia and slurred speech occur.

187.) A clinic nurse prepares to administer an MMR (measles, mumps, rubella) vaccine to a child. How is this vaccine best administered? 1. Intramuscularly in the deltoid muscle 2. Subcutaneously in the gluteal muscle 3. Subcutaneously in the outer aspect of the upper arm 4. Intramuscularly in the anterolateral aspect of the thigh

3. Subcutaneously in the outer aspect of the upper arm Rationale: The MMR vaccine is administered subcutaneously in the outer aspect of the upper arm. The gluteal muscle is most often used for intramuscular injections. The MMR vaccine is not administered by the intramuscular route.

46.) A postoperative client has received a dose of naloxone hydrochloride for respiratory depression shortly after transfer to the nursing unit from the postanesthesia care unit. After administration of the medication, the nurse checks the client for: 1. Pupillary changes 2. Scattered lung wheezes 3. Sudden increase in pain 4. Sudden episodes of diarrhea

3. Sudden increase in pain Rationale: Naloxone hydrochloride is an antidote to opioids and may also be given to the postoperative client to treat respiratory depression. When given to the postoperative client for respiratory depression, it may also reverse the effects of analgesics. Therefore, the nurse must check the client for a sudden increase in the level of pain experienced. Options 1, 2, and 4 are not associated with this medication.

58.) Isosorbide mononitrate (Imdur) is prescribed for a client with angina pectoris. The client tells the nurse that the medication is causing a chronic headache. The nurse appropriately suggests that the client: 1. Cut the dose in half. 2. Discontinue the medication. 3. Take the medication with food. 4. Contact the health care provider (HCP).

3. Take the medication with food. Rationale: Isosorbide mononitrate is an antianginal medication. Headache is a frequent side effect of isosorbide mononitrate and usually disappears during continued therapy. If a headache occurs during therapy, the client should be instructed to take the medication with food or meals. It is not necessary to contact the HCP unless the headaches persist with therapy. It is not appropriate to instruct the client to discontinue therapy or adjust the dosages.

245.) A client taking carbamazepine (Tegretol) asks the nurse what to do if he misses one dose. The nurse responds that the carbamazepine should be: 1. Withheld until the next scheduled dose 2. Withheld and the health care provider is notified immediately 3. Taken as long as it is not immediately before the next dose 4. Withheld until the next scheduled dose, which should then be doubled

3. Taken as long as it is not immediately before the next dose Rationale: Carbamazepine is an anticonvulsant that should be taken around the clock, precisely as directed. If a dose is omitted, the client should take the dose as soon as it is remembered, as long as it is not immediately before the next dose. The medication should not be double dosed. If more than one dose is omitted, the client should call the health care provider.

229.) A client who is taking lithium carbonate (Lithobid) is scheduled for surgery. The nurse informs the client that: 1. The medication will be discontinued a week before the surgery and resumed 1 week postoperatively. 2. The medication is to be taken until the day of surgery and resumed by injection immediately postoperatively. 3. The medication will be discontinued 1 to 2 days before the surgery and resumed as soon as full oral intake is allowed. 4. The medication will be discontinued several days before surgery and resumed by injection in the immediate postoperative period.

3. The medication will be discontinued 1 to 2 days before the surgery and resumed as soon as full oral intake is allowed. Rationale: The client who is on lithium carbonate must be off the medication for 1 to 2 days before a scheduled surgical procedure and can resume the medication when full oral intake is prescribed after the surgery. **lithium carbonate is an oral medication and is not given as an injection**

80.) A nurse is caring for a client who is taking phenytoin (Dilantin) for control of seizures. During data collection, the nurse notes that the client is taking birth control pills. Which of the following information should the nurse provide to the client? 1. Pregnancy should be avoided while taking phenytoin (Dilantin). 2. The client may stop taking the phenytoin (Dilantin) if it is causing severe gastrointestinal effects. 3. The potential for decreased effectiveness of the birth control pills exists while taking phenytoin (Dilantin). 4. The increased risk of thrombophlebitis exists while taking phenytoin (Dilantin) and birth control pills together.

3. The potential for decreased effectiveness of the birth control pills exists while taking phenytoin (Dilantin). Rationale: Phenytoin (Dilantin) enhances the rate of estrogen metabolism, which can decrease the effectiveness of some birth control pills. Options 1, 2, are 4 are not accurate.

129.) Megestrol acetate (Megace), an antineoplastic medication, is prescribed for the client with metastatic endometrial carcinoma. The nurse reviews the client's history and contacts the registered nurse if which diagnosis is documented in the client's history? 1. Gout 2. Asthma 3. Thrombophlebitis 4. Myocardial infarction

3. Thrombophlebitis Rationale: Megestrol acetate (Megace) suppresses the release of luteinizing hormone from the anterior pituitary by inhibiting pituitary function and regressing tumor size. Megestrol is used with caution if the client has a history of thrombophlebitis. **megestrol acetate is a hormonal antagonist enzyme and that a side effect is thrombotic disorders**

22.) A nurse is caring for a client after thyroidectomy and notes that calcium gluconate is prescribed for the client. The nurse determines that this medication has been prescribed to: 1. Treat thyroid storm. 2. Prevent cardiac irritability. 3. Treat hypocalcemic tetany. 4. Stimulate the release of parathyroid hormone.

3. Treat hypocalcemic tetany. Rationale: Hypocalcemia can develop after thyroidectomy if the parathyroid glands are accidentally removed or injured during surgery. Manifestations develop 1 to 7 days after surgery. If the client develops numbness and tingling around the mouth, fingertips, or toes or muscle spasms or twitching, the health care provider is notified immediately. Calcium gluconate should be kept at the bedside.

27.) Sildenafil (Viagra) is prescribed to treat a client with erectile dysfunction. A nurse reviews the client's medical record and would question the prescription if which of the following is noted in the client's history? 1. Neuralgia 2. Insomnia 3. Use of nitroglycerin 4. Use of multivitamins

3. Use of nitroglycerin Rationale: Sildenafil (Viagra) enhances the vasodilating effect of nitric oxide in the corpus cavernosum of the penis, thus sustaining an erection. Because of the effect of the medication, it is contraindicated with concurrent use of organic nitrates and nitroglycerin. Sildenafil is not contraindicated with the use of vitamins. Neuralgia and insomnia are side effects of the medication.

103.) A nurse is caring for a hospitalized client who has been taking clozapine (Clozaril) for the treatment of a schizophrenic disorder. Which laboratory study prescribed for the client will the nurse specifically review to monitor for an adverse effect associated with the use of this medication? 1. Platelet count 2. Cholesterol level 3. White blood cell count 4. Blood urea nitrogen level

3. White blood cell count Rationale: Hematological reactions can occur in the client taking clozapine and include agranulocytosis and mild leukopenia. The white blood cell count should be checked before initiating treatment and should be monitored closely during the use of this medication. The client should also be monitored for signs indicating agranulocytosis, which may include sore throat, malaise, and fever. Options 1, 2, and 4 are unrelated to this medication.

81.) A client with trigeminal neuralgia is being treated with carbamazepine (Tegretol). Which laboratory result would indicate that the client is experiencing an adverse reaction to the medication? 1. Sodium level, 140 mEq/L 2. Uric acid level, 5.0 mg/dL 3. White blood cell count, 3000 cells/mm3 4. Blood urea nitrogen (BUN) level, 15 mg/dL

3. White blood cell count, 3000 cells/mm3 Rationale: Adverse effects of carbamazepine (Tegretol) appear as blood dyscrasias, including aplastic anemia, agranulocytosis, thrombocytopenia, leukopenia, cardiovascular disturbances, thrombophlebitis, dysrhythmias, and dermatological effects. Options 1, 2, and 4 identify normal laboratory values.

Trough draw

30 minutes before scheduled dose

Peak draw

30-60 minutes after administration

118.) A nurse is caring for an older client with a diagnosis of myasthenia gravis and has reinforced self-care instructions. Which statement by the client indicates that further teaching is necessary? 1. "I rest each afternoon after my walk." 2. "I cough and deep breathe many times during the day." 3. "If I get abdominal cramps and diarrhea, I should call my doctor." 4. "I can change the time of my medication on the mornings that I feel strong."

4. "I can change the time of my medication on the mornings that I feel strong." Rationale: The client with myasthenia gravis should be taught that timing of anticholinesterase medication is critical. It is important to instruct the client to administer the medication on time to maintain a chemical balance at the neuromuscular junction. If not given on time, the client may become too weak to swallow. Options 1, 2, and 3 include the necessary information that the client needs to understand to maintain health with this neurological degenerative disease.

54.) A nurse reinforces discharge instructions to a postoperative client who is taking warfarin sodium (Coumadin). Which statement, if made by the client, reflects the need for further teaching? 1. "I will take my pills every day at the same time." 2. "I will be certain to avoid alcohol consumption." 3. "I have already called my family to pick up a Medic-Alert bracelet." 4. "I will take Ecotrin (enteric-coated aspirin) for my headaches because it is coated."

4. "I will take Ecotrin (enteric-coated aspirin) for my headaches because it is coated." Rationale: Ecotrin is an aspirin-containing product and should be avoided. Alcohol consumption should be avoided by a client taking warfarin sodium. Taking prescribed medication at the same time each day increases client compliance. The Medic-Alert bracelet provides health care personnel emergency information.

61.) A home health care nurse is visiting a client with elevated triglyceride levels and a serum cholesterol level of 398 mg/dL. The client is taking cholestyramine (Questran). Which of the following statements, if made by the client, indicates the need for further education? 1. "Constipation and bloating might be a problem." 2. "I'll continue to watch my diet and reduce my fats." 3. "Walking a mile each day will help the whole process." 4. "I'll continue my nicotinic acid from the health food store."

4. "I'll continue my nicotinic acid from the health food store." Rationale: Nicotinic acid, even an over-the-counter form, should be avoided because it may lead to liver abnormalities. All lipid-lowering medications also can cause liver abnormalities, so a combination of nicotinic acid and cholestyramine resin is to be avoided. Constipation and bloating are the two most common side effects. Walking and the reduction of fats in the diet are therapeutic measures to reduce cholesterol and triglyceride levels.

237.) A client who is on lithium carbonate (Lithobid) will be discharged at the end of the week. In formulating a discharge teaching plan, the nurse will instruct the client that it is most important to: 1. Avoid soy sauce, wine, and aged cheese. 2. Have the lithium level checked every week. 3. Take medication only as prescribed because it can become addicting. 4. Check with the psychiatrist before using any over-the-counter (OTC) medications or prescription medications.

4. Check with the psychiatrist before using any over-the-counter (OTC) medications or prescription medications. Rationale: Lithium is the medication of choice to treat manic-depressive illness. Many OTC medications interact with lithium, and the client is instructed to avoid OTC medications while taking lithium. Lithium is not addicting, and, although serum lithium levels need to be monitored, it is not necessary to check these levels every week. A tyramine-free diet is associated with monoamine oxidase inhibitors.

62.) A client is on nicotinic acid (niacin) for hyperlipidemia and the nurse provides instructions to the client about the medication. Which statement by the client would indicate an understanding of the instructions? 1. "It is not necessary to avoid the use of alcohol." 2. "The medication should be taken with meals to decrease flushing." 3. "Clay-colored stools are a common side effect and should not be of concern." 4. "Ibuprofen (Motrin) taken 30 minutes before the nicotinic acid should decrease the flushing."

4. "Ibuprofen (Motrin) taken 30 minutes before the nicotinic acid should decrease the flushing." Rationale: Flushing is a side effect of this medication. Aspirin or a nonsteroidal anti-inflammatory drug can be taken 30 minutes before taking the medication to decrease flushing. Alcohol consumption needs to be avoided because it will enhance this side effect. The medication should be taken with meals, this will decrease gastrointestinal upset. Taking the medication with meals has no effect on the flushing. Clay-colored stools are a sign of hepatic dysfunction and should be immediately reported to the health care provider (HCP).

218.) A film-coated form of diflunisal has been prescribed for a client for the treatment of chronic rheumatoid arthritis. The client calls the clinic nurse because of difficulty swallowing the tablets. Which initial instruction should the nurse provide to the client? 1. "Crush the tablets and mix them with food." 2. "Notify the health care provider for a medication change." 3. "Open the tablet and mix the contents with food." 4. "Swallow the tablets with large amounts of water or milk."

4. "Swallow the tablets with large amounts of water or milk." Rationale: Diflunisal may be given with water, milk, or meals. The tablets should not be crushed or broken open. Taking the medication with a large amount of water or milk should be tried before contacting the health care provider.

198.) Coal tar has been prescribed for a client with a diagnosis of psoriasis, and the nurse provides instructions to the client about the medication. Which statement by the client indicates a need for further instructions? 1. "The medication can cause phototoxicity." 2. "The medication has an unpleasant odor." 3. "The medication can stain the skin and hair." 4. "The medication can cause systemic effects."

4. "The medication can cause systemic effects." Rationale: Coal tar is used to treat psoriasis and other chronic disorders of the skin. It suppresses DNA synthesis, mitotic activity, and cell proliferation. It has an unpleasant odor, can frequently stain the skin and hair, and can cause phototoxicity. Systemic toxicity does not occur. **The name of the medication will assist in eliminating options 2 and 3**

87.) A client with acute muscle spasms has been taking baclofen (Lioresal). The client calls the clinic nurse because of continuous feelings of weakness and fatigue and asks the nurse about discontinuing the medication. The nurse should make which appropriate response to the client? 1. "You should never stop the medication." 2. "It is best that you taper the dose if you intend to stop the medication." 3. "It is okay to stop the medication if you think that you can tolerate the muscle spasms." 4. "Weakness and fatigue commonly occur and will diminish with continued medication use."

4. "Weakness and fatigue commonly occur and will diminish with continued medication use." Rationale: The client should be instructed that symptoms such as drowsiness, weakness, and fatigue are more intense in the early phase of therapy and diminish with continued medication use. The client should be instructed never to withdraw or stop the medication abruptly, because abrupt withdrawal can cause visual hallucinations, paranoid ideation, and seizures. It is best for the nurse to inform the client that these symptoms will subside and encourage the client to continue the use of the medication.

196.) A client has been prescribed amikacin (Amikin). Which of the following priority baseline functions should be monitored? 1. Apical pulse 2. Liver function 3. Blood pressure 4. Hearing acuity

4. Hearing acuity Rationale: Amikacin (Amikin) is an antibiotic. This medication can cause ototoxicity and nephrotoxicity; therefore, hearing acuity tests and kidney function studies should be performed before the initiation of therapy. Apical pulse, liver function studies, and blood pressure are not specifically related to the use of this medication.

226.) A client receiving lithium carbonate (Lithobid) complains of loose, watery stools and difficulty walking. The nurse would expect the serum lithium level to be which of the following? 1. 0.7 mEq/L 2. 1.0 mEq/L 3. 1.2 mEq/L 4. 1.7 mEq/L

4. 1.7 mEq/L Rationale: The therapeutic serum level of lithium ranges from 0.6 to 1.2 mEq/L. Serum lithium levels above the therapeutic level will produce signs of toxicity.

163.) A client with trigeminal neuralgia tells the nurse that acetaminophen (Tylenol) is taken on a frequent daily basis for relief of generalized discomfort. The nurse reviews the client's laboratory results and determines that which of the following indicates toxicity associated with the medication? 1. Sodium of 140 mEq/L 2. Prothrombin time of 12 seconds 3. Platelet count of 400,000 cells/mm3 4. A direct bilirubin level of 2 mg/dL

4. A direct bilirubin level of 2 mg/dL Rationale: In adults, overdose of acetaminophen (Tylenol) causes liver damage. Option 4 is an indicator of liver function and is the only option that indicates an abnormal laboratory value. The normal direct bilirubin is 0 to 0.4 mg/dL. The normal platelet count is 150,000 to 400,000 cells/mm3. The normal prothrombin time is 10 to 13 seconds. The normal sodium level is 135 to 145 mEq/L.

75.) A client with myasthenia gravis becomes increasingly weak. The health care provider prepares to identify whether the client is reacting to an overdose of the medication (cholinergic crisis) or increasing severity of the disease (myasthenic crisis). An injection of edrophonium (Enlon) is administered. Which of the following indicates that the client is in cholinergic crisis? 1. No change in the condition 2. Complaints of muscle spasms 3. An improvement of the weakness 4. A temporary worsening of the condition

4. A temporary worsening of the condition Rationale: An edrophonium (Enlon) injection, a cholinergic drug, makes the client in cholinergic crisis temporarily worse. This is known as a negative test. An improvement of weakness would occur if the client were experiencing myasthenia gravis. Options 1 and 2 would not occur in either crisis.

181.) A client is taking ticlopidine hydrochloride (Ticlid). The nurse tells the client to avoid which of the following while taking this medication? 1. Vitamin C 2. Vitamin D 3. Acetaminophen (Tylenol) 4. Acetylsalicylic acid (aspirin)

4. Acetylsalicylic acid (aspirin) Rationale: Ticlopidine hydrochloride is a platelet aggregation inhibitor. It is used to decrease the risk of thrombotic strokes in clients with precursor symptoms. Because it is an antiplatelet agent, other medications that precipitate or aggravate bleeding should be avoided during its use. Therefore, aspirin or any aspirin-containing product should be avoided.

56.) Heparin sodium is prescribed for the client. The nurse expects that the health care provider will prescribe which of the following to monitor for a therapeutic effect of the medication? 1. Hematocrit level 2. Hemoglobin level 3. Prothrombin time (PT) 4. Activated partial thromboplastin time (aPTT)

4. Activated partial thromboplastin time (aPTT) Rationale: The PT will assess for the therapeutic effect of warfarin sodium (Coumadin) and the aPTT will assess the therapeutic effect of heparin sodium. Heparin sodium doses are determined based on these laboratory results. The hemoglobin and hematocrit values assess red blood cell concentrations.

242.) A client who was started on anticonvulsant therapy with clonazepam (Klonopin) tells the nurse of increasing clumsiness and unsteadiness since starting the medication. The client is visibly upset by these manifestations and asks the nurse what to do. The nurse's response is based on the understanding that these symptoms: 1. Usually occur if the client takes the medication with food 2. Are probably the result of an interaction with another medication 3. Indicate that the client is experiencing a severe untoward reaction to the medication 4. Are worse during initial therapy and decrease or disappear with long-term use

4. Are worse during initial therapy and decrease or disappear with long-term use Rationale: Drowsiness, unsteadiness, and clumsiness are expected effects of the medication during early therapy. They are dose related and usually diminish or disappear altogether with continued use of the medication. It does not indicate that a severe side effect is occurring. It is also unrelated to interaction with another medication. The client is encouraged to take this medication with food to minimize gastrointestinal upset. **Eliminate options 2 and 3 first because they are comparable or alike and because of the word "severe" in option 3**

4.) The camp nurse asks the children preparing to swim in the lake if they have applied sunscreen. The nurse reminds the children that chemical sunscreens are most effective when applied: 1. Immediately before swimming 2. 15 minutes before exposure to the sun 3. Immediately before exposure to the sun 4. At least 30 minutes before exposure to the sun

4. At least 30 minutes before exposure to the sun Rationale: Sunscreens are most effective when applied at least 30 minutes before exposure to the sun so that they can penetrate the skin. All sunscreens should be reapplied after swimming or sweating.

178.) Methylergonovine (Methergine) is prescribed for a client with postpartum hemorrhage caused by uterine atony. Before administering the medication, the nurse checks which of the following as the important client parameter? 1. Temperature 2. Lochial flow 3. Urine output 4. Blood pressure

4. Blood pressure Rationale: Methylergonovine is an ergot alkaloid used for postpartum hemorrhage. It stimulates contraction of the uterus and causes arterial vasoconstriction. Ergot alkaloids are avoided in clients with significant cardiovascular disease, peripheral disease, hypertension, eclampsia, or preeclampsia. These conditions are worsened by the vasoconstrictive effects of the ergot alkaloids. The nurse would check the client's blood pressure before administering the medication and would follow agency protocols regarding withholding of the medication. Options 1, 2, and 3 are items that are checked in the postpartum period, but they are unrelated to the use of this medication.

175.) A nurse notes that a client is receiving lamivudine (Epivir). The nurse determines that this medication has been prescribed to treat which of the following? 1. Pancreatitis 2. Pharyngitis 3. Tonic-clonic seizures 4. Human immunodeficiency virus (HIV) infection

4. Human immunodeficiency virus (HIV) infection Rationale: Lamivudine is a nucleoside reverse transcriptase inhibitor and antiviral medication. It slows HIV replication and reduces the progression of HIV infection. It also is used to treat chronic hepatitis B and is used for prophylaxis in health care workers at risk of acquiring HIV after occupational exposure to the virus. **Note the letters "-vir" in the trade name for this medication**

19.) Tamoxifen is prescribed for the client with metastatic breast carcinoma. The nurse understands that the primary action of this medication is to: 1. Increase DNA and RNA synthesis. 2. Promote the biosynthesis of nucleic acids. 3. Increase estrogen concentration and estrogen response. 4. Compete with estradiol for binding to estrogen in tissues containing high concentrations of receptors.

4. Compete with estradiol for binding to estrogen in tissues containing high concentrations of receptors. Rationale: Tamoxifen is an antineoplastic medication that competes with estradiol for binding to estrogen in tissues containing high concentrations of receptors. Tamoxifen is used to treat metastatic breast carcinoma in women and men. Tamoxifen is also effective in delaying the recurrence of cancer following mastectomy. Tamoxifen reduces DNA synthesis and estrogen response.

217.) A health care provider prescribes auranofin (Ridaura) for a client with rheumatoid arthritis. Which of the following would indicate to the nurse that the client is experiencing toxicity related to the medication? 1. Joint pain 2. Constipation 3. Ringing in the ears 4. Complaints of a metallic taste in the mouth

4. Complaints of a metallic taste in the mouth Rationale: Ridaura is the one gold preparation that is given orally rather than by injection. Gastrointestinal reactions including diarrhea, abdominal pain, nausea, and loss of appetite are common early in therapy, but these usually subside in the first 3 months of therapy. Early symptoms of toxicity include a rash, purple blotches, pruritus, mouth lesions, and a metallic taste in the mouth.

94.) The client with acquired immunodeficiency syndrome has begun therapy with zidovudine (Retrovir, Azidothymidine, AZT, ZDV). The nurse carefully monitors which of the following laboratory results during treatment with this medication? 1. Blood culture 2. Blood glucose level 3. Blood urea nitrogen 4. Complete blood count

4. Complete blood count Rationale: A common side effect of therapy with zidovudine is leukopenia and anemia. The nurse monitors the complete blood count results for these changes. Options 1, 2, and 3 are unrelated to the use of this medication.

124.) A client with chronic renal failure is receiving ferrous sulfate (Feosol). The nurse monitors the client for which common side effect associated with this medication? 1. Diarrhea 2. Weakness 3. Headache 4. Constipation

4. Constipation Rationale: Feosol is an iron supplement used to treat anemia. Constipation is a frequent and uncomfortable side effect associated with the administration of oral iron supplements. Stool softeners are often prescribed to prevent constipation. **Focus on the name of the medication. Recalling that oral iron can cause constipation will easily direct you to the correct option.**

155.) Mycophenolate mofetil (CellCept) is prescribed for a client as prophylaxis for organ rejection following an allogeneic renal transplant. Which of the following instructions does the nurse reinforce regarding administration of this medication? 1. Administer following meals. 2. Take the medication with a magnesium-type antacid. 3. Open the capsule and mix with food for administration. 4. Contact the health care provider (HCP) if a sore throat occurs.

4. Contact the health care provider (HCP) if a sore throat occurs. Rationale: Mycophenolate mofetil should be administered on an empty stomach. The capsules should not be opened or crushed. The client should contact the HCP if unusual bleeding or bruising, sore throat, mouth sores, abdominal pain, or fever occurs because these are adverse effects of the medication. Antacids containing magnesium and aluminum may decrease the absorption of the medication and therefore should not be taken with the medication. The medication may be given in combination with corticosteroids and cyclosporine. **neutropenia can occur with this medication**

157.) A client receiving nitrofurantoin (Macrodantin) calls the health care provider's office complaining of side effects related to the medication. Which side effect indicates the need to stop treatment with this medication? 1. Nausea 2. Diarrhea 3. Anorexia 4. Cough and chest pain

4. Cough and chest pain Rationale: Gastrointestinal (GI) effects are the most frequent adverse reactions to this medication and can be minimized by administering the medication with milk or meals. Pulmonary reactions, manifested as dyspnea, chest pain, chills, fever, cough, and the presence of alveolar infiltrates on the x-ray, would indicate the need to stop the treatment. These symptoms resolve in 2 to 4 days following discontinuation of this medication. **Eliminate options 1, 2, and 3 because they are similar GI-related side effects. Also, use the ABCs— airway, breathing, and circulation**

132.) The client with non-Hodgkin's lymphoma is receiving daunorubicin (DaunoXome). Which of the following would indicate to the nurse that the client is experiencing a toxic effect related to the medication? 1. Fever 2. Diarrhea 3. Complaints of nausea and vomiting 4. Crackles on auscultation of the lungs

4. Crackles on auscultation of the lungs Rationale: Cardiotoxicity noted by abnormal electrocardiographic findings or cardiomyopathy manifested as congestive heart failure is a toxic effect of daunorubicin. Bone marrow depression is also a toxic effect. Nausea and vomiting are frequent side effects associated with the medication that begins a few hours after administration and lasts 24 to 48 hours. Fever is a frequent side effect, and diarrhea can occur occasionally. The other options, however, are not toxic effects. **keep in mind that the question is asking about a toxic effect and think: ABCs—airway, breathing, and circulation**

117.) A nurse has given the client taking ethambutol (Myambutol) information about the medication. The nurse determines that the client understands the instructions if the client immediately reports: 1. Impaired sense of hearing 2. Distressing gastrointestinal side effects 3. Orange-red discoloration of body secretions 4. Difficulty discriminating the color red from green

4. Difficulty discriminating the color red from green Rationale: Ethambutol causes optic neuritis, which decreases visual acuity and the ability to discriminate between the colors red and green. This poses a potential safety hazard when driving a motor vehicle. The client is taught to report this symptom immediately. The client is also taught to take the medication with food if gastrointestinal upset occurs. Impaired hearing results from antitubercular therapy with streptomycin. Orange-red discoloration of secretions occurs with rifampin (Rifadin).

190.) A child is hospitalized with a diagnosis of lead poisoning. The nurse assisting in caring for the child would prepare to assist in administering which of the following medications? 1. Activated charcoal 2. Sodium bicarbonate 3. Syrup of ipecac syrup 4. Dimercaprol (BAL in Oil)

4. Dimercaprol (BAL in Oil) Rationale: Dimercaprol is a chelating agent that is administered to remove lead from the circulating blood and from some tissues and organs for excretion in the urine. Sodium bicarbonate may be used in salicylate poisoning. Syrup of ipecac is used in the hospital setting in poisonings to induce vomiting. Activated charcoal is used to decrease absorption in certain poisoning situations. Note that dimercaprol is prepared with peanut oil, and hence should be avoided by clients with known or suspected peanut allergy.

203.) A nurse is preparing to give the postcraniotomy client medication for incisional pain. The family asks the nurse why the client is receiving codeine sulfate and not "something stronger." In formulating a response, the nurse incorporates the understanding that codeine: 1. Is one of the strongest opioid analgesics available 2. Cannot lead to physical or psychological dependence 3. Does not cause gastrointestinal upset or constipation as do other opioids 4. Does not alter respirations or mask neurological signs as do other opioids

4. Does not alter respirations or mask neurological signs as do other opioids Rationale: Codeine sulfate is the opioid analgesic often used for clients after craniotomy. It is frequently combined with a nonopioid analgesic such as acetaminophen for added effect. It does not alter the respiratory rate or mask neurological signs as do other opioids. Side effects of codeine include gastrointestinal upset and constipation. The medication can lead to physical and psychological dependence with chronic use. It is not the strongest opioid analgesic available.

159.) A nurse is caring for a client receiving morphine sulfate subcutaneously for pain. Because morphine sulfate has been prescribed for this client, which nursing action would be included in the plan of care? 1. Encourage fluid intake. 2. Monitor the client's temperature. 3. Maintain the client in a supine position. 4. Encourage the client to cough and deep breathe.

4. Encourage the client to cough and deep breathe. Rationale: Morphine sulfate suppresses the cough reflex. Clients need to be encouraged to cough and deep breathe to prevent pneumonia. **ABCs—airway, breathing, and circulation**

223.) A client with a psychotic disorder is being treated with haloperidol (Haldol). Which of the following would indicate the presence of a toxic effect of this medication? 1. Nausea 2. Hypotension 3. Blurred vision 4. Excessive salivation

4. Excessive salivation Rationale: Toxic effects include extrapyramidal symptoms (EPS) noted as marked drowsiness and lethargy, excessive salivation, and a fixed stare. Akathisia, acute dystonias, and tardive dyskinesia are also signs of toxicity. Hypotension, nausea, and blurred vision are occasional side effects.

231.) A client admitted to the hospital gives the nurse a bottle of clomipramine (Anafranil). The nurse notes that the medication has not been taken by the client in 2 months. What behaviors observed in the client would validate noncompliance with this medication? 1. Complaints of hunger 2. Complaints of insomnia 3. A pulse rate less than 60 beats per minute 4. Frequent handwashing with hot, soapy water

4. Frequent handwashing with hot, soapy water Rationale: Clomipramine is commonly used in the treatment of obsessive-compulsive disorder. Handwashing is a common obsessive-compulsive behavior. Weight gain is a common side effect of this medication. Tachycardia and sedation are side effects. Insomnia may occur but is seldom a side effect.

236.) A client is being treated for depression with amitriptyline hydrochloride. During the initial phases of treatment, the most important nursing intervention is: 1. Prescribing the client a tyramine-free diet 2. Checking the client for anticholinergic effects 3. Monitoring blood levels frequently because there is a narrow range between therapeutic and toxic blood levels of this medication 4. Getting baseline postural blood pressures before administering the medication and each time the medication is administered

4. Getting baseline postural blood pressures before administering the medication and each time the medication is administered Rationale: Amitriptyline hydrochloride is a tricyclic antidepressant often used to treat depression. It causes orthostatic changes and can produce hypotension and tachycardia. This can be frightening to the client and dangerous because it can result in dizziness and client falls. The client must be instructed to move slowly from a lying to a sitting to a standing position to avoid injury if these effects are experienced. The client may also experience sedation, dry mouth, constipation, blurred vision, and other anticholinergic effects, but these are transient and will diminish with time.

192.) A nurse is collecting medication information from a client, and the client states that she is taking garlic as an herbal supplement. The nurse understands that the client is most likely treating which of the following conditions? 1. Eczema 2. Insomnia 3. Migraines 4. Hyperlipidemia

4. Hyperlipidemia Rationale: Garlic is an herbal supplement that is used to treat hyperlipidemia and hypertension. An herbal supplement that may be used to treat eczema is evening primrose. Insomnia has been treated with both valerian root and chamomile. Migraines have been treated with feverfew.

73.) A client with myasthenia gravis is suspected of having cholinergic crisis. Which of the following indicate that this crisis exists? 1. Ataxia 2. Mouth sores 3. Hypotension 4. Hypertension

4. Hypertension Rationale: Cholinergic crisis occurs as a result of an overdose of medication. Indications of cholinergic crisis include gastrointestinal disturbances, nausea, vomiting, diarrhea, abdominal cramps, increased salivation and tearing, miosis, hypertension, sweating, and increased bronchial secretions.

76.) Carbidopa-levodopa (Sinemet) is prescribed for a client with Parkinson's disease, and the nurse monitors the client for adverse reactions to the medication. Which of the following indicates that the client is experiencing an adverse reaction? 1. Pruritus 2. Tachycardia 3. Hypertension 4. Impaired voluntary movements

4. Impaired voluntary movements Rationale: Dyskinesia and impaired voluntary movement may occur with high levodopa dosages. Nausea, anorexia, dizziness, orthostatic hypotension, bradycardia, and akinesia (the temporary muscle weakness that lasts 1 minute to 1 hour, also known as the "on-off phenomenon") are frequent side effects of the medication.

212.) Mannitol (Osmitrol) is being administered to a client with increased intracranial pressure following a head injury. The nurse assisting in caring for the client knows that which of the following indicates the therapeutic action of this medication? 1. Prevents the filtration of sodium and water through the kidneys 2. Prevents the filtration of sodium and potassium through the kidneys 3. Decreases water loss by promoting the reabsorption of sodium and water in the loop of Henle 4. Induces diuresis by raising the osmotic pressure of glomerular filtrate, thereby inhibiting tubular reabsorption of water and solutes

4. Induces diuresis by raising the osmotic pressure of glomerular filtrate, thereby inhibiting tubular reabsorption of water and solutes Rationale: Mannitol is an osmotic diuretic that induces diuresis by raising the osmotic pressure of glomerular filtrate, thereby inhibiting tubular reabsorption of water and solutes. It is used to reduce intracranial pressure in the client with head trauma.

225.) A nursing student is assigned to care for a client with a diagnosis of schizophrenia. Haloperidol (Haldol) is prescribed for the client, and the nursing instructor asks the student to describe the action of the medication. Which statement by the nursing student indicates an understanding of the action of this medication? 1. It is a serotonin reuptake blocker. 2. It inhibits the breakdown of released acetylcholine. 3. It blocks the uptake of norepinephrine and serotonin. 4. It blocks the binding of dopamine to the postsynaptic dopamine receptors in the brain.

4. It blocks the binding of dopamine to the postsynaptic dopamine receptors in the brain. Rationale: Haloperidol acts by blocking the binding of dopamine to the postsynaptic dopamine receptors in the brain. Imipramine hydrochloride (Tofranil) blocks the reuptake of norepinephrine and serotonin. Donepezil hydrochloride (Aricept) inhibits the breakdown of released acetylcholine. Fluoxetine hydrochloride (Prozac) is a potent serotonin reuptake blocker.

200.) A client is seen in the clinic for complaints of skin itchiness that has been persistent over the past several weeks. Following data collection, it has been determined that the client has scabies. Lindane is prescribed, and the nurse is asked to provide instructions to the client regarding the use of the medication. The nurse tells the client to: 1. Apply a thick layer of cream to the entire body. 2. Apply the cream as prescribed for 2 days in a row. 3. Apply to the entire body and scalp, excluding the face. 4. Leave the cream on for 8 to 12 hours and then remove by washing.

4. Leave the cream on for 8 to 12 hours and then remove by washing. Rationale: Lindane is applied in a thin layer to the entire body below the head. No more than 30 g (1 oz) should be used. The medication is removed by washing 8 to 12 hours later. Usually, only one application is required.

119.) A client with diabetes mellitus who has been controlled with daily insulin has been placed on atenolol (Tenormin) for the control of angina pectoris. Because of the effects of atenolol, the nurse determines that which of the following is the most reliable indicator of hypoglycemia? 1. Sweating 2. Tachycardia 3. Nervousness 4. Low blood glucose level

4. Low blood glucose level Rationale: β-Adrenergic blocking agents, such as atenolol, inhibit the appearance of signs and symptoms of acute hypoglycemia, which would include nervousness, increased heart rate, and sweating. Therefore, the client receiving this medication should adhere to the therapeutic regimen and monitor blood glucose levels carefully. Option 4 is the most reliable indicator of hypoglycemia.

44.) A client is receiving acetylcysteine (Mucomyst), 20% solution diluted in 0.9% normal saline by nebulizer. The nurse should have which item available for possible use after giving this medication? 1. Ambu bag 2. Intubation tray 3. Nasogastric tube 4. Suction equipment

4. Suction equipment Rationale: Acetylcysteine can be given orally or by nasogastric tube to treat acetaminophen overdose, or it may be given by inhalation for use as a mucolytic. The nurse administering this medication as a mucolytic should have suction equipment available in case the client cannot manage to clear the increased volume of liquefied secretions.

205.) A nurse is assisting in preparing to administer acetylcysteine (Mucomyst) to a client with an overdose of acetaminophen (Tylenol). The nurse prepares to administer the medication by: 1. Administering the medication subcutaneously in the deltoid muscle 2. Administering the medication by the intramuscular route in the gluteal muscle 3. Administering the medication by the intramuscular route, mixed in 10 mL of normal saline 4. Mixing the medication in a flavored ice drink and allowing the client to drink the medication through a straw

4. Mixing the medication in a flavored ice drink and allowing the client to drink the medication through a straw Rationale: Because acetylcysteine has a pervasive odor of rotten eggs, it must be disguised in a flavored ice drink. It is consumed preferably through a straw to minimize contact with the mouth. It is not administered by the intramuscular or subcutaneous route. **Knowing that the medication is a solution that is also used for nebulization treatments will assist you to select the option that indicates an oral route**

36.) The client has a PRN prescription for ondansetron (Zofran). For which condition should this medication be administered to the postoperative client? 1. Paralytic ileus 2. Incisional pain 3. Urinary retention 4. Nausea and vomiting

4. Nausea and vomiting Rationale: Ondansetron is an antiemetic used to treat postoperative nausea and vomiting, as well as nausea and vomiting associated with chemotherapy. The other options are incorrect.

143.) A client has just taken a dose of trimethobenzamide (Tigan). The nurse plans to monitor this client for relief of: 1. Heartburn 2. Constipation 3. Abdominal pain 4. Nausea and vomiting

4. Nausea and vomiting Rationale: Trimethobenzamide is an antiemetic agent used in the treatment of nausea and vomiting. The other options are incorrect.

17.) The client with ovarian cancer is being treated with vincristine (Oncovin). The nurse monitors the client, knowing that which of the following indicates a side effect specific to this medication? 1. Diarrhea 2. Hair loss 3. Chest pain 4. Numbness and tingling in the fingers and toes

4. Numbness and tingling in the fingers and toes Rationale: A side effect specific to vincristine is peripheral neuropathy, which occurs in almost every client. Peripheral neuropathy can be manifested as numbness and tingling in the fingers and toes. Depression of the Achilles tendon reflex may be the first clinical sign indicating peripheral neuropathy. Constipation rather than diarrhea is most likely to occur with this medication, although diarrhea may occur occasionally. Hair loss occurs with nearly all the antineoplastic medications. Chest pain is unrelated to this medication.

39.) The client with a gastric ulcer has a prescription for sucralfate (Carafate), 1 g by mouth four times daily. The nurse schedules the medication for which times? 1. With meals and at bedtime 2. Every 6 hours around the clock 3. One hour after meals and at bedtime 4. One hour before meals and at bedtime

4. One hour before meals and at bedtime Rationale: Sucralfate is a gastric protectant. The medication should be scheduled for administration 1 hour before meals and at bedtime. The medication is timed to allow it to form a protective coating over the ulcer before food intake stimulates gastric acid production and mechanical irritation. The other options are incorrect.

2.) Oral iron supplements are prescribed for a 6-year-old child with iron deficiency anemia. The nurse instructs the mother to administer the iron with which best food item? 1. Milk 2. Water 3. Apple juice 4. Orange juice

4. Orange juice Rationale: Vitamin C increases the absorption of iron by the body. The mother should be instructed to administer the medication with a citrus fruit or a juice that is high in vitamin C. Milk may affect absorption of the iron. Water will not assist in absorption. Orange juice contains a greater amount of vitamin C than apple juice.

15.) The client with small cell lung cancer is being treated with etoposide (VePesid). The nurse who is assisting in caring for the client during its administration understands that which side effect is specifically associated with this medication? 1. Alopecia 2. Chest pain 3. Pulmonary fibrosis 4. Orthostatic hypotension

4. Orthostatic hypotension Rationale: A side effect specific to etoposide is orthostatic hypotension. The client's blood pressure is monitored during the infusion. Hair loss occurs with nearly all the antineoplastic medications. Chest pain and pulmonary fibrosis are unrelated to this medication.

222.) A nurse has administered a dose of diazepam (Valium) to a client. The nurse would take which important action before leaving the client's room? 1. Giving the client a bedpan 2. Drawing the shades or blinds closed 3. Turning down the volume on the television 4. Per agency policy, putting up the side rails on the bed

4. Per agency policy, putting up the side rails on the bed Rationale: Diazepam is a sedative-hypnotic with anticonvulsant and skeletal muscle relaxant properties. The nurse should institute safety measures before leaving the client's room to ensure that the client does not injure herself or himself. The most frequent side effects of this medication are dizziness, drowsiness, and lethargy. For this reason, the nurse puts the side rails up on the bed before leaving the room to prevent falls. Options 1, 2, and 3 may be helpful measures that provide a comfortable, restful environment, but option 4 is the one that provides for the client's safety needs.

184.) A nurse preparing a client for surgery reviews the client's medication record. The client is to be nothing per mouth (NPO) after midnight. Which of the following medications, if noted on the client's record, should the nurse question? 1. Cyclobenzaprine (Flexeril) 2. Alendronate (Fosamax) 3. Allopurinol (Zyloprim) 4. Prednisone

4. Prednisone Rationale: Prednisone is a corticosteroid that can cause adrenal atrophy, which reduces the body's ability to withstand stress. Before and during surgery, dosages may be temporarily increased. Cyclobenzaprine is a skeletal muscle relaxant. Alendronate is a bone-resorption inhibitor. Allopurinol is an antigout medication.

189.) Prostaglandin E1 is prescribed for a child with transposition of the great arteries. The mother of the child asks the nurse why the child needs the medication. The nurse tells the mother that the medication: 1. Prevents hypercyanotic (blue or tet) spells 2. Maintains an adequate hormone level 3. Maintains the position of the great arteries 4. Provides adequate oxygen saturation and maintains cardiac output

4. Provides adequate oxygen saturation and maintains cardiac output Rationale: A child with transposition of the great arteries may receive prostaglandin E1 temporarily to increase blood mixing if systemic and pulmonary mixing are inadequate to maintain adequate cardiac output. Options 1, 2, and 3 are incorrect. In addition, hypercyanotic spells occur in tetralogy of Fallot. **Use the ABCs—airway, breathing, and circulation—to answer the question. The correct option addresses circulation**

13.) The client with squamous cell carcinoma of the larynx is receiving bleomycin intravenously. The nurse caring for the client anticipates that which diagnostic study will be prescribed? 1. Echocardiography 2. Electrocardiography 3. Cervical radiography 4. Pulmonary function studies

4. Pulmonary function studies Rationale: Bleomycin is an antineoplastic medication (Chemotheraputic Agents) that can cause interstitial pneumonitis, which can progress to pulmonary fibrosis. Pulmonary function studies along with hematological, hepatic, and renal function tests need to be monitored. The nurse needs to monitor lung sounds for dyspnea and crackles, which indicate pulmonary toxicity. The medication needs to be discontinued immediately if pulmonary toxicity occurs. Options 1, 2, and 3 are unrelated to the specific use of this medication.

144.) A client is taking docusate sodium (Colace). The nurse monitors which of the following to determine whether the client is having a therapeutic effect from this medication? 1. Abdominal pain 2. Reduction in steatorrhea 3. Hematest-negative stools 4. Regular bowel movements

4. Regular bowel movements Rationale: Docusate sodium is a stool softener that promotes the absorption of water into the stool, producing a softer consistency of stool. The intended effect is relief or prevention of constipation. The medication does not relieve abdominal pain, stop gastrointestinal (GI) bleeding, or decrease the amount of fat in the stools.

70.) Oxybutynin chloride (Ditropan XL) is prescribed for a client with neurogenic bladder. Which sign would indicate a possible toxic effect related to this medication? 1. Pallor 2. Drowsiness 3. Bradycardia 4. Restlessness

4. Restlessness Rationale: Toxicity (overdosage) of this medication produces central nervous system excitation, such as nervousness, restlessness, hallucinations, and irritability. Other signs of toxicity include hypotension or hypertension, confusion, tachycardia, flushed or red face, and signs of respiratory depression. Drowsiness is a frequent side effect of the medication but does not indicate overdosage.

66.) Trimethoprim-sulfamethoxazole (TMP-SMZ) is prescribed for a client. A nurse should instruct the client to report which symptom if it developed during the course of this medication therapy? 1. Nausea 2. Diarrhea 3. Headache 4. Sore throat

4. Sore throat Rationale: Clients taking trimethoprim-sulfamethoxazole (TMP-SMZ) should be informed about early signs of blood disorders that can occur from this medication. These include sore throat, fever, and pallor, and the client should be instructed to notify the health care provider if these symptoms occur. The other options do not require health care provider notification.

166.) Alendronate (Fosamax) is prescribed for a client with osteoporosis. The client taking this medication is instructed to: 1. Take the medication at bedtime. 2. Take the medication in the morning with breakfast. 3. Lie down for 30 minutes after taking the medication. 4. Take the medication with a full glass of water after rising in the morning.

4. Take the medication with a full glass of water after rising in the morning. Rationale: Precautions need to be taken with administration of alendronate to prevent gastrointestinal side effects (especially esophageal irritation) and to increase absorption of the medication. The medication needs to be taken with a full glass of water after rising in the morning. The client should not eat or drink anything for 30 minutes following administration and should not lie down after taking the medication.

67.) Phenazopyridine hydrochloride (Pyridium) is prescribed for a client for symptomatic relief of pain resulting from a lower urinary tract infection. The nurse reinforces to the client: 1. To take the medication at bedtime 2. To take the medication before meals 3. To discontinue the medication if a headache occurs 4. That a reddish orange discoloration of the urine may occur

4. That a reddish orange discoloration of the urine may occur Rationale: The nurse should instruct the client that a reddish-orange discoloration of urine may occur. The nurse also should instruct the client that this discoloration can stain fabric. The medication should be taken after meals to reduce the possibility of gastrointestinal upset. A headache is an occasional side effect of the medication and does not warrant discontinuation of the medication.

177.) A nurse is assisting in caring for a pregnant client who is receiving intravenous magnesium sulfate for the management of preeclampsia and notes that the client's deep tendon reflexes are absent. On the basis of this data, the nurse reports the finding and makes which determination? 1. The magnesium sulfate is effective. 2. The infusion rate needs to be increased. 3. The client is experiencing cerebral edema. 4. The client is experiencing magnesium toxicity.

4. The client is experiencing magnesium toxicity. Rationale: Magnesium toxicity can occur as a result of magnesium sulfate therapy. Signs of magnesium sulfate toxicity relate to the central nervous system depressant effects of the medication and include respiratory depression; loss of deep tendon reflexes; sudden decrease in fetal heart rate or maternal heart rate, or both; and sudden drop in blood pressure. Hyperreflexia indicates increased cerebral edema. An absence of reflexes indicates magnesium toxicity. The therapeutic serum level of magnesium for a client receiving magnesium sulfate ranges from 4 to 7.5 mEq/L (5 to 8 mg/dL).

232.) A client in the mental health unit is administered haloperidol (Haldol). The nurse would check which of the following to determine medication effectiveness? 1. The client's vital signs 2. The client's nutritional intake 3. The physical safety of other unit clients 4. The client's orientation and delusional status

4. The client's orientation and delusional status Rationale: Haloperidol is used to treat clients exhibiting psychotic features. Therefore, to determine medication effectiveness, the nurse would check the client's orientation and delusional status. Vital signs are routine and not specific to this situation. The physical safety of other clients is not a direct assessment of this client. Monitoring nutritional intake is not related to this situation.

Following the administration of sublingual nitroglycerin to a client experiencing an acute anginal attack, which assessment finding indicates to the nurse that the desired effect has been achieved? A) Client states chest pain is relieved. B) Client's pulse decreases from 120 to 90. C) Client's systolic blood pressure decreases from 180 to 90. D) Client's SaO2 level increases from 92% to 96%.

A) Client states chest pain is relieved. Nitroglycerin reduces myocardial oxygen consumption which decreases ischemia and reduces chest pain (A). (B and D) would also occur if the angina was relieved, but are not as significant as the client's subjective report of decreased pain. (C) may indicate a reduction in pain, or a potentially serious side effect of the medication.

99.) The client with acquired immunodeficiency syndrome and Pneumocystis jiroveci infection has been receiving pentamidine isethionate (Pentam 300). The client develops a temperature of 101° F. The nurse does further monitoring of the client, knowing that this sign would most likely indicate: 1. The dose of the medication is too low. 2. The client is experiencing toxic effects of the medication. 3. The client has developed inadequacy of thermoregulation. 4. The result of another infection caused by leukopenic effects of the medication.

4. The result of another infection caused by leukopenic effects of the medication. Rationale: Frequent side effects of this medication include leukopenia, thrombocytopenia, and anemia. The client should be monitored routinely for signs and symptoms of infection. Options 1, 2, and 3 are inaccurate interpretations.

121.) A client who is taking hydrochlorothiazide (HydroDIURIL, HCTZ) has been started on triamterene (Dyrenium) as well. The client asks the nurse why both medications are required. The nurse formulates a response, based on the understanding that: 1. Both are weak potassium-losing diuretics. 2. The combination of these medications prevents renal toxicity. 3. Hydrochlorothiazide is an expensive medication, so using a combination of diuretics is cost-effective. 4. Triamterene is a potassium-sparing diuretic, whereas hydrochlorothiazide is a potassium-losing diuretic.

4. Triamterene is a potassium-sparing diuretic, whereas hydrochlorothiazide is a potassium-losing diuretic. Rationale: Potassium-sparing diuretics include amiloride (Midamor), spironolactone (Aldactone), and triamterene (Dyrenium). They are weak diuretics that are used in combination with potassium-losing diuretics. This combination is useful when medication and dietary supplement of potassium is not appropriate. The use of two different diuretics does not prevent renal toxicity. Hydrochlorothiazide is an effective and inexpensive generic form of the thiazide classification of diuretics. **It is especially helpful to remember that hydrochlorothiazide is a potassium-losing diuretic and triamterene is a potassium-sparing diuretic**

160.) Meperidine hydrochloride (Demerol) is prescribed for the client with pain. Which of the following would the nurse monitor for as a side effect of this medication? 1. Diarrhea 2. Bradycardia 3. Hypertension 4. Urinary retention

4. Urinary retention Rationale: Meperidine hydrochloride (Demerol) is an opioid analgesic. Side effects of this medication include respiratory depression, orthostatic hypotension, tachycardia, drowsiness and mental clouding, constipation, and urinary retention.

140.) The client has a new prescription for metoclopramide (Reglan). On review of the chart, the nurse identifies that this medication can be safely administered with which condition? 1. Intestinal obstruction 2. Peptic ulcer with melena 3. Diverticulitis with perforation 4. Vomiting following cancer chemotherapy

4. Vomiting following cancer chemotherapy Rationale: Metoclopramide is a gastrointestinal (GI) stimulant and antiemetic. Because it is a GI stimulant, it is contraindicated with GI obstruction, hemorrhage, or perforation. It is used in the treatment of emesis after surgery, chemotherapy, and radiation.

104.) Disulfiram (Antabuse) is prescribed for a client who is seen in the psychiatric health care clinic. The nurse is collecting data on the client and is providing instructions regarding the use of this medication. Which is most important for the nurse to determine before administration of this medication? 1. A history of hyperthyroidism 2. A history of diabetes insipidus 3. When the last full meal was consumed 4. When the last alcoholic drink was consumed

4. When the last alcoholic drink was consumed Rationale: Disulfiram is used as an adjunct treatment for selected clients with chronic alcoholism who want to remain in a state of enforced sobriety. Clients must abstain from alcohol intake for at least 12 hours before the initial dose of the medication is administered. The most important data are to determine when the last alcoholic drink was consumed. The medication is used with caution in clients with diabetes mellitus, hypothyroidism, epilepsy, cerebral damage, nephritis, and hepatic disease. It is also contraindicated in severe heart disease, psychosis, or hypersensitivity related to the medication.

180.) A health care provider (HCP) writes a prescription for digoxin (Lanoxin), 0.25 mg daily. The nurse teaches the client about the medication and tells the client that it is important to: 1. Count the radial and carotid pulses every morning. 2. Check the blood pressure every morning and evening. 3. Stop taking the medication if the pulse is higher than 100 beats per minute. 4. Withhold the medication and call the HCP if the pulse is less than 60 beats per minute.

4. Withhold the medication and call the HCP if the pulse is less than 60 beats per minute. Rationale: An important component of taking this medication is monitoring the pulse rate; however, it is not necessary for the client to take both the radial and carotid pulses. It is not necessary for the client to check the blood pressure every morning and evening because the medication does not directly affect blood pressure. It is most important for the client to know the guidelines related to withholding the medication and calling the HCP. The client should not stop taking a medication.

185.) Which of the following herbal therapies would be prescribed for its use as an antispasmodic? Select all that apply. 1.Aloe 2.Kava 3.Ginger 4.Chamomile 5.Peppermint oil

4.Chamomile 5.Peppermint oil Rationale: Chamomile has a mild sedative effect and acts as an antispasmodic and anti-inflammatory. Peppermint oil acts as an antispasmodic and is used for irritable bowel syndrome. Topical aloe promotes wound healing. Aloe taken orally acts as a laxative. Kava has an anxiolytic, sedative, and analgesic effect. Ginger is effective in relieving nausea.

A client with hyperlipidemia receives a prescription for niacin (Niaspan). Which client teaching is most important for the nurse to provide? A) Expected duration of flushing. B) Symptoms of hyperglycemia. C) Diets that minimize GI irritation. D) Comfort measures for pruritis.

A) Expected duration of flushing. Flushing of the face and neck, lasting up to an hour, is a frequent reason for discontinuing niacin. Inclusion of this effect in client teaching (A) may promote compliance in taking the medication. While (B, C, and D) are all worthwhile instructions to help clients minimize or cope with normal side effects associated with niacin (Niaspan), flushing is intense and causes the most concern for the client.

A client is receiving digoxin for the onset of supraventricular tachycardia (SVT). Which laboratory findings should the nurse identify that places this client at risk? A) Hypokalemia. B) Hyponatremia. C) Hypercalcemia. D) Low uric acid levels.

A) Hypokalemia. Hypokalemia affects myocardial contractility, so (A) places this client at greatest risk for dysrhythmias that may be unresponsive to drug therapy. Although an imbalance of serum electrolytes, (B and C), can effect cardiac rhythm, the greatest risk for the client receiving digoxin is (A). (D) does not cause any interactions related to digoxin therapy for supraventricular tachycardia (SVT).

receiving Digoxin for the onset supraventricular tachycardia (SVT)

hypokalemia

After abdominal surgery, a male client is prescribed low molecular weight heparin. during administration of the medication, the client asks the nurse why he is receiving this medication. Which is the best response for the nurse to provide. A. This medication is a blood thinner given to prevent blood clot formation B. this medication enhances antibiotics to prevent infection C. This medication dissolves any glots that develop int he legs D. This abdominal injection assists in the healing of the abdominal wound

A

The nurse is reviewing the use of the patient-controlled analgesia (PCA) pump with a client in the immediate postoperative period. The client will receive morphine 1mg IV/hour basal rate with 1mg IV every 15minutes per PCA to toal 5mg IV max per hour. WHat assessment has the highest priority before initating the PCA pump

A life-threatening side effect of intravenous administration of morphine sulfate, an opiate narcotic, is respiratory depression (B). The PCA pump should be stopped and the healthcare provider notified if the client's respiratory rate falls below 12 breaths per minute, and the nurse should anticipate adjustments in the client's dosage before the PCA pump is restarted.

A client is being treated for osteoporosis with alendronate (fosamax), and the nurse has completed discharge teaching regarding medication administration. Which morning schedule would indicated to the nurse that the client teaching has been effective? A. take medication, go for a30 minutes morning walk, then eat breakfast B. take med, rest in bed for 30 min., eat breakfast, go for morning walk C. take medication with breakfast, then take a 30 minute morning walk D. go for a 30minute morning walk, eat breakfast, then take the med

A.

Where would a RN direct a pt to obtain more info about prescribed medications

A pharmacist

A client is receiving digoxin for the onset of supraventricular tachycardia (SVT). Which laboratory findings should the nurse identify that places this client at risk? A) Hypokalemia. B) Hyponatremia. C) Hypercalcemia. D) Low uric acid levels.

A. Hypokalemia affects myocardial contractility, so (A) places this client at greatest risk for dysrhythmias that may be unresponsive to drug therapy. Although an imbalance of serum electrolytes, (B and C), can effect cardiac rhythm, the greatest risk for the client receiving digoxin is (A). (D) does not cause any interactions related to digoxin therapy for supraventricular tachycardia (SVT).

An adult client is given a prescription for a scopolamine patch (Transderm Scop) to prevent motion sickness while on a cruise. Which information should the nurse provide to the client? A) Apply the patch at least 4 hours prior to departure. B) Change the patch every other day while on the cruise. C) Place the patch on a hairless area at the base of the skull. D) Drink no more than 2 alcoholic drinks during the cruise.

A) Apply the patch at least 4 hours prior to departure. Scopolamine, an anticholinergic agent, is used to prevent motion sickness and has a peak onset in 6 hours, so the client should be instructed to apply the patch at least 4 hours before departure (A) on the cruise ship. The duration of the transdermal patch is 72 hours, so (B) is not needed. Scolopamine blocks muscarinic receptors in the inner ear and to the vomiting center, so the best application site of the patch is behind the ear, not at the base of the skull (C). Anticholinergic medications are CNS depressants, so the client should be instructed to avoid alcohol (D) while using the patch.

Which medication should the nurse caution the client about taking while receiving an opioid analgesic

benzodiazepines

An adult client has prescriptions for morphine sulfate 2.5 mg IV q6h and ketorolac (Toradol) 30 mg IV q6h. Which action should the nurse implement? A) Administer both medications according to the prescription. B) Hold the ketorolac to prevent an antagonistic effect. C) Hold the morphine to prevent an additive drug interaction. D) Contact the healthcare provider to clarify the prescription.

A) Administer both medications according to the prescription. Morphine and ketorolac (Toradol) can be administered concurrently (A), and may produce an additive analgesic effect, resulting in the ability to reduce the dose of morphine, as seen in this prescription. Toradol is an antiinflammatory analgesic, and does not have an antagonistic effect with morphine (B), like an agonist-antagonist medication would have. An additive analgesic effect is desirable (C), because it allows a reduced dose of morphine. This prescription does not require any clarification, and can be administered safely as written (D).

Which instruction(s) should the nurse give to a female client who just received a prescription for oral metronidazole (Flagyl) for treatment of trichomonas vaginalis? (Select all that apply.) A) Increase fluid intake, especially cranberry juice. B) Do not abruptly discontinue the medication; taper use. C) Check blood pressure daily to detect hypertension. D) Avoid drinking alcohol while taking this medication. E) Use condoms until treatment is completed. F) Ensure that all sexual partners are treated at the same time.

A) Increase fluid intake, especially cranberry juice. D) Avoid drinking alcohol while taking this medication. E) Use condoms until treatment is completed. F) Ensure that all sexual partners are treated at the same time. Correct selections are (A, D, E, and F). Increased fluid intake and cranberry juice (A) are recommended for prevention and treatment of urinary tract infections, which frequently accompany vaginal infections. It is not necessary to taper use of this drug (B) or to check the blood pressure daily (C), as this condition is not related to hypertension. Flagyl can cause a disulfiram-like reaction if taken in conjunction with ingestion of alcohol, so the client should be instructed to avoid alcohol (D). All sexual partners should be treated at the same time (E) and condoms should be used until after treatment is completed to avoid reinfection (F).

A client is admitted to the coronary care unit with a medical diagnosis of acute myocardial infarction. Which medication prescription decreases both preload and afterload? A) Nitroglycerin. B) Propranolol (Inderal). C) Morphine. D) Captopril (Capoten).

A) Nitroglycerin. Nitroglycerin (A) is a nitrate that causes peripheral vasodilation and decreases contractility, thereby decreasing both preload and afterload. (B) is a beta adrenergic blocker that decreases both heart rate and contractility, but only decreases afterload. Morphine (C) decreases myocardial oxygen consumption and preload. Capoten (D) is an angiotensin converting enzyme (ACE) inhibitor that acts to prevents vasoconstriction, thereby decreasing blood pressure and afterload.

Which method of medication administration provides the client with the greatest first-pass effect? A) Oral. B) Sublingual. C) Intravenous. D) Subcutaneous.

A) Oral. The first-pass effect is a pharmacokinetic phenomenon that is related to the drug's metabolism in the liver. After oral (A) medications are absorbed from the gastrointestinal tract, the drug is carried directly to the liver via the hepatic portal circulation where hepatic inactivation occurs and reduces the bioavailability of the drug. Alternative method of administration, such as sublingual (B), IV (C), and subcutaneous (D) routes, avoid this first-pass effect.

A healthcare provider prescribes cephalexin monohydrate (Keflex) for a client with a postoperative infection. It is most important for the nurse to assess for what additional drug allergy before administering this prescription? A) Penicillins. B) Aminoglycosides. C) Erythromycins. D) Sulfonamides.

A) Penicillins. Cross-allergies exist between penicillins (A) and cephalosporins, such as cephalexin monohydrate (Keflex), so checking for penicillin allergy is a wise precaution before administering this drug.

A client is receiving ampicillin sodium (Omnipen) for a sinus infection. The nurse should instruct the client to notify the healthcare provider immediately if which symptom occurs? A) Rash. B) Nausea. C) Headache. D) Dizziness.

A) Rash. Rash (A) is the most common adverse effect of all penicillins, indicating an allergy to the medication which could result in anaphylactic shock, a medical emergency. (B, C, and D) are common side effects of penicillins that should subside after the body adjusts to the medication. These would not require immediate medical care unless the symptoms persist beyond the first few days or become extremely severe.

A client is receiving methylprednisolone (Solu-Medrol) 40 mg IV daily. The nurse anticipates an increase in which laboratory value as the result of this medication? A) Serum glucose. B) Serum calcium. C) Red blood cells. D) Serum potassium.

A) Serum glucose. Solu-Medrol is a corticosteroid with glucocorticoid and mineralocorticoid actions. These effects can lead to hyperglycemia (A), which is reflected as an increase in the serum glucose value. The client taking Solu-Medrol is at risk for hypocalcemia (B) and hypokalemia (D), which result in a decrease, not an increase, in the serum calcium and serum potassium levels. This medication does not adversely affect the RBC count (C).

A client is being treated for osteoporosis with alendronate (Fosamax), and the nurse has completed discharge teaching regarding medication administration. Which morning schedule would indicate to the nurse that the client teaching has been effective? A) Take medication, go for a 30 minute morning walk, then eat breakfast. B) Take medication, rest in bed for 30 minutes, eat breakfast, go for morning walk. C) Take medication with breakfast, then take a 30 minute morning walk. D) Go for a 30 minute morning walk, eat breakfast, then take medication.

A) Take medication, go for a 30 minute morning walk, then eat breakfast. Alendronate (Fosamax) is best absorbed when taken thirty minutes before eating in the morning. The client should also be advised to remain in an upright position for at least thirty minutes after taking the medication to reduce the risk of esophageal reflux and irritation. (A) is the best schedule to meet these needs. (B, C, and D) do not meet these criteria.

After abdominal surgery, a male client is prescribed low molecular weight heparin (LMWH). During administration of the medication, the client asks the nurse why he is receiving this medication. Which is the best response for the nurse to provide? A) This medication is a blood thinner given to prevent blood clot formation. B) This medication enhances antibiotics to prevent infection. C) This medication dissolves any clots that develop in the legs. D) This abdominal injection assists in the healing of the abdominal wound.

A) This medication is a blood thinner given to prevent blood clot formation. Unfractionated heparin or low molecular weight heparin (LMWH) is an anticoagulant that inhibits thrombin-mediated conversion of fibrinogen to fibrin and is given prophylactically to prevent postoperative venous thrombosis (A) or to treat pulmonary embolism or deep vein thrombosis following knee and abdominal surgeries. Heparin does not dissolve clots but prevents clot extension or further clot formation (C). The anticoagulant heparin does not prevent infection (B) or influence operative wound healing (D).

The nurse is transcribing a new prescription for spironolactone (aldactone) for a client who receives an angiotensin-converting enzyme inhibitor. Which action should the nurse implement A. verify both prescriptions with the HCP B. report the med interactions to the nurse manager C. hold the ACE inhibitor and give the new prescription D. Transcribe and send the prescription to the pharmacy

A. the concomitant use of an ACE inhibitor and a potassium-sparing diuretic sucha s spironolactone, should be given with caution b/c the two drugs may interact to cause an elevation in serum potassium levels.

A category X drug is prescribed for a young adult female client. Which instruction is most important for the nurse to teach this client? A) Use a reliable form of birth control. B) Avoid exposure to ultra violet light. C) Refuse this medication if planning pregnancy. D) Abstain from intercourse while on this drug.

A) Use a reliable form of birth control. Drugs classified in the category X place a client who is in the first trimester of pregnancy at risk for teratogenesis, so women in the childbearing years should be counseled to use a reliable form of birth control (A) during drug therapy. (B) is not a specific precaution with Category X drugs. The client should be encouraged to discuss plans for pregnancy with the healthcare provider, so a safer alternative prescription (C) can be provided if pregnancy occurs. Although the risk of birth defects during pregnancy explains the restriction of these drugs during pregnancy, (D) is not indicated.

A female client with rheumatoid arthritis take ibuprofen (Motrin) 600 mg PO 4 times a day. To prevent gastrointestinal bleeding, misoprostol (Cytotec) 100 mcg PO is prescribed. Which information is most important for the nurse to include in client teaching? A) Use contraception during intercourse. B) Ensure the Cytotec is taken on an empty stomach. C) Encourage oral fluid intake to prevent constipation. D) Take Cytotec 30 minutes prior to Motrin.

A) Use contraception during intercourse. Cytotec, a synthetic form of a prostaglandin, is classified as pregnancy Category X and can act as an abortifacient, so the client should be instructed to use contraception during intercourse (A) to prevent loss of an early pregnancy. (B) is not necessary. A common side effect of Cytotec is diarrhea, so constipation prevention strategies are usually not needed (C). Cytotec and Motrin should be taken together (D) to provide protective properties against gastrointestinal bleeding.

The nurse is transcribing a new prescription for spironolactone (Adactone) for a client who receives an angiotensin-converting enzyme (ACE) inhibitor. Which action should the nurse implement? A) Verify both prescriptions with the healthcare provider. B) Report the medication interactions to the nurse manager. C) Hold the ACE inhibitor and give the new prescription. D) Transcribe and send the prescription to the pharmacy.

A) Verify both prescriptions with the healthcare provider. The concomitant use of an angiotensin-converting enzyme (ACE) inhibitor and a potassium-sparing diuretic such as spironolactone, should be given with caution because the two drugs may interact to cause an elevation in serum potassium levels. Although the client is currently receiving an ACE inhibitor, verifying both prescriptions (A) alerts the healthcare provider about the client's medication regimen and provides the safest action before administering the medication. (B) is not necessary at this time. Holding the prescribed antihypertensive medication (C) places the client at risk. The nurse should inform the healthcare provider of the client's medication history before proceeding with the fulfillment of the prescription (D).

The nurse is preparing the 0900 dose of losartan (Cozaar), an angiotensin II receptor blocker (ARB), for a client with hypertension and heart failure. The nurse reviews the client's laboratory results and notes that the client's serum potassium level is 5.9 mEq/L. What action should the nurse take first? A) Withhold the scheduled dose. B) Check the client's apical pulse. C) Notify the healthcare provider. D) Repeat the serum potassium level.

A) Withhold the scheduled dose. The nurse should first withhold the scheduled dose of Cozaar (A) because the client is hyperkalemic (normal range 3.5 to 5 mEq/L). Although hypokalemia is usually associated with diuretic therapy in heart failure, hyperkalemia is associated with several heart failure medications, including ARBs. Because hyperkalemia may lead to cardiac dysrhythmias, the nurse should check the apical pulse for rate and rhythm (B), and the blood pressure. Before repeating the serum study (D), the nurse should notify the healthcare provider (C) of the findings.

A client with a dysrhythmia is to receive procainamide (Pronestyl) in 4 divided doses over the next 24 hours. What dosing schedule is best for the nurse to implement? A) q6h. B) QID. C) AC and bedtime. D) PC and bedtime.

A) q6h. Pronestyl is a class 1A antidysrhythmic. It should be taken around-the-clock (A) so that a stable blood level of the drug can be maintained, thereby decreasing the possibility of hypotension (an adverse effect) occurring because of too much of the drug circulating systemically at any particular time of day. (B, C, and D) do not provide an around-the-clock dosing schedule. Pronestyl may be given with food if GI distress is a problem, but an around-the-clock schedule should still be maintained.

A client is receiving ampicillin sodium (Omnipen) for a sinus infection. The nurse should instruct the client to notify the healthcare provider immediately if which symptom occurs? A) Rash. B) Nausea. C) Headache. D) Dizziness.

A. Rash (A) is the most common adverse effect of all penicillins, indicating an allergy to the medication which could result in anaphylactic shock, a medical emergency. (B, C, and D) are common side effects of penicillins that should subside after the body adjusts to the medication. These would not require immediate medical care unless the symptoms persist beyond the first few days or become extremely severe.

An adult client is given a prescription for a scopolamine patch (Transderm Scop) to prevent motion sickness while on a cruise. Which information should the nurse provide to the client? A) Apply the patch at least 4 hours prior to departure. B) Change the patch every other day while on the cruise. C) Place the patch on a hairless area at the base of the skull. D) Drink no more than 2 alcoholic drinks during the cruise.

A. Scopolamine, an anticholinergic agent, is used to prevent motion sickness and has a peak onset in 6 hours, so the client should be instructed to apply the patch at least 4 hours before departure (A) on the cruise ship. The duration of the transdermal patch is 72 hours, so (B) is not needed. Scolopamine blocks muscarinic receptors in the inner ear and to the vomiting center, so the best application site of the patch is behind the ear, not at the base of the skull (C). Anticholinergic medications are CNS depressants, so the client should be instructed to avoid alcohol (D) while using the patch.

A client is receiving methylprednisolone (Solu-Medrol) 40 mg IV daily. The nurse anticipates an increase in which laboratory value as the result of this medication? A) Serum glucose. B) Serum calcium. C) Red blood cells. D) Serum potassium.

A. Solu-Medrol is a corticosteroid with glucocorticoid and mineralocorticoid actions. These effects can lead to hyperglycemia (A), which is reflected as an increase in the serum glucose value. The client taking Solu-Medrol is at risk for hypocalcemia (B) and hypokalemia (D), which result in a decrease, not an increase, in the serum calcium and serum potassium levels. This medication does not adversely affect the RBC count (C).

The nurse is preparing the 0900 dose of losartan (Cozaar), an angiotensin II receptor blocker (ARB), for a client with hypertension and heart failure. The nurse reviews the client's laboratory results and notes that the client's serum potassium level is 5.9 mEq/L. What action should the nurse take first? A) Withhold the scheduled dose. B) Check the client's apical pulse. C) Notify the healthcare provider. D) Repeat the serum potassium level.

A. The nurse should first withhold the scheduled dose of Cozaar (A) because the client is hyperkalemic (normal range 3.5 to 5 mEq/L). Although hypokalemia is usually associated with diuretic therapy in heart failure, hyperkalemia is associated with several heart failure medications, including ARBs. Because hyperkalemia may lead to cardiac dysrhythmias, the nurse should check the apical pulse for rate and rhythm (B), and the blood pressure. Before repeating the serum study (D), the nurse should notify the healthcare provider (C) of the findings.

A category X drug is prescribed for a young adult female client. Which instruction is most important for the nurse to teach this client? A) Use a reliable form of birth control. B) Avoid exposure to ultra violet light. C) Refuse this medication if planning pregnancy. D) Abstain from intercourse while on this drug.

A. Drugs classified in the category X place a client who is in the first trimester of pregnancy at risk for teratogenesis, so women in the childbearing years should be counseled to use a reliable form of birth control (A) during drug therapy. (B) is not a specific precaution with Category X drugs. The client should be encouraged to discuss plans for pregnancy with the healthcare provider, so a safer alternative prescription (C) can be provided if pregnancy occurs. Although the risk of birth defects during pregnancy explains the restriction of these drugs during pregnancy, (D) is not indicated.

Which method of medication administration provides the client with the greatest first-pass effect? A) Oral. B) Sublingual. C) Intravenous. D) Subcutaneous.

A. The first-pass effect is a pharmacokinetic phenomenon that is related to the drug's metabolism in the liver. After oral (A) medications are absorbed from the gastrointestinal tract, the drug is carried directly to the liver via the hepatic portal circulation where hepatic inactivation occurs and reduces the bioavailability of the drug. Alternative method of administration, such as sublingual (B), IV (C), and subcutaneous (D) routes, avoid this first-pass effect.

identify as being at high risk for complications during the use of an opioid analgesic

a young adult with inflammatory bowel Dz

A healthcare provider prescrives cephalexin monhydrate (Keflex) for a client with a postoperative infection. It is most important for the nurse to assess for what additional drug allergy before administering this prescription? A. Penicillins B. Aminoglycosides C. Erythromycins D.Sulfonamides

A. Cross-allergies exist between penicillins and cephalosporines, such as keflex. so checking for penicillin allergy is a wise precaution

A female client with RA takes ibuprofen (motrin) 600mg PO 4xday. To preven GI bleeding, misoprostol (cytotec) 100mcg PO is prescribed. Which information is most important for the nurse to include in client teaching? A. use contraception during intercourse B. ensure the cytotec is taken on an empty stomach C. encourage oral fluid intake to prevent constipation D. take cytotec 30min prior to motrin

A. Cytotec, a synthetic form of prostaglandin, is classified as pregnancy category X and can act as an abortifacient, so the client should be instructed to use contraception during intercourse to prevent loss of early pregnancy

An adult client has prescriptions for morphine sulfate 2.5 mg IV q6h and ketorlac (toradol) 30mg IV q6h. which action should the nurse implement? A. administer both medications according to the prescription B. Hold the ketorolac to prevent an antagonist effect C. Hold the morphine to prevent an additive drug interaction D. Contact the healthcare provider to clarify the prescription

A. Morphine and ketorolac can be administered concurrently and may produce additive analgesic effect resulting in ability to reduce the dose of morphine, as seen in this prescription

A client with hyperlipidemia recieves a prescription for niacin (niaspan). which client teaching is most important for the nurse to provide a. expected duration of flushing b. symptoms of hyperglycemia c. diets that minimize gi irritation d. comfort measure for pruritis

A. flushing of the face and neck, lasting up to an hour, is a frequent reason for discontinuing niacin. inclusion of this effect in clietn teaching may promote compliance in taking the med.

A client is admitted to the coronary care unit with a medical diagnosis of acute myocardial infarction. which medication prescription decreases both preload and afterload a. nitroglycerin b. propranolol c. propranolol d. captopril

A. nitroglycerin is a nitrate that causes peripheral vasodilation and decreases contractility, thereby decreasing both preload and afterload

Following the administration of sublingual nitroglycerin to a client experiencing an acute anginal attack, which assessment finding indicates to the nurse that the desired effect has been achieved? A. Client states chest pain is relieved B. Client's pulse decreases from 120 to 90 C. Client's systolic blood pressure decreases from 180 to 90 D. Clients SaO2 level increases from 92% to 96%

A. nitroglycerin reduces mycocardial oxygen consumption which decreases ischemia and reduces chest pain

The client with a dysrhythmia is to receive procainamide (pronestyl) in 4 divided doses over the next 24 hours. What dosing schedule is best for the nurse to implement? A. q4h B. QID C. AC and bedtime D. PC and bedtime

A. q6h

Which instructions should the nurse give to a femail client who just recieved a prescription for oral metronidazole (flagyl) for treatment of trichomonas vaginalis (select all that apply) A. increase fluid intake, especially cranberry juice B. Do not abruptly discontinue the medication; taper use C. Check blood pressure daily to detect hypertension D. Avoid drinking alcohol while taking this medication E. Use condoms until treatment is completed F. Ensure that all sexual partners are treated at the same time

ADEF

•-olol

beta blockers

Which method of medication administration provides the client with the greatest first pass effect

inspect the clients' oral mucosa for ulcerations

Herpes Simplex 2 is treated with?

Acyclovir

After IM injection of penicillin the pt develops severe difficulty breathing and swollen tongue (Anaphylaxis)

Administer epinephrine

Which change in data indicates to the nurse that the desired effect of the angiotension II receptor antagonist valsartan (Diovan) has been achieved

blood pressure reduced from 160/90 to 130/80

Potassium sparing diuretic need to watch for hyperkalemia

Aldactone (spirinolactone)

Which of the pts would be at highest risk for an adverse reaction

An 84 yo with diabetes, HF, hypertension and takes 8 medications per day

A pt taking Digoxin is also prescribed Propanolol. The 2 drugs combined may cause a serious decrease in HR

An increased adverse effect

Patient on benzos

Answer is not narcan/ FLUMAZENIL

Pt is prescribed Lunesta for insomnia

Anterograde amnesia (memory loss of events right before taking drug)

Patient on Heparin going for surgery in a.m.,-priority

Assess patient for bleeds

A pt with diabetes gets a beta blocker (mask the signs of hypoglycemia and blocks beta receptors - inhibits glycogenolysis)

Atenolol (more selective)

A pt taking Levadopa/Carbadopa (dopaminergic) and experiences a frequent "on - off" episodes

Avoid high protein meals - competes with drug

When assessing an adolescent who recently overdosed on acetaminophen (Tylenol), it is most important for the nurse to assess for pain in which area of the body? A) Flank. B) Abdomen. C) Chest. D) Head.

B) Abdomen. Acetaminophen toxicity can result in liver damage; therefore, it is especially important for the nurse to assess for pain in the right upper quadrant of the abdomen (B), which might indicate liver damage. (A, C, and D) are not areas where pain would be anticipated.

The healthcare provider prescribes digitalis (Digoxin) for a client diagnosed with congestive heart failure. Which intervention should the nurse implement prior to administering the digoxin? A) Observe respiratory rate and depth. B) Assess the serum potassium level. C) Obtain the client's blood pressure. D) Monitor the serum glucose level.

B) Assess the serum potassium level. Hypokalemia (decreased serum potassium) will precipitate digitalis toxicity in persons receiving digoxin (B). (A and C) will not affect the administration of digoxin. (D) should be monitored if he/she is a diabetic and is perhaps receiving insulin.

Which medications should the nurse caution the client about taking while receiving an opioid analgesic? A) Antacids. B) Benzodiazepines. C) Antihypertensives. D) Oral antidiabetics.

B) Benzodiazepines. Respiratory depression increases with the concurrent use of opioid analgesics and other central nervous system depressant agents, such as alcohol, barbiturates, and benzodiazepines (B). (A and D) do not interact with opiates to produce adverse effects. Antihypertensives (C) may cause morphine-induced hypotension, but should not be withheld without notifying the healthcare provider.

While taking a nursing history, the client states, "I am allergic to penicillin." What related allergy to another type of antiinfective agent should the nurse ask the client about when taking the nursing history? A) Aminoglycosides. B) Cephalosporins. C) Sulfonamides. D) Tetracyclines.

B) Cephalosporins. Cross allergies exist between penicillins and cephalosporins (B). Penicillin allergies are unrelated to allergies associated with (A, C, or D).

The nurse is teaching a client with cancer about opioid management for intractable pain and tolerance related side effects. The nurse should prepare the client for which side effect that is most likely to persist during long-term use of opioids? A) Sedation. B) Constipation. C) Urinary retention. D) Respiratory depression.

B) Constipation. The client should be prepared to implement measures for constipation (B) which is the most likely persistent side effect related to opioid use. Tolerance to opiate narcotics is common, and the client may experience less sedation (A) and respiratory depression (D) as analgesic use continues. Opioids increase the tone in the urinary bladder sphincter, which causes retention (C) but may subside.

A client with congestive heart failure (CHF) is being discharged with a new prescription for the angiotensin-converting enzyme (ACE) inhibitor captopril (Capoten). The nurse's discharge instruction should include reporting which problem to the healthcare provider? A) Weight loss. B) Dizziness. C) Muscle cramps. D) Dry mucous membranes.

B) Dizziness. Angiotensin-converting enzyme (ACE) inhibitors are used in CHF to reduce afterload by reversing vasoconstriction common in heart failure. This vasodilation can cause hypotension and resultant dizziness (B). (A) is desired if fluid overload is present, and may occur as the result of effective combination drug therapy such as diuretics with ACE inhibitors. (C) often indicates hypokalemia in the client receiving diuretics. Excessive diuretic administration may result in fluid volume deficit, manifested by symptoms such as (D).

The nitrate isosorbide dinitrate (Isordil) is prescribed for a client with angina. Which instruction should the nurse include in this client's discharge teaching plan? A) Quit taking the medication if dizziness occurs. B) Do not get up quickly. Always rise slowly. C) Take the medication with food only. D) Increase your intake of potassium-rich foods.

B) Do not get up quickly. Always rise slowly. An expected side effect of nitrates is orthostatic hypotension and the nurse should address how to prevent it--by rising slowly (B). Dizziness is expected, and the client should not quit taking the medication without notifying the healthcare provider (A). (C and D) are not indicated when taking this medication.

Which dosing schedule should the nurse teach the client to observe for a controlled-release oxycodone prescription? A) As needed. B) Every 12 hours. C) Every 24 hours. D) Every 4 to 6 hours.

B) Every 12 hours. A controlled-release oxycodone provides long-acting analgesia to relieve moderate to severe pain, so a dosing schedule of every 12 hours (B) provides the best around-the-clock pain management. Controlled-release oxycodone is not prescribed for breakthrough pain on a PRN or as needed schedule (A). (C) is inadequate for continuous pain management. Using a schedule of every 4 to 6 hours (D) may jeopardize patient safety due to cumulative effects.

A peak and trough level must be drawn for a client receiving antibiotic therapy. What is the optimum time for the nurse to obtain the trough level? A) Sixty minutes after the antibiotic dose is administered. B) Immediately before the next antibiotic dose is given. C) When the next blood glucose level is to be checked. D) Thirty minutes before the next antibiotic dose is given.

B) Immediately before the next antibiotic dose is given. Trough levels are drawn when the blood level is at its lowest, which is typically just before the next dose is given (B). (A, C, and D) do not describe the optimum time for obtaining a trough level of an antibiotic.

A client with Parkinson's disease is taking carbidopa-levodopa (Sinemet). Which observation by the nurse should indicate that the desired outcome of the medication is being achieved? A) Decreased blood pressure. B) Lessening of tremors. C) Increased salivation. D) Increased attention span.

B) Lessening of tremors. Sinemet increases the amount of levodopa to the CNS (dopamine to the brain). Increased amounts of dopamine improve the symptoms of Parkinson's, such as involuntary movements, resting tremors (B), shuffling gait, etc. (A) is a side effect of Sinemet. Decreased drooling would be a desired effect, not (C). Sinemet does not affect (D).

A client with osteoarthritis receives a new prescription for celecoxib (Celebrex) orally for symptom management. The nurse notes the client is allergic to sulfa. Which action is most important for the nurse to implement prior to administering the first dose? A) Review the client's hemoglobin results. B) Notify the healthcare provider. C) Inquire about the reaction to sulfa. D) Record the client's vital signs.

B) Notify the healthcare provider. Celebrex contains a sulfur molecule, which can lead to an allergic reaction in individuals who are sensitive to sulfonamides, so the healthcare provider should be notified of the client's allergies (B). Although (A, C, and D) are important assessments, it is most important to notify the healthcare provider for an alternate prescription.

In teaching a client who had a liver transplant about cyclosporine (Sandimmune), the nurse should encourage the client to report which adverse response to the healthcare provider? A) Changes in urine color. B) Presence of hand tremors. C) Increasing body hirsutism. D) Nausea and vomiting.

B) Presence of hand tremors. Neurological complications, such as hand tremors (B), occur in about 50% of clients taking cyclosporine and should be reported. Although this drug can be nephrotoxic, (A) typically does not occur. (C and D) are common side effects, but are not usually severe.

Following heparin treatment for a pulmonary embolism, a client is being discharged with a prescription for warfarin (Coumadin). In conducting discharge teaching, the nurse advises the client to have which diagnostic test monitored regularly after discharge? A) Perfusion scan. B) Prothrombin Time (PT/INR). C) Activated partial thromboplastin (APTT). D) Serum Coumadin level (SCL).

B) Prothrombin Time (PT/INR). When used for a client with pulmonary embolus, the therapeutic goal for warfarin therapy is a PT 1½ to 2½ times greater than the control, or an INR of 2 to 3 (B). A perfusion might be performed to monitor lung function, but not monthly (A). APTT is monitored for the client receiving heparin therapy (C). A blood level for Coumadin cannot be measured (D).

A client with heart failure is prescribed spironolactone (Aldactone). Which information is most important for the nurse to provide to the client about diet modifications? A) Do not add salt to foods during preparation. B) Refrain for eating foods high in potassium. C) Restrict fluid intake to 1000 ml per day. D) Increase intake of milk and milk products.

B) Refrain for eating foods high in potassium. Spironolactone (Aldactone), an aldosterone antagonist, is a potassium-sparing diuretic, so a diet high in potassium should be avoided (B), including potassium salt substitutes, which can lead to hyperkalemia. Although (A) is a common diet modification in heart failure, the risk of hyperkalemia is more important with Aldactone. Restriction of fluids (C) or increasing milk and milk products (D) are not indicated with this prescription.

A 43-year-old female client is receiving thyroid replacement hormone following a thyroidectomy. What adverse effects associated with thyroid hormone toxicity should the nurse instruct the client to report promptly to the healthcare provider? A) Tinnitus and dizziness. B) Tachycardia and chest pain. C) Dry skin and intolerance to cold. D) Weight gain and increased appetite.

B) Tachycardia and chest pain. Thyroid replacement hormone increases the metabolic rate of all tissues, so common signs and symptoms of toxicity include tachycardia and chest pain (B). (A, C, and D) do not indicate a thyroid hormone toxicity.

The nurse is reviewing the use of the patient-controlled analgesia (PCA) pump with a client in the immediate postoperative period. The client will receive morphine 1 mg IV per hour basal rate with 1 mg IV every 15 minutes per PCA to total 5 mg IV maximally per hour. What assessment has the highest priority before initiating the PCA pump? A) The expiration date on the morphine syringe in the pump. B) The rate and depth of the client's respirations. C) The type of anesthesia used during the surgical procedure. D) The client's subjective and objective signs of pain.

B) The rate and depth of the client's respirations. A life-threatening side effect of intravenous administration of morphine sulfate, an opiate narcotic, is respiratory depression (B). The PCA pump should be stopped and the healthcare provider notified if the client's respiratory rate falls below 12 breaths per minute, and the nurse should anticipate adjustments in the client's dosage before the PCA pump is restarted. (A, C, and D) provide helpful information, but are not as high a priority as the assessment described in (B).

In evaluating the effects of lactulose (Cephulac), which outcome should indicate that the drug is performing as intended? A) An increase in urine output. B) Two or three soft stools per day. C) Watery, diarrhea stools. D) Increased serum bilirubin.

B) Two or three soft stools per day. Lactulose is administered to reduce blood ammonia by excretion of ammonia through the stool. Two to three stools a day indicate that lactulose is performing as intended (B). (A) would be expected if the patient received a diuretic. (C) would indicate an overdose of lactulose and is not expected. Lactulose does not affect (D).

Which symptoms are serious adverse effects of beta-adrenergic blockers such as propranolol (Inderal)? A) Headache, hypertension, and blurred vision. B) Wheezing, hypotension, and AV block. C) Vomiting, dilated pupils, and papilledema. D) Tinnitus, muscle weakness, and tachypnea.

B) Wheezing, hypotension, and AV block. (B) represents the most serious adverse effects of beta-blocking agents. AV block is generally associated with bradycardia and results in potentially life-threatening decreases in cardiac output. Additionally, wheezing secondary to bronchospasm and hypotension represent life-threatening respiratory and cardiac disorders. (A, C, and D) are not associated with beta-blockers.

The healthcare provider prescribes naloxone (Narcan) for a client in the emergency room. Which assessment data would indicate that the naloxone has been effective? The client's A) statement that the chest pain is better. B) respiratory rate is 16 breaths/minute. C) seizure activity has stopped temporarily. D) pupils are constricted bilaterally.

B) respiratory rate is 16 breaths/minute. Naloxone (Narcan) is a narcotic antagonist that reverses the respiratory depression effects of opiate overdose, so assessment of a normal respiratory rate (B) would indicate that the respiratory depression has been halted. (A, C, and D) are not related to naloxone (Narcan) administration.

Which symptoms are serious adverse effects of beta-adrenergic blockers such as propranolol (Inderal)? A) Headache, hypertension, and blurred vision. B) Wheezing, hypotension, and AV block. C) Vomiting, dilated pupils, and papilledema. D) Tinnitus, muscle weakness, and tachypnea.

B. (B) represents the most serious adverse effects of beta-blocking agents. AV block is generally associated with bradycardia and results in potentially life-threatening decreases in cardiac output. Additionally, wheezing secondary to bronchospasm and hypotension represent life-threatening respiratory and cardiac disorders. (A, C, and D) are not associated with beta-blockers.

The nitrate isosorbide dinitrate (Isordil) is prescribed for a client with angina. Which instruction should the nurse include in this client's discharge teaching plan? A) Quit taking the medication if dizziness occurs. B) Do not get up quickly. Always rise slowly. C) Take the medication with food only. D) Increase your intake of potassium-rich foods.

B. An expected side effect of nitrates is orthostatic hypotension and the nurse should address how to prevent it--by rising slowly (B). Dizziness is expected, and the client should not quit taking the medication without notifying the healthcare provider (A). (C and D) are not indicated when taking this medication.

A client with congestive heart failure (CHF) is being discharged with a new prescription for the angiotensin-converting enzyme (ACE) inhibitor captopril (Capoten). The nurse's discharge instruction should include reporting which problem to the healthcare provider? A) Weight loss. B) Dizziness. C) Muscle cramps. D) Dry mucous membranes.

B. Angiotensin-converting enzyme (ACE) inhibitors are used in CHF to reduce afterload by reversing vasoconstriction common in heart failure. This vasodilation can cause hypotension and resultant dizziness (B). (A) is desired if fluid overload is present, and may occur as the result of effective combination drug therapy such as diuretics with ACE inhibitors. (C) often indicates hypokalemia in the client receiving diuretics. Excessive diuretic administration may result in fluid volume deficit, manifested by symptoms such as (D).

A client with osteoarthritis receives a new prescription for celecoxib (Celebrex) orally for symptom management. The nurse notes the client is allergic to sulfa. Which action is most important for the nurse to implement prior to administering the first dose? A) Review the client's hemoglobin results. B) Notify the healthcare provider. C) Inquire about the reaction to sulfa. D) Record the client's vital signs.

B. Celebrex contains a sulfur molecule, which can lead to an allergic reaction in individuals who are sensitive to sulfonamides, so the healthcare provider should be notified of the client's allergies (B). Although (A, C, and D) are important assessments, it is most important to notify the healthcare provider for an alternate prescription.

In evaluating the effects of lactulose (Cephulac), which outcome should indicate that the drug is performing as intended? A) An increase in urine output. B) Two or three soft stools per day. C) Watery, diarrhea stools. D) Increased serum bilirubin.

B. Lactulose is administered to reduce blood ammonia by excretion of ammonia through the stool. Two to three stools a day indicate that lactulose is performing as intended (B). (A) would be expected if the patient received a diuretic. (C) would indicate an overdose of lactulose and is not expected. Lactulose does not affect (D).

The healthcare provider prescribes naloxone (Narcan) for a client in the emergency room. Which assessment data would indicate that the naloxone has been effective? The client's A) statement that the chest pain is better. B) respiratory rate is 16 breaths/minute. C) seizure activity has stopped temporarily. D) pupils are constricted bilaterally.

B. Naloxone (Narcan) is a narcotic antagonist that reverses the respiratory depression effects of opiate overdose, so assessment of a normal respiratory rate (B) would indicate that the respiratory depression has been halted. (A, C, and D) are not related to naloxone (Narcan) administration.

In teaching a client who had a liver transplant about cyclosporine (Sandimmune), the nurse should encourage the client to report which adverse response to the healthcare provider? A) Changes in urine color. B) Presence of hand tremors. C) Increasing body hirsutism. D) Nausea and vomiting.

B. Neurological complications, such as hand tremors (B), occur in about 50% of clients taking cyclosporine and should be reported. Although this drug can be nephrotoxic, (A) typically does not occur. (C and D) are common side effects, but are not usually severe.

A client with Parkinson's disease is taking carbidopa-levodopa (Sinemet). Which observation by the nurse should indicate that the desired outcome of the medication is being achieved? A) Decreased blood pressure. B) Lessening of tremors. C) Increased salivation. D) Increased attention span.

B. Sinemet increases the amount of levodopa to the CNS (dopamine to the brain). Increased amounts of dopamine improve the symptoms of Parkinson's, such as involuntary movements, resting tremors (B), shuffling gait, etc. (A) is a side effect of Sinemet. Decreased drooling would be a desired effect, not (C). Sinemet does not affect (D).

A client with heart failure is prescribed spironolactone (Aldactone). Which information is most important for the nurse to provide to the client about diet modifications? A) Do not add salt to foods during preparation. B) Refrain for eating foods high in potassium. C) Restrict fluid intake to 1000 ml per day. D) Increase intake of milk and milk products.

B. Spironolactone (Aldactone), an aldosterone antagonist, is a potassium-sparing diuretic, so a diet high in potassium should be avoided (B), including potassium salt substitutes, which can lead to hyperkalemia. Although (A) is a common diet modification in heart failure, the risk of hyperkalemia is more important with Aldactone. Restriction of fluids (C) or increasing milk and milk products (D) are not indicated with this prescription.

A peak and trough level must be drawn for a client receiving antibiotic therapy. What is the optimum time for the nurse to obtain the trough level? A) Sixty minutes after the antibiotic dose is administered. B) Immediately before the next antibiotic dose is given. C) When the next blood glucose level is to be checked. D) Thirty minutes before the next antibiotic dose is given.

B. Trough levels are drawn when the blood level is at its lowest, which is typically just before the next dose is given (B). (A, C, and D) do not describe the optimum time for obtaining a trough level of an antibiotic.

While taking a nursing history, the client states, "I am allergice to penicillin." what related allergy to another type of anti-infective agent should the nurse ask the client about when taking nursing history. A. aminoglycosides B. Cephalosporins C. Sulfonamides D. Tetracyclines

B. Cross allergies exist between penicillins and cephalosporins

A 43-year-old female client is receiving thyroid replacement hormone following a thyroidectomy. What adverse effects associated with thyroid hormone toxicity should the nurse instruct the client to report promptly to the healthcare provider? A) Tinnitus and dizziness. B) Tachycardia and chest pain. C) Dry skin and intolerance to cold. D) Weight gain and increased appetite.

B. Thyroid replacement hormone increases the metabolic rate of all tissues, so common signs and symptoms of toxicity include tachycardia and chest pain (B). (A, C, and D) do not indicate a thyroid hormone toxicity.

Following heparin treatment for a PE, a client is being discharged with a prescription for warfarin. In conducting discharge teaching, the nurse advises the client to have which diagnostic test monitored regularly? A. Perfusion scan B. Prothrombin time (PT/INR) C. Activated Partial thromboplastin (APTT) D. Serum Coumadin Level (SCL)

B. When used for a client with PE, the therapeutic goal for wafarin therapy is a PT 1/5 to 2/5 times greater than the control or an INR of 2-3. A perfusion might be preformed to monitor lung function but not monthly. APTT is monitored for Heparin.

When assessing an adolescent who recently overdosed on acetaminophen (tylonel), it is most important for the nurse to assess for pain in which area of the body a. flank b. abdomen c. chest d. head

B. acetaminophen toxicisty an result in liver damage; therefore, it is especially important for the nurse to assess for pain in the right upper quadrant of the abdomen (which might indicated liver damage)

The healthcare provider prescribes digitalis (Digoxin) for a client diagnosed with congestive heart failure. Which intervention should the nurse implement prior to adminstering the digoxin? A. Observe resp rate and depth B. assess the serum potassium level C. Obtain the clients blood pressure D. Monitor the serum glucose level

B. hypokalemia (decreased serum potassium) will precipitate digitalis toxicity in persons receiving digoxin

Which medications should the nurse caution the client about taking while receiving an opioid analgesic? A. Antacids. B. Benzodiasepines C. Antihypertensives D. Oral antidiabetics

B. respiratory depression increases with the concurrent use of opioid analgesics and other cns depressant agents, such as alcohol, barbiturates, and benzodiasepines

Pt is on Marplan (MAOIs - have the most food interactions) for depression

Bananas, smoked fish, and cheese

Trichomoniasis

Flagyl

nurse is teaching a client with cancer about opioid management for intractable pain and tolerance related side effects

constipation

A client asks the nurse if glipizide (Glucotrol) is an oral insulin. Which response should the nurse provide? A. Yes it is an oral insulin and has the same actions and properties as intermediate insulin B. Yes, it is an oral insulin and is distributed, metabolized, and excreted in the same manner as insulin C. No it is not an oral insulin and can be used only when some beta cell function is present D. No, it is not an oral insulin, but it is effective for those who are resistant to injectable insulins

C

A client asks the nurse if glipizide (Glucotrol) is an oral insulin. Which response should the nurse provide? A) "Yes, it is an oral insulin and has the same actions and properties as intermediate insulin." B) "Yes, it is an oral insulin and is distributed, metabolized, and excreted in the same manner as insulin." C) "No, it is not an oral insulin and can be used only when some beta cell function is present." D) "No, it is not an oral insulin, but it is effective for those who are resistant to injectable insulins."

C) "No, it is not an oral insulin and can be used only when some beta cell function is present." An effective oral form of insulin has not yet been developed (C) because when insulin is taken orally, it is destroyed by digestive enzymes. Glipizide (Glucotrol) is an oral hypoglycemic agent that enhances pancreatic production of insulin. (A, B, and D) do not provide accurate information.

A client is taking hydromorphone (Dilaudid) PO q4h at home. Following surgery, Dilaudid IV q4h PRN and butorphanol tartrate (Stadol) IV q4h PRN are prescribed for pain. The client received a dose of the Dilaudid IV four hours ago, and is again requesting pain medication. What intervention should the nurse implement? A) Alternate the two medications q4h PRN for pain. B) Alternate the two medications q2h PRN for pain. C) Administer only the Dilaudid q4h PRN for pain. D) Administer only the Stadol q4h PRN for pain.

C) Administer only the Dilaudid q4h PRN for pain. Dilaudid is an opioid agonist. Stadol is an opioid agonist-antagonist. Use of an agonist-antagonist for the client who has been receiving opioid agonists may result in abrupt withdrawal symptoms, and should be avoided (C). (A, B, and D) do not reflect good nursing practice.

A client's dose of isosorbide dinitrate (Imdur) is increased from 40 mg to 60 mg PO daily. When the client reports the onset of a headache prior to the next scheduled dose, which action should the nurse implement? A) Hold the next scheduled dose of Imdur 60 mg and administer a PRN dose of acetaminophen (Tylenol). B) Administer the 40 mg of Imdur and then contact the healthcare provider. C) Administer the 60 mg dose of Imdur and a PRN dose of acetaminophen (Tylenol). D) Do not administer the next dose of Imdur or any acetaminophen until notifying the healthcare provider.

C) Administer the 60 mg dose of Imdur and a PRN dose of acetaminophen (Tylenol). Imdur is a nitrate which causes vasodilation. This vasodilation can result in headaches, which can generally be controlled with acetaminophen (C) until the client develops a tolerance to this adverse effect. (A and B) may result in the onset of angina if a therapeutic level of Imdur is not maintained. Lying down (D) is less likely to reduce the headache than is a mild analgesic.

A client receiving albuterol (Proventil) tablets complains of nausea every evening with her 9 p.m. dose. What action should the nurse take to alleviate this side effect? A) Change the time of the dose. B) Hold the 9 p.m. dose. C) Administer the dose with a snack. D) Administer an antiemetic with the dose.

C) Administer the dose with a snack. Administering oral doses with food (C) helps minimize GI discomfort. (A) would be appropriate only if changing the time of the dose corresponds to meal times while at the same time maintaining an appropriate time interval between doses. (B) would disrupt the dosing schedule, and could result in a nontherapeutic serum level of the medication. (D) should not be attempted before other interventions, such as (C), have been proven ineffective in relieving the nausea.

The healthcare provider prescribes naproxen (Naproxen) twice daily for a client with osteoarthritis of the hands. The client tells the nurse that the drug does not seem to be effective after three weeks. Which is the best response for the nurse to provide? A) The frequency of the dosing is necessary to increase the effectiveness. B) Therapeutic blood levels of this drug are reached in 4 to 6 weeks. C) Another type of nonsteroidal antiinflammatory drug may be indicated. D) Systemic corticosteroids are the next drugs of choice for pain relief.

C) Another type of nonsteroidal antiinflammatory drug may be indicated. Individual responses to nonsteroidal antiinflammatory drugs are variable, so (C) is the best response. Naproxen is usually prescribed every 8 hours, so (A) is not indicated. The peak for naproxen is one to two hours, not (B). Corticosteroids are not indicated for osteoarthritis (D).

Which action is most important for the nurse to implement prior to the administration of the antiarrhythmic drug adenosine (Adenocard)? A) Assess pupillary response to light. B) Instruct the client that facial flushing may occur. C) Apply continuous cardiac monitoring. D) Request that family members leave the room.

C) Apply continuous cardiac monitoring. Adenosine (Adenocard) is an antiarrhythmic drug used to restore a normal sinus rhythm in clients with rapid supraventricular tachycardia. The client's heart rate should be monitored continuously (C) for the onset of additional arrhythmias while receiving adenosine. (A and B) are valuable nursing interventions, but are of less importance than monitoring for potentially fatal arrhythmias. Family members may be asked to leave the room because of the potential for an emergency situation (D), however, this is also of less priority than (C).

Which antidiarrheal agent should be used with caution in clients taking high dosages of aspirin for arthritis? A) Loperamide (Imodium). B) Probanthine (Propantheline). C) Bismuth subsalicylate (Pepto Bismol). D) Diphenoxylate hydrochloride with atropine (Lomotil).

C) Bismuth subsalicylate (Pepto Bismol). Bismuth subsalicylate (Pepto Bismol) contains a subsalicylate that increases the potential for salicylate toxicity when used concurrently with aspirin (acetylsalicylic acid, another salicylate preparation). (A, B, and D) do not pose the degree of risk of drug interaction with aspirin as Pepto Bismol would.

A client is receiving metoprolol (Lopressor SR). What assessment is most important for the nurse to obtain? A) Temperature. B) Lung sounds. C) Blood pressure. D) Urinary output.

C) Blood pressure. It is most important to monitor the blood pressure (C) of clients taking this medication because Lopressor is an antianginal, antiarrhythmic, antihypertensive agent. While (A and B) are important data to obtain on any client, they are not as important for a client receiving Lopressor as (C). Intake and output ratios and daily weights should be monitored while taking Lopressor to assess for signs and symptoms of congestive heart failure, but (D) alone does not have the importance of (C).

A postoperative client has been receiving a continuous IV infusion of meperidine (Demerol) 35 mg/hr for four days. The client has a PRN prescription for Demerol 100 mg PO q3h. The nurse notes that the client has become increasingly restless, irritable and confused, stating that there are bugs all over the walls. What action should the nurse take first? A) Administer a PRN dose of the PO meperidine (Demerol). B) Administer naloxone (Narcan) IV per PRN protocol. C) Decrease the IV infusion rate of the meperidine (Demerol) per protocol. D) Notify the healthcare provider of the client's confusion and hallucinations.

C) Decrease the IV infusion rate of the meperidine (Demerol) per protocol. The client is exhibiting symptoms of Demerol toxicity, which is consistent with the large dose of Demerol received over four days. (C) is the most effective action to immediately decrease the amount of serum Demerol. (A) will increase the toxic level of medication in the bloodstream. Naloxone (B) is an opioid antagonist that is used during an opioid overdose, but it is not beneficial during Demerol toxicity and can precipitate seizures. The healthcare provider should be notified (D), but that is not the initial action the nurse should take; first the amount of drug infusing should be decreased.

The nurse is assessing a client who is experiencing anaphylaxis from an insect sting. Which prescription should the nurse prepare to administer this client? A) Dopamine. B) Ephedrine. C) Epinephrine. D) Diphenhydramine.

C) Epinephrine. Epinephrine (C) is an adrenergic agent that stimulate beta receptors to increase cardiac automaticity in cardiac arrest and relax bronchospasms in anaphylaxis. Dopamine (A) is a vasopressor used to treat clients with shock. Ephedrine (B) causes peripheral vasoconstriction and is used in the treatment of nasal congestion. Diphenhydramine (D) is an antihistamine decongestant used in the treatment of mild allergic reactions and motion sickness.

A medication that is classified as a beta-1 agonist is most commonly prescribed for a client with which condition? A) Glaucoma. B) Hypertension. C) Heart failure. D) Asthma.

C) Heart failure. Beta-1 agonists improve cardiac output by increasing the heart rate and blood pressure and are indicated in heart failure (C), shock, atrioventricular block dysrhythmias, and cardiac arrest. Glaucoma (A) is managed using adrenergic agents and beta-adrenergic blocking agents. Beta-1 blocking agents are used in the management of hypertension (B). Medications that stimulate beta-2 receptors in the bronchi are effective for bronchoconstriction in respiratory disorders, such as asthma (D).

Which nursing intervention is most important when caring for a client receiving the antimetabolite cytosine arabinoside (Arc-C) for chemotherapy? A) Hydrate the client with IV fluids before and after infusion. B) Assess the client for numbness and tingling of extremities. C) Inspect the client's oral mucosa for ulcerations. D) Monitor the client's urine pH for increased acidity.

C) Inspect the client's oral mucosa for ulcerations. Cytosine arabinoside (Arc-C) affects the rapidly growing cells of the body, therefore stomatitis and mucosal ulcerations are key signs of antimetabolite toxicity (C). (A, B, and D) are not typical interventions associated with the administration of antimetabolites.

A client who was prescribed atorvastatin (Lipitor) one month ago calls the triage nurse at the clinic complaining of muscle pain and weakness in his legs. Which statement reflects the correct drug-specific teaching the nurse should provide to this client? A) Increase consumption of potassium-rich foods since low potassium levels can cause muscle spasms. B) Have serum electrolytes checked at the next scheduled appointment to assess hyponatremia, a cause of cramping. C) Make an appointment to see the healthcare provider, because muscle pain may be an indication of a serious side effect. D) Be sure to consume a low-cholesterol diet while taking the drug to enhance the effectiveness of the drug.

C) Make an appointment to see the healthcare provider, because muscle pain may be an indication of a serious side effect. Myopathy, suggested by the leg pain and weakness, is a serious, and potentially life-threatening, complication of Lipitor, and should be evaluated immediately by the healthcare provider (C). Although electrolyte imbalances such as (A or B) can cause muscle spasms in some cases, this is not the likely cause of leg pain in the client receiving Lipitor, and evaluation by the healthcare provider should not be delayed for any reason. A low-cholesterol diet is recommended for those taking Lipitor since the drug is used to lower total cholesterol (D), but diet is not related to the leg pain symptom.

The nurse is assessing the effectiveness of high dose aspirin therapy for an 88-year-old client with arthritis. The client reports that she can't hear the nurse's questions because her ears are ringing. What action should the nurse implement? A) Refer the client to an audiologist for evaluation of her hearing. B) Advise the client that this is a common side effect of aspirin therapy. C) Notify the healthcare provider of this finding immediately. D) Ask the client to turn off her hearing aid during the exam.

C) Notify the healthcare provider of this finding immediately. Tinnitus is an early sign of salicylate toxicity. The healthcare provider should be notified immediately (C), and the medication discontinued. (A and D) are not needed, and (B) is inaccurate.

A client has myxedema, which results from a deficiency of thyroid hormone synthesis in adults. The nurse knows that which medication should be contraindicated for this client? A) Liothyronine (Cytomel) to replace iodine. B) Furosemide (Lasix) for relief of fluid retention. C) Pentobarbital sodium (Nembutal Sodium) for sleep. D) Nitroglycerin (Nitrostat) for angina pain.

C) Pentobarbital sodium (Nembutal Sodium) for sleep. Persons with myxedema are dangerously hypersensitive to narcotics, barbiturates (C), and anesthetics. They do tolerate liothyronine (Cytomel) (A) and usually receive iodine replacement therapy. These clients are also susceptible to heart problems such as angina for which nitroglycerin (Nitrostat) (D) would be indicated, and congestive heart failure for which furosemide (Lasix) (B) would be indicated.

Which nursing diagnosis is important to include in the plan of care for a client receiving the angiotensin-2 receptor antagonist irbesartan (Avapro)? A) Fluid volume deficit. B) Risk for infection. C) Risk for injury. D) Impaired sleep patterns.

C) Risk for injury. Avapro is an antihypertensive agent, which acts by blocking vasoconstrictor effects at various receptor sites. This can cause hypotension and dizziness, placing the client at high risk for injury (C). Avapro does not act as a diuretic (A), impact the immune system (B), or alter sleep patterns (D).

An antacid (Maalox) is prescribed for a client with peptic ulcer disease. The nurse knows that the purpose of this medication is to A) decrease production of gastric secretions. B) produce an adherent barrier over the ulcer. C) maintain a gastric pH of 3.5 or above. D) decrease gastric motor activity.

C) maintain a gastric pH of 3.5 or above. The objective of antacids is to neutralize gastric acids and keep pH of 3.5 or above (C) which is necessary for pepsinogen inactivity. (A) is the purpose of H2 receptor antagonists (cimetidine, ranitidine). (B) is the purpose of sucralfate (Carafate). (D) is the purpose of anticholinergic drugs which are often used in conjunction with antacids to allow the antacid to remain in the stomach longer.

An antacid (maalox) is prescribed for a client with PUD. The nurse knows that the purpose of this medication is to A. Decrease production of gastric secretions B. produce an adherent barrier over the ulcer C. Maintain a gastric pH of 3.5 or above D. decrease gastric motor activity

C.

Which action is most important for the nurse to implement prior to the administration of the antiarrhythmic drug adenosine (Adenocard)? A) Assess pupillary response to light. B) Instruct the client that facial flushing may occur. C) Apply continuous cardiac monitoring. D) Request that family members leave the room.

C. Adenosine (Adenocard) is an antiarrhythmic drug used to restore a normal sinus rhythm in clients with rapid supraventricular tachycardia. The client's heart rate should be monitored continuously (C) for the onset of additional arrhythmias while receiving adenosine. (A and B) are valuable nursing interventions, but are of less importance than monitoring for potentially fatal arrhythmias. Family members may be asked to leave the room because of the potential for an emergency situation (D), however, this is also of less priority than (C).

A client receiving albuterol (Proventil) tablets complains of nausea every evening with her 9 p.m. dose. What action should the nurse take to alleviate this side effect? A) Change the time of the dose. B) Hold the 9 p.m. dose. C) Administer the dose with a snack. D) Administer an antiemetic with the dose.

C. Administering oral doses with food (C) helps minimize GI discomfort. (A) would be appropriate only if changing the time of the dose corresponds to meal times while at the same time maintaining an appropriate time interval between doses. (B) would disrupt the dosing schedule, and could result in a nontherapeutic serum level of the medication. (D) should not be attempted before other interventions, such as (C), have been proven ineffective in relieving the nausea.

A medication that is classified as a beta-1 agonist is most commonly prescribed for a client with which condition? A) Glaucoma. B) Hypertension. C) Heart failure. D) Asthma.

C. Beta-1 agonists improve cardiac output by increasing the heart rate and blood pressure and are indicated in heart failure (C), shock, atrioventricular block dysrhythmias, and cardiac arrest. Glaucoma (A) is managed using adrenergic agents and beta-adrenergic blocking agents. Beta-1 blocking agents are used in the management of hypertension (B). Medications that stimulate beta-2 receptors in the bronchi are effective for bronchoconstriction in respiratory disorders, such as asthma (D).

Which nursing intervention is most important when caring for a client receiving the antimetabolite cytosine arabinoside (Arc-C) for chemotherapy? A) Hydrate the client with IV fluids before and after infusion. B) Assess the client for numbness and tingling of extremities. C) Inspect the client's oral mucosa for ulcerations. D) Monitor the client's urine pH for increased acidity.

C. Cytosine arabinoside (Arc-C) affects the rapidly growing cells of the body, therefore stomatitis and mucosal ulcerations are key signs of antimetabolite toxicity (C). (A, B, and D) are not typical interventions associated with the administration of antimetabolites.

A client is taking hydromorphone (Dilaudid) PO q4h at home. Following surgery, Dilaudid IV q4h PRN and butorphanol tartrate (Stadol) IV q4h PRN are prescribed for pain. The client received a dose of the Dilaudid IV four hours ago, and is again requesting pain medication. What intervention should the nurse implement? A) Alternate the two medications q4h PRN for pain. B) Alternate the two medications q2h PRN for pain. C) Administer only the Dilaudid q4h PRN for pain. D) Administer only the Stadol q4h PRN for pain.

C. Dilaudid is an opioid agonist. Stadol is an opioid agonist-antagonist. Use of an agonist-antagonist for the client who has been receiving opioid agonists may result in abrupt withdrawal symptoms, and should be avoided (C). (A, B, and D) do not reflect good nursing practice.

A client's dose of isosorbide dinitrate (Imdur) is increased from 40 mg to 60 mg PO daily. When the client reports the onset of a headache prior to the next scheduled dose, which action should the nurse implement? A) Hold the next scheduled dose of Imdur 60 mg and administer a PRN dose of acetaminophen (Tylenol). B) Administer the 40 mg of Imdur and then contact the healthcare provider. C) Administer the 60 mg dose of Imdur and a PRN dose of acetaminophen (Tylenol). D) Do not administer the next dose of Imdur or any acetaminophen until notifying the healthcare provider.

C. Imdur is a nitrate which causes vasodilation. This vasodilation can result in headaches, which can generally be controlled with acetaminophen (C) until the client develops a tolerance to this adverse effect. (A and B) may result in the onset of angina if a therapeutic level of Imdur is not maintained. Lying down (D) is less likely to reduce the headache than is a mild analgesic.

The healthcare provider prescribes naproxen (Naproxen) twice daily for a client with osteoarthritis of the hands. The client tells the nurse that the drug does not seem to be effective after three weeks. Which is the best response for the nurse to provide? A) The frequency of the dosing is necessary to increase the effectiveness. B) Therapeutic blood levels of this drug are reached in 4 to 6 weeks. C) Another type of nonsteroidal antiinflammatory drug may be indicated. D) Systemic corticosteroids are the next drugs of choice for pain relief.

C. Individual responses to nonsteroidal antiinflammatory drugs are variable, so (C) is the best response. Naproxen is usually prescribed every 8 hours, so (A) is not indicated. The peak for naproxen is one to two hours, not (B). Corticosteroids are not indicated for osteoarthritis (D).

Which antidiarrheal agent should be used with caution in clients taking high dosages of aspirin for arthritis? A) Loperamide (Imodium). B) Probanthine (Propantheline). C) Bismuth subsalicylate (Pepto Bismol). D) Diphenoxylate hydrochloride with atropine (Lomotil).

C. Bismuth subsalicylate (Pepto Bismol) contains a subsalicylate that increases the potential for salicylate toxicity when used concurrently with aspirin (acetylsalicylic acid, another salicylate preparation). (A, B, and D) do not pose the degree of risk of drug interaction with aspirin as Pepto Bismol would.

The nurse is assessing a client who is experiencing anaphylaxis from an insect sting. Which prescription should the nurse prepare to administer to this client A. Dopamine B. Ephedrine C. Epinephrine D. Diphenhydramine

C. Epinephrine (C) is an adrenergic agent that stimulate beta receptors to increase cardiac automaticity in cardiac arrest and relax bronchospasms in anaphylaxis. Dopamine (A) is a vasopressor used to treat clients with shock. Ephedrine (B) causes peripheral vasoconstriction and is used in the treatment of nasal congestion. Diphenhydramine (D) is an antihistamine decongestant used in the treatment of mild allergic reactions and motion sickness.

A client is receiving metoprolol (Lopressor SR). What assessment is most important for the nurse to obtain? A) Temperature. B) Lung sounds. C) Blood pressure. D) Urinary output.

C. It is most important to monitor the blood pressure (C) of clients taking this medication because Lopressor is an antianginal, antiarrhythmic, antihypertensive agent. While (A and B) are important data to obtain on any client, they are not as important for a client receiving Lopressor as (C). Intake and output ratios and daily weights should be monitored while taking Lopressor to assess for signs and symptoms of congestive heart failure, but (D) alone does not have the importance of (C).

The nurse is assessing the effectiveness of high dose aspirin therapy for an 88-year-old client with arthritis. The client reports that she can't hear the nurse's questions because her ears are ringing. What action should the nurse implement? A) Refer the client to an audiologist for evaluation of her hearing. B) Advise the client that this is a common side effect of aspirin therapy. C) Notify the healthcare provider of this finding immediately. D) Ask the client to turn off her hearing aid during the exam.

C. Tinnitus is an early sign of salicylate toxicity. The healthcare provider should be notified immediately (C), and the medication discontinued. (A and D) are not needed, and (B) is inaccurate.

admission to the emergency center, an adult client with acute status asthmaticus is prescribed this series of medications In which order

albuterol (Proventil) puffs salmeterol (Serevent Diskus) prednisone (Deltasone) orally gentamicin (Garamycinim) IM

Which nursing diagnosis is important to include in the plan of care for a client recieving the angiotensin II receptor antagonist irbesartan (avapro)? A. Fluid volume deficit B.Risk for infection C. Risk for injury D. Impaired sleep patterns

C. Avapro is an antihypertensive agent, which acts by blocking vasoconstrictor effects at various receptor sites. This can cause hypotension and dizziness, placing the client at high risk for injury

A postoperative client has been recieving a continuous IV infusion of meperidine (demerol) 35mg/hr for four days. The client has a PRN prescription for Demorol 100mg PO Q3H. The nurse notes that the client has become increasingly restless, irritable and confused, stating that there are bugs all over the walls. What action should the nurse take FIRST? A. Administer a PRN dose of the PO meperidine (demorol) B. Administer naloxone (narcan) IV per PRN protocol C. Decrease the IV infusion rate of the demerol per protocol D. notify the healthcare provider of the clients confusion and hallucinations

C. The client is exhibiting symptoms of demerol toxicity, which is consistent with the large dose of demerol recieved over four days. C. is the most effective action to immediately decrease the amount of serum demerol.

A client has myxedema, which results from a deficiency of thyroid hormone synthesis in adults. The nurse knows that which medication should be contraindicated for this client? A. liothyronine (cytomel) to replace iodine B. Furosemide (Lasix) for relief of fluid retention C. Pentobarbital sodium for sleep D. nitroglycerin for angina pain

C. persons with myxedema are dangerously hypersensitive to narcotics, barbiturates, and anesthetics. They do not tolerate liothyronine and usually receive iodine replacement therapy. These clients are also suceptable to heart problems such as angina for which nitroglycerine would be indicated and and congestive heart failure for which furosemide would be indicated

Upon admission to the emergency center, an adult client with acute status asthmaticus is prescribed this series of medications. In which order should the nurse administer the prescribed medications? (Arrange from first to last.) A) Prednisone (Deltasone) orally. B) Gentamicin (Garamycin) IM. C) Albuterol (Proventil) puffs. D) Salmeterol (Serevent Diskus).

CDAB

A client in renal failure asks why he is being given antacids

Calcium and aluminum antacids bind phosphates and help to keep phosphates from being absorbed into blood stream thereby preventing rising phosphate levels, and must be taken with meals

A client in renal failure asks why he is being given antacids

Calcium and aluminum antacids bind phosphates and help to keep phosphates from being absorbed into blood stream thereby preventing rising phosphate levels, and must be taken with meals

Drawing insulin

Clear (regular) first then cloudy (NPH)

RN receives a handwritten medication order - can't read

Contact prescriber to clarify order

•-done

opioid analgesics

The RN is evaluating the kidney fx of an 82 yo pt before administration of medications -

Creatinine clearance - a measure of how the kidneys are fx by excreting creatinine. (BUN - good indicator of volume and dehydration) (Creatinine - is reflective very much by your BUN or level of hydration)

A client who has been taking levodopa PO TID to control the symptoms of Parkinsons disease has a new prescription for sustained release levodopa/carbidopa (sinemet 25/100 PO BID. The client took his levodopa at 0800. Which instruction should the nurse include in the teaching plan for this client? A. take the first dose of sinemet today, as soon as your prescription is filld B. Since you already took your levodopa, wait until tomorrow to take the sinemet C. Take both drugs for the 1st week, then switch to taking only the sinemet. D. You can begin taking the sinemet this evening, but do not take anymore levodopa

D

Which client should the nurse identify as being at the highest risk for complications during the use of an opioid analgesic? A. an older client with type 2 diabetes B. A client with chronic rheumatoid arthritis C. A client with a open compound fracture D. A young adult with inflammatory bowel disease

D

Which client should the nurse identify as being at highest risk for complications during the use of an opioid analgesic? A) An older client with Type 2 diabetes mellitus. B) A client with chronic rheumatoid arthritis. C) A client with a open compound fracture. D) A young adult with inflammatory bowel disease.

D) A young adult with inflammatory bowel disease. The principal indication for opioid use is acute pain, and a client with inflammatory bowel disease (D) is at risk for toxic megacolon or paralytic ileus related to slowed peristalsis, a side effect of morphine. Adverse effects of morphine do not pose as great a risk for (A, B, and C) as the client with bowel disease.

A client is receiving clonidine (Catapres) 0.1 mg/24hr via transdermal patch. Which assessment finding indicates that the desired effect of the medication has been achieved? A) Client denies recent episodes of angina. B) Change in peripheral edema from +3 to +1. C) Client denies recent nausea or vomiting. D) Blood pressure has changed from 180/120 to 140/70.

D) Blood pressure has changed from 180/120 to 140/70. Catapres acts as a centrally-acting analgesic and antihypertensive agent. (D) indicates a reduction in hypertension. Catapres does not affect (A, B, or C), so these findings do not indicate desired outcomes of Catapres.

Which change in data indicates to the nurse that the desired effect of the angiotensin II receptor antagonist valsartan (Diovan) has been achieved? A) Dependent edema reduced from +3 to +1. B) Serum HDL increased from 35 to 55 mg/dl. C) Pulse rate reduced from 150 to 90 beats/minute. D) Blood pressure reduced from 160/90 to 130/80.

D) Blood pressure reduced from 160/90 to 130/80. Diovan is an angiotensin receptor blocker, prescribed for the treatment of hypertension. The desired effect is a decrease in blood pressure (D). (A, B, and C) do not describe effects of Diovan.

A client is admitted to the hospital for diagnostic testing for possible myasthenia gravis. The nurse prepares for intravenous administration of edrophonium chloride (Tensilon). What is the expected outcome for this client following administration of this pharmacologic agent? A) Progressive difficulty with swallowing. B) Decreased respiratory effort. C) Improvement in generalized fatigue. D) Decreased muscle weakness.

D) Decreased muscle weakness. Administration of edrophonium chloride (Tensilon), a cholinergic agent, will temporarily reduce muscle weakness (D), the most common complaint of newly-diagnosed clients with myasthenia gravis. This medication is used to diagnose myasthenia gravis due to its short duration of action. This drug would temporarily reverse (A and B), not increase these symptoms. (C) is not a typical complaint of clients with myasthenia gravis, but weakness of specific muscles, especially after prolonged use, is a common symptom.

Which drug is used as a palliative treatment for a client with tumor-induced spinal cord compression? A) Morphine Sulfate (Duromorph). B) Ibuprofen (Advil). C) Amitriptyline (Amitril). D) Dexamethasone (Decadron).

D) Dexamethasone (Decadron). Dexamethasone (D) is a palliative treatment modality to manage symptoms related to compression due to tumor growth. Morphine sulphate (A) is an opioid analgesic used in oncology to manage severe or intractable pain. Ibuprofen (B), a nonsteroidal antiinflammatory drug (NSAID), provides relief for mild to moderate pain, suppression of inflammation, and reduction of fever. Amitriptyline (C), a tricyclic antidepressant, is often prescribed for pain related to neuropathic origin and provides a reduction in opioid dosage.

A client receiving Doxorubicin (Adriamycin) intravenously (IV) complains of pain at the insertion site, and the nurse notes edema at the site. Which intervention is most important for the nurse to implement? A) Assess for erythema. B) Administer the antidote. C) Apply warm compresses. D) Discontinue the IV fluids.

D) Discontinue the IV fluids. Doxorubicin is an antineoplastic agent that causes inflammation, blistering, and necrosis of tissue upon extravasation. First, all IV fluids should be discontinued at the site (D) to prevent further tissue damage by the vesicant. Erythema is one sign of infiltration and should be noted, but edema and pain at the infusion site require stopping the IV fluids (A). Although an antidote may be available (B), additional fluids contribute to the trauma of the subcutaneous tissues. Depending on the type of vesicant, warm or cold compresses (C) may be prescribed after the infusion is discontinued.

An older client with a decreased percentage of lean body mass is likely to receive a prescription that is adjusted based on which pharmacokinetic process? A) Absorption. B) Metabolism. C) Elimination. D) Distribution.

D) Distribution. A decreased lean body mass in an older adult affects the distribution of drugs (D), which affects the pharmacokinetics of drugs. Decreased gastric pH, delayed gastric emptying, decreased splanchnic blood flow, decreased gastrointestinal absorption surface areas and motility affect (A) in the older adult population. Decreased hepatic blood flow, decreased hepatic mass, and decreased activity of hepatic enzymes affect (B) in older adults. Decreased renal blood flow, decreased glomerular filtration rate, decreased tubular secretion, and decreased number of nephrons affects (C) in an older adult.

Dobutamine (Dobutrex) is an emergency drug most commonly prescribed for a client with which condition? A) Shock. B) Asthma. C) Hypotension. D) Heart failure.

D) Heart failure. Dobutamine is a beta-1 adrenergic agonist that is indicated for short term use in cardiac decompensation or heart failure (D) related to reduced cardiac contractility due to organic heart disease or cardiac surgical procedures. Alpha and beta adrenergic agonists, such as epinephrine and dopamine, are sympathomimetics used in the treatment of shock (A). Other selective beta-2 adrenergic agonists, such as terbutaline and isoproterenol, are indicated in the treatment of asthma (B). Although dobutamine improves cardiac output, it is not used to treat hypotension (C).

A client has a continuous IV infusion of dopamine (Intropin) and an IV of normal saline at 50 ml/hour. The nurse notes that the client's urinary output has been 20 ml/hour for the last two hours. Which intervention should the nurse initiate? A) Stop the infusion of dopamine. B) Change the normal saline to a keep open rate. C) Replace the urinary catheter. D) Notify the healthcare provider of the urinary output.

D) Notify the healthcare provider of the urinary output. The main effect of dopamine is adrenergic stimulation used to increase cardiac output, which should also result in increased urinary output. A urinary output of less than 20 ml/hour is oliguria and should be reported to the healthcare provider (D) so that the dose of dopamine can be adjusted. Depending on the current rate of administration, the dose may need to be increased or decreased. If the dose is decreased, it should be titrated down, rather than abruptly discontinued (A). Fluid intake may need to be increased, rather than (B). The urinary catheter is draining and does not need to be replaced (C).

A client with giardiasis is taking metronidazole (Flagyl) 2 grams PO. Which information should the nurse include in the client's instruction? A) Notify the clinic of any changes in the color of urine. B) Avoid overexposure to the sun. C) Stop the medication after the diarrhea resolves. D) Take the medication with food.

D) Take the medication with food. Flagyl, an amoebicide and antibacterial agent, may cause gastric distress, so the client should be instructed to take the medication on a full stomach (D). Urine may be red-brown or dark from Flagyl, but this side effect is not necessary to report (A). Photosensitivity (B) is not a side effect associated with Flagyl. Despite the resolution of clinical symptoms, antiinfective medications should be taken for their entire course because stopping the medication (C) can increase the risk of resistant organisms.

A client with coronary artery disease who is taking digoxin (Lanoxin) receives a new prescription for atorvastatin (Lipitor). Two weeks after initiation of the Lipitor prescription, the nurse assesses the client. Which finding requires the most immediate intervention? A) Heartburn. B) Headache. C) Constipation. D) Vomiting.

D) Vomiting. Vomiting, anorexia and abdominal pain are early indications of digitalis toxicity. Since Lipitor increases the risk for digitalis toxicity, this finding requires the most immediate intervention by

A client who has been taking levodopa PO TID to control the symptoms of Parkinson's disease has a new prescription for sustained release levodopa/carbidopa (Sinemet 25/100) PO BID. The client took his levodopa at 0800. Which instruction should the nurse include in the teaching plan for this client? A) Take the first dose of Sinemet today, as soon as your prescription is filled. B) Since you already took your levodopa, wait until tomorrow to take the Sinemet. C) Take both drugs for the first week, then switch to taking only the Sinemet. D) You can begin taking the Sinemet this evening, but do not take any more levodopa.

D) You can begin taking the Sinemet this evening, but do not take any more levodopa. Carbidopa significantly reduces the need for levodopa in clients with Parkinson's disease, so the new prescription should not be started until eight hours after the previous dose of levodopa (D), but can be started the same day (B). (A and C) may result in toxicity.

A client is being treated for hyperthyroidism with propylthiouracil (PTU). The nurse knows that the action of this drug is to A) decrease the amount of thyroid-stimulating hormone circulating in the blood. B) increase the amount of thyroid-stimulating hormone circulating in the blood. C) increase the amount of T4 and decrease the amount of T3 produced by the thyroid. D) inhibit synthesis of T3 and T4 by the thyroid gland.

D) inhibit synthesis of T3 and T4 by the thyroid gland. PTU is an adjunct therapy used to control hyperthyroidism by inhibiting production of thyroid hormones (D). It is often prescribed in preparation for thyroidectomy or radioactive iodine therapy. Thyroid-stimulating hormone (TSH) is produced by the pituitary gland, and PTU does not affect the pituitary (A and B). PTU inhibits the synthesis of all thyroid hormones--both T3 and T4(C).

A client has a continuous IV infusion of dopamine and an IV of normal saline at 50ml/hour. The nurse noes that the client's urinary output has been 20ml/hour for the last two hours. Which intervention should the nurse initiate? A. stop the infusion of dopamine B. change the normal saline to a keep open rate C. replace the urinary catheter D. notify the healthcare provider of the urinary output.

D.

An older client with a decreased percentage of lean body mass is likely to receive a prescription that is adjusted based on which pharmacokinetic process? A) Absorption. B) Metabolism. C) Elimination. D) Distribution.

D. A decreased lean body mass in an older adult affects the distribution of drugs (D), which affects the pharmacokinetics of drugs. Decreased gastric pH, delayed gastric emptying, decreased splanchnic blood flow, decreased gastrointestinal absorption surface areas and motility affect (A) in the older adult population. Decreased hepatic blood flow, decreased hepatic mass, and decreased activity of hepatic enzymes affect (B) in older adults. Decreased renal blood flow, decreased glomerular filtration rate, decreased tubular secretion, and decreased number of nephrons affects (C) in an older adult.

A client is admitted to the hospital for diagnostic testing for possible myasthenia gravis. The nurse prepares for intravenous administration of edrophonium chloride (Tensilon). What is the expected outcome for this client following administration of this pharmacologic agent? A) Progressive difficulty with swallowing. B) Decreased respiratory effort. C) Improvement in generalized fatigue. D) Decreased muscle weakness.

D. Administration of edrophonium chloride (Tensilon), a cholinergic agent, will temporarily reduce muscle weakness (D), the most common complaint of newly-diagnosed clients with myasthenia gravis. This medication is used to diagnose myasthenia gravis due to its short duration of action. This drug would temporarily reverse (A and B), not increase these symptoms. (C) is not a typical complaint of clients with myasthenia gravis, but weakness of specific muscles, especially after prolonged use, is a common symptom.

Which drug is used as a palliative treatment for a client with tumor-induced spinal cord compression? A) Morphine Sulfate (Duromorph). B) Ibuprofen (Advil). C) Amitriptyline (Amitril). D) Dexamethasone (Decadron).

D. Dexamethasone (D) is a palliative treatment modality to manage symptoms related to compression due to tumor growth. Morphine sulphate (A) is an opioid analgesic used in oncology to manage severe or intractable pain. Ibuprofen (B), a nonsteroidal antiinflammatory drug (NSAID), provides relief for mild to moderate pain, suppression of inflammation, and reduction of fever. Amitriptyline (C), a tricyclic antidepressant, is often prescribed for pain related to neuropathic origin and provides a reduction in opioid dosage.

Dobutamine (Dobutrex) is an emergency drug most commonly prescribed for a client with which condition? A) Shock. B) Asthma. C) Hypotension. D) Heart failure

D. Dobutamine is a beta-1 adrenergic agonist that is indicated for short term use in cardiac decompensation or heart failure (D) related to reduced cardiac contractility due to organic heart disease or cardiac surgical procedures. Alpha and beta adrenergic agonists, such as epinephrine and dopamine, are sympathomimetics used in the treatment of shock (A). Other selective beta-2 adrenergic agonists, such as terbutaline and isoproterenol, are indicated in the treatment of asthma (B). Although dobutamine improves cardiac output, it is not used to treat hypotension (C).

A client receiving Doxorubicin (Adriamycin) intravenously (IV) complains of pain at the insertion site, and the nurse notes edema at the site. Which intervention is most important for the nurse to implement? A) Assess for erythema. B) Administer the antidote. C) Apply warm compresses. D) Discontinue the IV fluids.

D. Doxorubicin is an antineoplastic agent that causes inflammation, blistering, and necrosis of tissue upon extravasation. First, all IV fluids should be discontinued at the site (D) to prevent further tissue damage by the vesicant. Erythema is one sign of infiltration and should be noted, but edema and pain at the infusion site require stopping the IV fluids (A). Although an antidote may be available (B), additional fluids contribute to the trauma of the subcutaneous tissues. Depending on the type of vesicant, warm or cold compresses (C) may be prescribed after the infusion is discontinued.

A client with giardiasis is taking metronidazole (Flagyl) 2 grams PO. Which information should the nurse include in the client's instruction? A) Notify the clinic of any changes in the color of urine. B) Avoid overexposure to the sun. C) Stop the medication after the diarrhea resolves. D) Take the medication with food.

D. Flagyl, an amoebicide and antibacterial agent, may cause gastric distress, so the client should be instructed to take the medication on a full stomach (D). Urine may be red-brown or dark from Flagyl, but this side effect is not necessary to report (A). Photosensitivity (B) is not a side effect associated with Flagyl. Despite the resolution of clinical symptoms, antiinfective medications should be taken for their entire course because stopping the medication (C) can increase the risk of resistant organisms.

A client is receiving clonidine (catapres) 0.1mg/24hr via transdermal patch. Which assessment finding indicates that the desired effect of the medication has been achieved? A. client denies recent episodes of angina B. Change in peripheral edema from +3 to +1 C. Client denies recent nausea or vomiting D. Blood pressure has changed from 180/120 to 140/70

D. Catapres acts as a centrally-acting analgesic and antihypertensive agent. D. indicates a reduction in hypertention

A clietn is being treated for hyperthyroidism with propylthiouracil (PTU). The nurse knows that the action of this drug is to A. decrease the amount of the thyroid-stimulating hormone circulating in the blood B. increase the amount of thyroid-stimulating hormone circulating in the blood C. increase the amount of T4 and decrease the amount of T3 produced by the thyroid D. inhibit synthesis of T3 and T4 by the thyroid gland

D. PTU is an adjunct therapy used to control hyperthyroidism by inhibiting production of thyroid hormones. It is often prescribed in prep for thyroidectomy or radioactive iodine therapy

Which change in data indicates to the nurse the desired effect of the angiotensin II receptor antagonist has been achieved A. Dependent edema reduced form +3 to +1 B. Serum HDL increased from 35 to 55mg/dl C. PUlse rate reduced from 150 to 90 beats/min D. Blood pressure reducedf rom 160/90 to 130.80

D. angiotensin II receptor antagonist (blocker), prescribed from treatment of HTN. The desired effect is a decrease in blood pressure.

receiving doxorubicin (Adriamycin) IV complains of pain at the insertion site and the nurse notes edema at the site

D/C IV fluids

The most important factor in an adverse drug reaction in the elderly population is

Declining renal fx

patient with hyperthyroidism taking inderal (propanalol)

Decreases pulse rate

A pt is prescribed Venlafaxine and the pt asks what the purpose of medication is, you should state

Depression and anxiety

Nurse is conducting DC teaching about anti-anxiety drug diazepam (valium)

Evaluate the ingredients of all over-the-counter drugs for alcohol content

Using bronchodilators before steroids for asthma teaching

Exhale completely, inhale deeply, hold breath for 10 seconds

Patient Dx with bipolar-how to know if meds are effective

Family states patient is doing better with manic phases

Client getting Tofranil (Imipramine)

Give medication at night

An RN is teaching a parent about administration of aderol to treat their child's ADD

Give the dose in the morning b4 school. (amphetamine - child needs it to concentrate)

A pt prescribed CCB Diltiazem to treat hypertension

Grapefruit juice

Which method of medication administration provides the client with the greatest first pass effect

oral

Peptic ulcer med-what action

Histamine 2 agonist

RN is prepared to administer Epinephrine to pt that has a severe allergic reaction

IV - no first pass effect

Nursing instruction most important for patient on Zyloprim

Increase fluid intake

Pt is prescribed Cimetidine (histamine 2 antagonist) to treat gastric ulcer

Inhibit the action of histamine at receptor sites and block gastric acid secretion

Parkinson's disease

Levodopa/Carbidopa

The healthcare provider prescribes naproxen (Naproxen) twice daily for a client with osteoarthritis of the hands

another type of non- steroidal anti-inflammatory drug may be indicated

A pt receives Dopamine for shock (hypotention, vascular collapse, comatose)

Mean arterial pressure

•-ide

oral hypoglycemics

Candidiasis is treated with?

Nystatin

Medication calculation-patient weighs equal to 16kg-order for Tamiflu 45mg BID

Must round up-answer is 3.8ml

Pt is concerned about developing Alzheimer's disease

Naproxen (NSAID's)

Nurse should observe most closely for drug toxicity when a client receives med that has what characteristic

Narrow therapeutic index

Pt asks the nurse if glipizide (Glucotrol) is oral insulin

No, it is not oral insulin and can be used only when some Beta cell function is present

Amphetamines can cause growth suppression in children

Provide snack or meal b4 giving med (because amphetamines suppresses appetite)

Digoxin has 36-48 hr half life - because of the length of half life, the RN expects to be dosing this medication

Once a day

Pyridium for bladder infection

Orange/red/pink urine

•-lam

anti-anxiety agents

Lipitor (statins) , can be tame at what time?

PM only-no grapefruit juice

•-pam

anti-anxiety agents

•-dine

anti-ulcer agents (H2 histamine blockers)

•-vir

anti-virais

•-micin

antibiotics

Pt on Lithium

Polydypsia (increased thirst), slurred speech, and fine hand tremors

•-mycin

antibiotics

heparin Tx for a pulmonary embolism a client is being discharged with a Rx for warfarin (Coumadin)

Prothrombin Time (PT/INR)

Which dosing schedule should the nurse teach the client to observe for a controlled release oxycodone Rx

Q 12H

Patient with dysrhythmia is to receive procainamide (Pronestyl) in a 4 divided dose over the next 24 hours

Q 6Hr

Which dosing schedule should the nurse teach the client to observe for a controlled-release oxycodone prescription

Q12H

Know the insulins and their peak/onset (there are several Qs about this in different formats

Rapid-Lispro (Humalog) and Aspart (Novolog) Onset: 5-15 minutes Peak: .75-1.5 hours Short acting- regular (humulin) Onset: 30-60 minutes Peak: 2-3 hours (IV ok) Intermediate acting- NPH Onset: 1-2 hours Pea:k 6-12 hours Long acting- Glargine (lantus) Onset: 1.1 hour Peak: 14-20 hours (DO NOT MIX)

182.) A client with angina pectoris is experiencing chest pain that radiates down the left arm. The nurse administers a sublingual nitroglycerin tablet to the client. The client's pain is unrelieved, and the nurse determines that the client needs another nitroglycerin tablet. Which of the following vital signs is most important for the nurse to check before administering the medication? 1. Temperature 2. Respirations 3. Blood pressure 4. Radial pulse rate

Rationale: Nitroglycerin acts directly on the smooth muscle of the blood vessels, causing relaxation and dilation. As a result, hypotension can occur. The nurse would check the client's blood pressure before administering the second nitroglycerin tablet. Although the respirations and apical pulse may be checked, these vital signs are not affected as a result of this medication. The temperature also is not associated with the administration of this medication.

134.) A nurse reinforces instructions to a client who is taking levothyroxine (Synthroid). The nurse tells the client to take the medication: 1. With food 2. At lunchtime 3. On an empty stomach 4. At bedtime with a snack

Rationale: Oral doses of levothyroxine (Synthroid) should be taken on an empty stomach to enhance absorption. Dosing should be done in the morning before breakfast. **Note that options 1, 2, and 4 are comparable or alike in that these options address administering the medication with food.**

206.) A client is receiving baclofen (Lioresal) for muscle spasms caused by a spinal cord injury. The nurse monitors the client, knowing that which of the following is a side effect of this medication? 1. Muscle pain 2. Hypertension 3. Slurred speech 4. Photosensitivity

Rationale: Side effects of baclofen include drowsiness, dizziness, weakness, and nausea. Occasional side effects include headache, paresthesia of the hands and feet, constipation or diarrhea, anorexia, hypotension, confusion, and nasal congestion. Paradoxical central nervous system excitement and restlessness can occur, along with slurred speech, tremor, dry mouth, nocturia, and impotence. **Option 3 is most closely associated with a neurological disorder**

Pt receives a drug that blocks adrenergic receptors Orthostatic hypotension (no BP to the brain)

Reflex tachacardia is caused by hypotention

Rifampin for TB

Rusty-orange/red colored urine and body fluids

Phenobarbitol

Seizures

A breast-feeding pt is prescribed an antimicrobial medication

Take immediately after breast-feeding

Which assessment best determines the effectiveness of Sumatriptan (triptans are used for treatment of Migraines so...

Termination of the migraine

Chlamydia

Tetracycline

Upon admission to the emergency center, an adult client with acute status asthmaticus is prescribed this series of medications. In which order should the nurse administer the prescribed medications? (Arrange from first to last.) A) Prednisone (Deltasone) orally. B) Gentamicin (Garamycin) IM. C) Albuterol (Proventil) puffs. D) Salmeterol (Serevent Diskus).

The best sequence of administration is (C, D, A and B). Status asthmaticus is potentially a life-threatening respiratory event, so albuterol (C), a beta2 adrenergic agonist and bronchodilator, should be administered by inhalation first to provide rapid and deep topical penetration to relieve bronchospasms, dilate the bronchioles, and increase oxygenation. In stepwise management of persistent asthma, a long-action bronchodilator, such as salmeterol (Serevent Diskus) (D) with a 12-hour duration of action should be given next. Prednisone (A), an oral corticosteroid, provides prolonged anti-inflammatory effects and should be given after the client's respiratory distress begins to resolves. Gentamicin (B), an antibiotic, is given deep IM, which can be painful, and may require repositioning the client, so should be last in the sequence.

Administration of schedule IV drug, the RN understands

The drug has acceptable medical application with low potential for abuse

Preparing to administer a drug to a pt with an infection

The drug will destroy the microorganism

Pt with mild symptoms of Alzheimer's is prescribed Donepezil (Aricept)

The drug will stop the damage to the neurons in my brain

Pt with schizophrenia is prescribed chlorpromazine (Thorazine - first generation antipsychotic) oral concentrate

The medication may cause excessive salivation And No direct skin contact (both correct - not a good question)

A 24 yo female gets Triazolam (Halcion) for insomnia at home (remember to use the process of elimination)

The medication will not alter my breathing"

Best time to give patient Abx (I think)

Time was like 1000, 1400, 1200, and 0400...best to give around the clock

Vasopressin

Vasoconstrictor

Pt states that he's on Oxycodone and the dose that he currently receives does not provide the same pain relief

Tolerance

Which best assessment best determines the pt is developing tardive dyskinesia

Twisting, writhing, worm-like movements of tongue

In evaluating the effects of lactulose (Cefulac)

Two or three soft stools per day

•-statin

antihyperlipidemics

older client with a decreased percentage of lean body receives a Rx that is adjusted based on which pharmacokinetic process?

distribution

RN doesn't understand why a pt is to receive a prescribed med

Verity the reason with the prescribing healthcare provider for use

When assessing an adolescent who recently overdosed on acetaminophen (Tylenol), it is most important for the nurse to assess for pain in which area of the body

abdomen

•-pril

ace inhibitors

prescriptions for morphine sulphate 2.5 mg IV q6h and ketorolac (Toradol) 30 mg IV q6h

administer both medications according to the prescription

taking hydromorphone (Dilaudid) PO q4h at home. Following surgery Dilaudid IV q4h PRN and but butorphanol tartrate (Stadol) IV q4h PRN. The pt received a dose of the drug 4 hours ago and is again requesting pain medication

administer only the Dilaudid q4h PRN for pain

dose of isosorbide dinitrate (Imdur) is increased from 40 mg to 60 mg PO daily

administer the 60 mg dose of Imdur and a PRN dose of acetaminophen (Tylenol)

•-mide

diuretics

•-zide

diuretics

pt states "I am allergic to penicillin"

cephalosporins

Following sublingual nitroglycerin to a pt experiencing an acute anginal attack

client states chest pain is relieved

A client with coronary artery disease who is taking digoxin (Lanoxin) receives a new prescription for atorvastatin (Lipitor). Two weeks after initiation of the Lipitor prescription, the nurse assesses the client. Which finding requires the most immediate intervention? A) Heartburn. B) Headache. C) Constipation. D) Vomiting.

d. Vomiting, anorexia and abdominal pain are early indications of digitalis toxicity. Since Lipitor increases the risk for digitalis toxicity, this finding requires the most immediate intervention by the nurse (D). (A, B and C) are expected side effects of Lipitor.

postoperative and has been receiving a continuous IV infusion of mepreidine (Demerol) 35 mg per hr for 4 days and has a PRN Rx for Demerol for 100 mg PO q3h

decrease the IV infusion rate of the mepreidine (Demerol) per protocol

admitted to the hospital for Dx testing for possible myasthenia gravis prepares for IV administration of edrophonium chloride (Tensilon)

decreases muscle weakness

Which drug is used as a palliative treatment for a client with tumor-induced spinal cord compression

dexamethasone (Decadron)

assessing a client that is experiencing anaphylaxis from an insect sting

epinephrine

hyperlipidemia receives a prescription for niacin (Niaspan)

expected duration of flushing

Dobutamine (Dobutrex) is an emergency drug most commonly prescribed for which condition

heart failure

medication described as a Beta1 agonist is most commonly prescribed for a client with which condition

heart failure

A pt with HF is getting Furosemide (Lasix) (loop diuretic is K+ wasting) Oranges, spinach, and potatoes

high in potassium

A peak and trough level must be drawn for a client receiving antibiotic therapy

immediately before the next antibiotic dose is given

instruction to give to a female who just received a Rx for oral metronidazole (Flagyl) for treatment of trichomonas vaginalis

increase fluid intake, especially cranberry juice avoid drinking alcohol while taking this medication use condoms until Tx is completed ensure that all sexual partners are Tx at the same time

A client is being treated for hyperthyroidism with propylthiouracil (PTU)

inhibit synthesis of T3 and T4 by the thyroid gland

•-nuim

neuromuscular blockers

admitted to the coronary care unit with a medical Dx of acute myocardial infarction

nitroglycerin

osteoarthritis receives a new prescription for celecoxib (Celebrex) orally for symptom management. The nurse notes the client is allergic to sulfa

notify the healthcare provider

continuous IV infusion of dopamine (Intiropin) and an IV of normal saline at 50 ml/hr the nurse notes that the clients urinary output has been 20 ml/hr for the last 2 hours

notify the healthcare provider of the urinary output

assessing the effectiveness of high dose aspirin therapy for an 88-year-old client with arthritis

notify the healthcare provider of this finding immediately

receiving ampicillin sodium (Omnipen) for a sinus infection

rash

spironolactone (Aldactone) for heart failure is Rx

refrain from eating foods high in potassium

•-sone

steroids

43-year-old female receiving thyroid replacement hormone following a thyroidectomy

tachycardia and chest pain

A client is being treated for osteoporosis with alendronate (Fosamax) and the nurse has completed discharge teaching regarding medication administration

take medication, go for a 30 minute morning walk, then eat breakfast

giardiasis is taking metronidazole (Flagyl) 2 grams PO

take the medication with food

The healthcare provider prescribes naloxone (Narcan) for a client in the emergency room

the clients' respiratory rate is 16 breaths/min

reviewing the use of the patient controlled analgesics (PCA) pump with a client in the immediate postoperative period The client will receive Morphine 1 mg IV per hour basal rate with 1 mg IV every 15 minutes per PCA to total 5 mg IV maximally per hour

the rate and depth of the clients' respirations

A category X drug is prescribed for a young adult female client

use a reliable form of birth control

rheumatoid arthritis takes ibuprophin (Motrin) 600 mg PO 4 times a day. To prevent gastrointestinal bleeding, misoprostol (Cytotec) 100 mcg PO is prescribed

use contraception during intercourse

preparing the 0900 dose of losartan (Cozaar) an angiotension-II receptor blocker (ARB) for a client with hypertension and heart failure

withhold the scheduled dose

The nurse is transcribing a new prescription for spironolactone (Adactone) for pt who receives an angiotension-converting enzyme (ACE) inhibitor

verify both prescriptions with the healthcare provider

coronary artery Dz. is taking digoxin (Lanoxin) receives a new prescription for atorvastatin (Lipitor)

vomiting

Which symptoms are serious adverse affects of Beta-adrenergic blockers such as propranolol (Inderal)

wheezing, hypotension and AV block


Kaugnay na mga set ng pag-aaral

A202 Chapter 2 Job-Order Costing

View Set

Chapter 29 - Disorders of Children and Adolescents (Questions)

View Set

System's Analysis and Design Ch. 8-11

View Set

U.S Government- Unit 1: The History of Civics and Government

View Set